Você está na página 1de 258

1.

The average growth of head circumference in (centimeters/ month) in the


age from
8 – 9 months is as follows:
A. 2.00 cm /month.
B. 1.50 cm /month.
C. 1.00 cm /month.
D. 0.50 cm /month.
E. 0.25 cm /month.

a2- Aortic stenosis is characterized by all of the following features EXCEPT:


(A) a thickened aortic valve
(B) frequent symptoms
(C) ventricular hypertrophy
(D) a harsh systolic ejection murmur
(El a systolic ejection click

3) A newborn male has excess abdominal skin, deficiency of the abdominal


musculature, and cryptorchidism. MOST likely etiology of these findings is:
chronic amniotic fluid leakage during
pregnancy
B. early urethral obstruction
C. extrophy of the bladder
D. polycystic kidneys
E. renal agenesis

4) Upon physical examination of a 3-year-old girl who is new to your practice,


you note a continuous grade 2 to grade 3 murmur at the upper right sternal
border while she is sitting. In the supine position, you note only a grade 2 low-
pitched systolic murmur at the apex. Measurements of blood pressure, pulses,
and precordial palpation as well as auscultation of the first and second heart
sounds are normal.
Of the following, the MOST appropriate next step is to
A. reassure the parents about the benign prognosis (Still`s murmur)
B. request a cardiology consultation
C. request chest radiography
D. request echocardiography
E. request electrocardiography
5. A 6-week-old infant has a history of noisy breathing. The noise was first noted
shortly after birth, is inspiratory in nature, is worse now that the infant has a
viral respiratory illness, and remits almost completely when the child is asleep.
What is the most likely etiology of this child's noisy breathing?
(A) Asthma
(B) Bronchopulmonary dysplasia
(C) Cystic fibrosis
(D) Laryngomalacia
(E) Tuberculosis

6) A 6-month-old usually healthy girl has a fever of (39.8°C). There are no


accompanying symptoms. A urinalysis shows "1 +" leukocyte esterase, 10
WBC/HPF, and moderate bacteria.
What is the most reasonable step to be taken next ?
(A) Obtain a renal ultrasound
(B) Begin broad-spectrum, intravenous antibiotics
(C) Obtain a dimercaptosuccinic acid renal scan
(D) Obtain a urine culture by catheter and consider starting antibiotics
(E) Check for costovertebral angle tenderness, and if absent, exclude the
diagnosis of urinary tract infection

7. All of the following conditions are characterized by hypochromic, microcytic


red blood cells EXCEPT :
(A) iron deficiency anemia
(B) thalassemia major
(C) thalassemia minor
(D) Hereditary Spherocytosis
(E) Anemia of chronic disease
8. All of the following disorders are associated with prolonged bleeding time
EXCEPT :
(A) hemophilia A
(B) von Willebrand's disease
(C) aspirin-induced thrombocytopathia
(D) Bernard-Soulier syndrome
(E) idiopathic thrombocytopenic purpura

9) Obligatory vaccination during the first year of life in Palestine includes all
except:
A. BCG
B. DPT
C. Polio-vaccine
D. Meningococcal vaccine
E. Hemophilus influenzae vaccine

10. In cephalhematoma one of the following statements is correct:


A. SOFT EDEMA
B. Aspiration is the best treatment.
C. Hyperbilirubinemia may complicate the picture.
D. All of the above.
E. None of the above

11) Daily vitamin D requirement in a 4 months old infant is:


A. 60.000 IU
B. 400 IU
C. 1500 IU
D. 100 IU
E. 1000 IU

12) Kwashiorkor has all the following except:


A. Edema
B. Anorexia
C. Color changes of hair
D. Spasticity
E. Skin changes
13) Two years old infant with noisy breathing, barking cough and inspiratory
distress is suffering from:
A. Pneumothorax
B. viral croup
C. Heart failure
D. Bronchial asthma
E. Pleural effusion

14) Compared with human milk, cow milk formula is MORE likely to contain a(n)
A. greater concentration of essential fatty acids
B. higher protein concentration
C. increased lactose content
D. lower calcium-phosphorus ratio
E. lower iron concentration
15. A 6-year-old girl can write her name and can count 10 objects.
Of the following, the MOST likely additional activity of which she is capable is:
A. hitting a baseball
B. knowing her right hand from her left
C. making a simple meal
D. printing neatly in small letters
E. sounding out words while reading

16.Which of the following laboratory findings is unusual in patients with simple


( nutritional ) rickets ?
(A) Aminoaciduria
(B) Hyperphosphaturia
(C) Elevated levels of serum alkaline phosphatase
(D) Hypercalciuria
(E) Hypophosphatemia

17.A 4 month old with vitamin D deficiency rickets would be expected to show
all of the following except:
A. Craniotabes
B. Bow legs
C. Rosary
D. Low serum phosphate level
E. High alkaline phosphate levels

18.All of the following are true about kwashiorkor except:


A. The presence of edema
B. Rash in sun exposed areas
C. Hypoalbuminemia
D. Weak muscles
E. An increased susceptibility to infection

19. In the Palestinian National Immunization Program, all of the following are
correct EXCEPT:
A. Salk vaccine is given at one and two months of age.
B. Hep. B vaccine is given at birth, one month and six months of age.
C. A booster dose of DPT is given at six years of age.
D. MMR vaccine is given at age of 15 months (12 & 18)
E. Rubella vaccine is given to girls at age of 12 years.
20. all are causes of hypovolemic hyponatremia except :
Hypothrodism (euovolumic)

21. The arthritis of acute rheumatic fever usually:


A. Is monoarticular
B. Heals without deformity
C. Appears after the fever subsides
D. Is seen only in patients with concurrent carditis
E. Involves large and small joints equally

22. a 12 year old girl is at the 5th percentile for height .she is an otherwise
healthy child. The bone age is greater than two and half standard deviation
below the chronologic age. Of the following, the most likely cause of patient’s
short stature is
A. Constitutional growth delay
B. Genetic short-stature
C. Growth hormone deficiency
D. Hypothyroidism
E. Ulcerative colitis

23- A mother brings her previously healthy 2-year-old son to the emergency
department because he looked pale when he woke up this morning. She also
reports fever and decreased activity over the last few days. He was diagnosed
with a U T I and given trimethoprim-sulfamethoxazole 3 days ago. On physical
examination, he is markedly pale and appears jaundiced. His H R is 170 b/min,
and his R R is 30 b/min. The rest of his physical examination findings are
unremarkable. Lab .evaluation reveals a hemoglobin concentration of 5 g/dL Of
the following, the MOST likely cause of this boy's anemia is:
A. a plastic anemia
B. glucose-6-phosphate dehydrogenises deficiency
C. hereditary spherocytosis
D. sickle cell hemolytic crisis
E. transient erythroblastopenia of childhood

24- Which of the following would be expected in a 6-month-old child with a


large ventricular septal defect ?
A) cyanosis
(B) an enlarged heart
(C) a continuous cardiac murmur
(D) decreased pulmonary vasculature on roentgeno-gram
(E) evidence of predominantly right ventricular hypertrophy on ECG

25. iron deficiency in a child who has unrepaired cyanotic congenital heart
disease is MOST likely to increase the risk for
A. congestive heart failure
B. digoxin toxicity
C. osteoarthropathy
D. pulmonary hemorrhage
E. stroke

A 15-yr-old-girl is admitted with a 5-day history of worsening fever, left flank


pain, and vomiting. Physical examination reveals an ill-appearing, dehydrated
adolescent. Pulse rate is 110/min; temperature is 40°C. Left costovertebral angle
tenderness is present. Serum sodium is 131 mmol/L, potassium 6.7 mmol/L,
chloride 108 mmol/L, and bicarbonate 15 mmol/L. The most likely diagnosis is:
A - Addison's disease
B - Ingestion of high potassium-containing foods
C - Distal renal tubular acidosis
D - Acute pyelonephritis
E - Congenital adrenal hyperplasia

Urine dipstick testing of a specimen obtained from a febrile 4-yr-old child with
acute viral gastroenteritis shows specific gravity 1.030, pH 5.0, 2+ proteinuria,
and no blood cells. The most likely cause of the patient's proteinuria is:
A - Transient proteinuria
B - Nephrotic syndrome
C - Orthostatic proteinuria
D - Acute glomerulonephritis
E - Chronic glomerulonephritis

28. Which one of the following statements is true about shigellosis?


A. Neurologic complications occur in about 40% of patients.
B. Animal reservoirs include poultry and shellfish.
C. Neomycin, orally, is the antibiotic of choice.
D. Bacteremia is common.
E. The disease is rare in institutions for the retarded.
Regarding the maintenance of diet in diabetes mellitus, the following are
recommended:
A Six main meals per day.
B A high fibre diet.
C Use of highly refined sugars.
D A high protein diet.
E Increasing the medium chain fat contents of the diet

A child presents at 18 months of age with hypoglycaemia and a convulsion. The


following suggest a diagnosis of ketotic hypoglycaemia:
A. Macrosomia
B. Absence of ketones in the urine
C. 3cm liver
D. reduced plasma insulin levels
E. Elevated serum allanene

Recurrent oral ulcer+diarrhea_ Crohn’s disease

32. A 6 month-old infant can do all of the following except:


A. Sits with support.
B. Turns over.
C. Reaches and holds an object.
D. Creeps on abdomen (9 months).
E. Bears weight on legs when held on standing position

33. A developmentally normal child who is just able to walk with one hand held
and says mama and dada with meaning is probably what age?
A. 8 months
B. 10 months
C. 12 months
D. 14 months
E. 16 months

34. A baby is jaundiced at 12 hours old. The causes to consider include all of the
following except:
A-Congenital infection.
B-Haemolytic diseases.
C-Congenital spherocytosis.
D-Biliary atresia.
E-ABO incompatibility

35. A 14-year-old girl awakens with a mild sore throat, low-grade fever,and a
diffuse maculopapular rash. During the next 24 h, she develops tender swelling
of her wrists and redness of her eyes. In addition, her physician notes mild
tenderness and marked swelling of her posterior cervical and occipital lymph
nodes. Four days after the onset of her illness, the rash has vanished. The most
likely diagnosis of this girl’s condition is
a. Rubella
b. Rubeola
c. Roseola
d. Erythema infectiosum
e. Erythema multiforme

36. ) A 14-year-old boy is evaluated for short stature. He has no significant past
medical history and is considered otherwise healthy by his parents. He eats a
normal diet and has regular meals. His height and weight have been consistently
at the 5th percentile since early childhood. His physical examination is normal,
with genitalia at Tanner stage 3. Which of the following is the most likely
laboratory finding for this boy?
A. Bone age that is equivalent to chronologic age
B. Decreased complement C3 level
C. Decreased serum albumin concentration
D. Decreased thyroid stimulating hormone
E. Increased serum creatinine concentration

37) A 6-day-old girl who was born at home is being evaluated for bruising and
gastrointestinal bleeding. Laboratory findings include partial thromboplastin
time and prothrombin time, greater than 2 minutes; serum bilirubin, 4.7 mg/dL;
alanine aminotransferase, 18 mg/dL; platelet count, 330,000/mm3; and
hemoglobin, 16.3 g/dL. Which of the following is the most likely cause of her
bleeding?
A. Factor VIII deficiency
B. Factor IX deficiency
C. Idiopathic thrombocytopenic purpura
D. Liver disease (no jaundice, normal liver enzymes!!!)
E. Vitamin K deficiency
38) A newborn girl is noted to be drooling saliva, and she chokes violently when
she is first fed. On physical examination, she is found to have abdominal
distention and an imperforate anus. There is no fistula to the perineum or
vagina. Examination of the urine reveals no meconium in it. Echocardiogram
and renal sonogram are reported as negative for other congenital defects. X-ray
films show abundant gas in the gastrointestinal tract. Pictures taken with a
metal marker taped to the anus, and the baby hanging upside down, show that
there is a significant distance (2.5 cm) between the blind end of the rectum and
the anal marker. Before a diverting colostomy is performed, which of the
following steps should be taken?
A. A soft nasogastric tube passed, and x-rays taken
B. Barium injected through the anal dimple
C. Barium swallow
D. Nasogastric suction for at least 24 hours
E. Surgical exploration of the perineum to see whether a primary repair is
possible

39. A 15-year-old girl with type 1 diabetes mellitus presents to her primary care
doctor for a routine check up. Perusal of her blood sugar chart indicates that her
recorded blood glucose levels are routinely between 120 and 150 mg/dL before
breakfast, dinner and bedtime, with the normal being 116 mg/dL. She is on NPH
and regular insulin. Which of the following is the next appropriate step?
A. Decrease the dosage of NPH
B. Decrease the dosage of Regular insulin
C. Make no changes and obtain a glycosylated Hb test
D. Increase the dosage of NPH
E. Decrease the dosage of Regular insulin

40. A 12-year-old girl with a history of asthma has been admitted to intensive
care units two times in the past and has had three emergency department visits
during the past 12 months. Her only medication is inhaled albuterol as needed,
and she uses it two to three times a day. She has nocturnal symptoms about
two times a week. She is free of symptoms now but reports that she gets short
of breath easily. Which of the following pharmacologic interventions is most
appropriate?
A. Anticholinergic agent
B. Cromolyn sodium
C. Inhaled corticosteroid
D. Long-term bronchodilator
E.Nedocromil sodium

41. An 11-year-old boy presents with fever and sore throat. A rapid-strep test
confirms streptococcal pharyngitis. He is leaving for a summer camp in 2 days.
In the past, he has had problem finishing the whole course of antibiotic
treatment. Which of the following is the best treatment for his streptococcal
pharyngitis?
A. A single dose of benzathine penicillin G intramuscularly
B. A single dose of ceftriaxone intramuscularly
C. A single dose of procaine penicillin G intramuscularly
D. Erythromycin orally for 5 days
E. Penicillin V orally for 5 days

42. A 3-week-old boy presents to the physician's office with a 1-week history of
forceful, projectile vomiting. He has been vomiting after almost every feeding.
The vomitus contains mostly undigested formula and is non-bilious. On
examination, his oral mucosa is dry, his anterior fontanel appears to be
depressed, and his capillary refill is 3-4 seconds. An abdominal examination
reveals an olive-sized mass in the epigastrium. Which of the following
electrolyte findings will most likely be seen?
A. Hypochloremic metabolic acidosis
B. Hypochloremic metabolic alkalosis
C. Normal electrolytes
D. Respiratory acidosis with metabolic compensation
E. Respiratory alkalosis

43. A term male infant is found to be cyanotic shortly after birth and requires
endotracheal intubation. On physical examination, his blood pressure is 68/34
mm Hg (equal in all four extremities), pulse is 180/min, and respirations are
32/min. His precordium is dynamic, has a grade III systolic murmur, and a single
S2. Chest radiography shows a normal heart size and increased pulmonary
vascular markings. An arterial blood gas on an FiO2 of 100% shows pH 7.34;
PaCO2, 47 mm Hg; PaO2, 46 mm Hg. Which of the following diagnoses is most
consistent with these findings?
A. Atrial septal defect
B. Hypoplastic left heart syndrome
C. Patent ductus arteriosus
D. Tetralogy of Fallot
E. Total anomalous pulmonary venous return

44. A 3 year old male has sudden onset of dry cough with a small amount of
bright red blood produced when he coughs. He has had no fevers, runny nose,
or vomiting. In the Emergency Department a chest x-ray shows hyperexpansion
of the right lung and clear lung fields. Which of the following is the most likely
etiology of this child's symptoms?
A. Bacterial pneumonia
B. Cystic fibrosis
C. Foreign body aspiration
D. Pulmonary arteriovenous malformation
E. Tuberculosis

45. All are causes of hyperkalemia except: b-adrenergic agonist


46. bloody diarrhea + high fever: shiglla
47. all symptoms of hypoglycemia except: loss of appétit
48. in second year of life , length gain : 10-12 cm

49.. A 6-month-old boy developed a weepy, crusted dermatitis around the eyes,
nose, mouth, diaper area, hands, and feet about 4 weeks after being weaned
from human milk to formula. He is listless, recently developed diarrhea, and has
stopped gaining weight. In addition to the dermatitis, he has sparse hair that is
fine and lightly pigmented.
Of the following, this constellation of findings is MOST consistent with a
deficiency in
A. copper
B. thiamine
C. vitamin A
D. vitamin C
E. zinc

50. You are evaluating a 4-year-old boy for bowing of the legs and short stature.
Findings include: creatinine, 0.4 mg/dL; calcium, 9.2 mg/dL; phosphorus, 2.5
mg/dL; (normal value: 2.4 to 4.1 mg/dL) alkaline phosphatase, 950 IU/L; intact
parathyroid hormone, 40 pg/mL (normal, 10 to 60 pg/mL); 25-hydroxyvitamin D,
30 pg/mL (normal, 12 to 40 pg/mL); 1,25-dihydroxyvitamin D, 30 pg/mL
(normal, 25 to 45 pg/mL); and urinary tubular reabsorption of phosphorus, 50%
(normal, >85%).
The MOST likely cause of these findings is
A. familial x linked hypophosphatemic rickets
B. hypophosphatasia
C. renal osteodystrophy
D. vitamin D-deficiency rickets
E. vitamin D-dependent rickets

51. The parents of a healthy term baby ask you at the 12-month health
supervision visit what they should expect of the baby developmentally by the
15-month visit.
Of the following, the milestone MOST likely to be met by 15 months of age is
A. drawing a circle
B. drinking from a cup
C. having a vocabulary of at least 50 words
D. throwing a ball overhand
E. walking well without tripping

52. what is true about vesicourethral reflux: 30% of UTIs in children are caused
by VUR

53.All true about PSAGN except: normal C3


54. All true about PSAGN except: massive edema
55. . All true about MCNS except: to relieve edema, give oral diuretic
56. All true about MCNS except: hypertension is characteristics
57. compared to intrinsic ARF, What is true about prerenal ARF: urine Na=10
mEq /L (<20)
58. .All are present in chronic renal failure except: macrocytic anemia
59. what is true about Von-willbrand disease: mucocutanous bleeding is
prominent symptom
60. Respiratory synctial virus increase risk of: asthma
64. young boy, his height ,weight at 3rd percentile what is true: 97% of same age
taller than him
65. ride tricycle, what is age: 3yrs
66. down syndrome: increase risk of hypothyroidism
67. case about trisomy 13, associated with: holoposencephaly
68. measles ,all are true except: meningitis as complication
69. subacute sclerosing panencehalitis, all true except: ‫كل الخيارات كانت صحيحة‬
:‫موجودة بالكلمة يف الساليد ربما قصدهم‬
Male are affected twice as often as female

70. mump, all true except: 80-90% subclinical


(30-40% is true)
71.about varicella vaccine, all true except: contraindicated in isolated humoral
immunodeficiency.
72. risk factor of listeria meningitis: T-lymphocyte defect.
73. treatment of listerosis: ampicllin with or without gentamicin
74. all causes of acute upper airway obstruction except: diphtheria,
laryngomalica (not sure)
75. case about bacterial trachetitis, what is organism: S. aureus
76.all true about forgein body aspiration except: hyperinflation of opposite lung
77. adolescent male, sudden onset of cough, dyspnea, cyanosis: pneumothorax
78.all are indication of sweat test except: recurrent cystitis

79. 2,5 yrs , diarrhea 3-5 times daily for 2 mos with normal height, normal
weight, stool with undigested carrot: toddler diarrhea, malabsorption,
celiac(most likely)

80. young boy with watery diarrhea, vomiting for 3 days, the mother sought
medical help, describe ORS, given without improvement, after two days
convulsion, what is the cause: Metabolic acidosis, hyponatremia dehydration,
isonatremia, hypernatremia deh (not sure)

81. what is true about asthma: increased functional residual capacity


82. compared to ASD, atrioventicular septal defect associated with: increase risk
of early development of pulmonary hypertension
83. about secondum ASD, what is true: transvenous device closure
84. PDA 10 yrs, what is true: early operative, IE prophylaxis until procedure
performed(not sure)
85. case 2yrs about coarctation of aorta, what you see: BP in upper limb> lower
limb
86. case about tetralogy of fallot (emergency), what is your step: give oxygen
and morphin
87. case ,cyanosis, weak pulse, prominent Rt ventricle, what your diagnosis:
hypoplastic left heart syn.
88. all true about RDS except: improve within first 24 hrs

89. 10 min baby(CS), grunting after 2-5 min, stop and baby is vigorous, what is
your step: obtain chest x-ray, observe 10 min if repeat admit to NCU (not sure)

90. Hydroxurea in sickle cell anemia: increase hemoglobulin F


91. all in cretinism except: diarrhea
92. what is true about physiological jaundice: rise in serum bilirubin<
5mg/dl/day, clinical jaundice persist at least 1wks(not sure)
93. what is true about subgleal hemorrhage: cross suture line
94. what is clinical manifestation of carditis in RF: systolic murmur at apex

95. all true about erb's palsy except: may be associated with horner syn.

96. case about complex partial seizure (lip masking, blank stare, lack of
responsiveness), what is treatment: carbamazepine

97. all are true about infantile spasm except: occur in first 6 wks of life
98. what is true about simple febrile convulsion: occur in early stage of illness

99. all are true about neonatal infection except: when meconium is stained, you
suspect E.coli

100. case (girl look ill, peteicha ,cell line decreased), what is your diagnosis:
marrow failure

5th Year Medical Students – Final Paediatrics Exam 2015


Dosens of resources, including: Nelson review, Lange Paediatrics, Pretest, Case
File Paediatrics, Previous exams, USMLE, new questions specially those about
growth & development (Dr. Anwar)
1. What is false about cranial nerves? (Trochlear nerve moves the eye
superiorly)
2. Hemolysis in premature baby is associated with: (Vit. E def)
3. What reflex is difficult to demonstrate in a 2 week old kid? Moro, crossed
Adductor reflex, planter grasp, palmar grasp, (parachuting)
4. Known complication in HSP? (Nephritis), thrombocytopenia, faciitis, serositis,
5. A case of Turner syndrome, what is the most common cause of HTN?
Essential HTN, renal failure, (coarctation of aorta), ovarian failure, renal
artery stenosis
6. What is the calorie requirement in a 2 month old child? 40, 60, 80, 100, 200
7. Sore throat treated with ampicillin then rash appeared: (Infectious
mononucleosis)
8. 3 days fever then subsided followed by 3 days rash: (Roseola infantum)
9. Aplastic crisis in hemolytic disorders: (Parvovirus B 19)
10. Diarrhea caused by ETEC causing: Severe diarrhea and dehydration, self-
limited diarrhea? - not sure about the answer 
11. All of the following can cause false negative sweat chloride test except:
(edema)
12. Q about Type II DM familial pattern compared with Type I DM
13. Nelson review: The breast-fed infant of a strict vegan may develop which
vitamin deficiency if the mother is not receiving supplements of this vitamin?
K, B6, B12, Folate, Biotin
14. Meningitis complications?
15. Q about treatment of acute bronchiolitis?
16. Lesion is a neonate with eosinophilic aspirate: (Erythema toxicum
neonatorum)
17. What is the right about mumps: Occur in parotid gland only,
(Menignioencephalitis may precede parotid enlargement)

18. Baby 5 days after born at home come with mucocutaneous bleeding ,
pt:20 , ptt:27 , platelet 200,000 what is the cause
Von-wallebrand disease, Factor 8 deficiency, (Vitamin k deficient), Platelet
dysfunction
Thrombocytopenia
19. Apgar score for active crying baby, with grimace, lower limbs cyanotic:
6,7,8,9,10
20. Child with toxic appearance, drooling extended his neck, his vaccine is up
to date, what you will do?
Iv antibiotic against staphylococcus aureus, (Intubation with anesthesia)
21. Baby with wide pulse pressure , 1000 gram, 29 days after 5 days , come
with tachypnea, no need for oxygen saturation: what you will do Albuterol,
(Indomethacin)

22. Girl with 40 c temperature with purple lesion, vomiting, what is the cause
(Meningococcus), hemophilus influenza
23. CSF indicate 80 portion, predominant neutrophil, and negative gram stain,
what is the cause?
Viral, (partially treated bacterial), bacterial.
24. Baby sit with minimal support, does not have pincer grip, 6,9,12
25. Baby start creep on knee and hand, say bye bye? 10 weeks
26. 5 kg baby with 10% dehydration, with no sodium deficit, what is the
amount of fluid that you have to give in 24 hours?
750, 1000, 1500

27. Baby going to do surgery, what is the maintenance fluid? ( based


on100,50,20 rule)
What is false?
(Photosensitivity-riboflavin)
Biotin- alopecia
Nerve tenderness- thiamine
28. Baby with sore throat, rapid test indicate positive streptococcus, what you
will do?
Give penicillin V, do dnas antibody test
29. Baby with Kawasaki , after management what will you do?
ECG in each visit until adolescence, (Echo serial until the first year)
30. What of these symptoms of rheumatic fever does not relieve with aspirin
Fever, arthritis, (chorea)

Child with accidently positive dipstick +2 protein what you will do?
Repeat the dipstick, do 24 hour urine protein collection (not sure about the
answer)

Patient with absence seizure, 10 second with abnormal movement of his face,
the DOC?
Carbamezapin, (valproic acid), phenytoin

Febrile seizure the first time, what you will do


Assure the parents, admitting and medication

Child obese, obsessive with eat, he will have?


Cryptocridism with small penis (Prader-Willi)
What is the most common complication of mumps in a 17-year old male?
Meningitis, (Orchitis)

Baby hyperbillirobinemia sustained in the 8 month, he is well, blood film show


no abnormal cells, the last exam was bilirubin 12.5, direct = .4 >> what is in your
mind
Gilbert, spherocytosis, polycythemia, hypothyroidism

Baby love to sleep, constipation, ?

Baby well , with bleeding thrombocytopenia, after viral infection 1 week ago?
Give hime immuniglobuline, steroid, admit and iv antibiotic

Baby with barking cough?


Croup caused by hemophillis, croup caused by parainfluenza

What is wrong about mild persistent asthma?

All are normal finding in newborn except:


Monglian spot, vaginal bleeding lost , strawberry navios, langus

Child with tanner stage 2, his height in the fifth percentile since early childhooh,
what you will expect?

Child 4 years old, with hieghth at 2 year age and bone age at 18 monthes , what
shall he have ?
Hypothyroidism, congenital adrenal hyperplasia, diabetes, psychological
depriviation

Baby born with good weight and normal height, since that he fail to gain weight
and failure to thrive with diarrhea and recurrent respiratory infection, what you
will do? Chloride sweat test

CAH, you will have]hyponatremia, hyperkalemia, 17-ketoprogesterone secretion


decrease in utine,

Diabetes type 2:
The etiology is antibody against beta cells, recently int decrease in the children,
strongly inhertid even more thant diabetes type 1

What is the mean stage of puberty in the femal


9, 11, 14

Every thing in kwashiorkor except:


Edema, dermatological changes, skin changes, spastisity

What is true about infantile spasm :


The most common type is the mixed type, occurs between 6 weeks and ..

U wave and flat t : what is true


Hypokalemia, hyperkalemia, hypernatremia, hyponatremia

All are non-blanching rash except : rosella , petechial, Kawasaki

Baby with stridor that decrease when sleep and prone position: laryngeomalacia

Baby born with white reflex, hydrocephalus, he has congenital infection with :
rubella, CMV, syphilis

Baby at 18 month , what he can do:


Go up and down stairs with on step
Help you dress and undress
Make sentences with 2 to 3 words

You have baby with PSCGN , you find that c3 in decrease , you give him
treatment , to confirm your diagnosis , when you will measure the c3 again
In 4 weeks, 8 weeks, 1 week

In nutritional rickets what is true


Decrease phosphorus and calcium, increased parathyroid hormone and
decrease 25…
Scarlet fever what is wrong, incubation period more than 7 dayes

Baby cyanosed with no pulse in radial and femoral after 2 days , dominnt right
heart with decrease r in v5,6 what is the cause :
Hypoplastic left heart syndrome, transposition of the great vessels, total anomly
of pulmonary veins return

Multiple vesicles in the tonsils and soft palate, what is true


Herpangia, herpes simplex

Child can reide tricycle, =no his age and sex, socially , when he first do these
skills
3-4 years,5-6 years, 2-3 years

Baby with sever dyspnea, wheeze, his old brother has common cold last week,
no history of asthma or allergy , what you will do
Oxgen 30%. Subutamlol for 4 hours, admit

Baby with no productive cough, well child, x ray show multiple infiltration in
both side of chest, what you will give him:
Doxcillin, azinthromycin, amoxicillin

All are used for treatment of TOF spells except:


Morphine, epinephrine, oxygen , sodium bicarbonate

Patient have shortness of breath 2-3 times in the year, what is the best
treatment
sulmotarol when needed, inhaled b2 agonist, inhaled steroids

What is wrong:
triple Wight at 1 year
double weight at 5 months
guardable Wight at 36 month

Patient with rocky bottom, clenched hand, with minimal dismorphic features in
the face he will have, trisomy 13, trisomy 18

Baby 6 weeks severe cyanosed with tachypnea, with no pulse in the femoral
artery ,you will do…ct for the brain, give him prostaglandine analog

Baby with sore throat and exudate over the tosilles after 2 weeks he developed
hematuria, smoky urine , what is the most appropriate test that you will
do,phalangeal swab culture, anti dnase antibody
What is true:, balckfan diamond present after 1 yeaer, common in fancon
20epatosplenomegaly, in fanconi hb f increased

All are present in 4 mohtes rickets baby except: bow legs, craniotabes, rickets
rosary

Child with sever flankd pain and ternderenss, hypotensive hsever high fever,
what you will do .. admit him with IV antibiotic + blood culture, admiht him with
IV antiobiotic + clean catch culture , IV antibiotic + lumber puncture

All are present in down syndrome except: cleft palate, brachydactaly, upward
palperable fissure

What is the best time eruption for teeth eruptionL first molar, central insciors,
cannins, second molar …… first molar, laternal insciores, cannies, first premolar

Baby with no murmer at birth ,after 9 weeks come iweh holosystolic muremer
in the left

Cow milk more than breast feeding in : essential fatty acids, more protein, more
iron

Baby squat with boat shape heart in x ray: TOF

All are non- blanching rash except: roesolla


Medical Students Quiz
===================
Encircle the letter of the best single answer:
1. All are correct for measles EXCEPT :
The incubation period is 8-12 days.
Infants are protected for at least 4 months after birth.
Koplik spots are pathognomonic for the disease.
Immunoglobulins are protective if given upto 7 days after exposure.
Measles vaccine is indicated for exposed children.
2. All of the following are correct causative organisms for the corresponding
disease EXCEPT:
A. Measles – Robeola virus.
B. German measles – RNA virus.
C. Scarlet fever – Gp B strept. Toxin.
D. Roseola infantum – herpes simplex virus.
E. Erythema infectiosum – parvovirus.
3. The following is correct for Rubella:
A. Incubation period is 10 days.
B. Patient is infectious for 10 days after onset of rash.
C. Infants with congenital rubella secretes the virus in urine for a year.
D. Transplacental antibodies are not protective.
Immunoglobulins has no role in protection.

4. All are recognized complications for Scarlet fever EXCEPT :


A. Pancreatitis.
B. Otitis media
C. Pneumonia.
D. Meningitis.
5. All of the following are correct incubation periods EXCEPT:
A. Measles 10 days.
B. Rubella 2-3 weeks.
C. Mumps 2-3 weeks.
D. Scarlet fever 7 days.
E. Chicken pox 11-20 days.
Quiz in Pneumonia
1. All are common pathogen in infants < 3months with pneumonia EXCEPT:
A. gp B strept.
B. Chlamydia.
C. H.influenza.
D. Staph
E. Gm –ve organisms.
2. Common pathogens in pneumonia from 3 months to 5 years are:
A. H. influenza and staph.
B. Strept pneumonia & H. influenza.
C. H.influenza & gp A strept.
D. Strept pneumonia & gp A strept.
E. M.catarrhalis & staph.
3. Drug of choice for pneumonia in children from 3 months to 5 years is:
A. penicillin
B. cotrimoxazole.
C. Beta-lactam
D. Tetracycline.
E. Gentamycin.
4. The following is correct for drug therapy for pneumonia EXCEPT:
A. generally given from 7 to 10 days.
B. In staph empyema should complete 3-4 weeks treatment.
C. 3 weeks in pleural effusion.
D. 7 days for gram negative organism
E. 10 days for Chlamydia.
5. All are indications for hospitalization in a case of pneumonia EXCEPT:
A. moderate disease from onset.
B. Younger age.
C. Other underlying disease.
D. Socio-economic cause.
E. Failure to respond to oral treatment.
Immunization
1. A 5-year-old girl with asymptomatic human immunodeficiency virus (HIV)
infection is being seen for a preschool evaluation.
Of the following, the vaccine that is CONTRAINDICATED is
A. conjugated Haemophilus influenzae type b
B. diphtheria-tetanus toxoids with pertussis
C. measles-mumps-rubella
D. oral poliovirus
E. split virus influenza

2. Which of the following vaccines should NOT be given to children with severe
combined immunodeficiency syndrome:
A. DPT.
B. Measles vaccine
C. Salk polio vaccine.
D. Hep. B vaccine.
E. Pneumococcal vaccine.

3. In the Palestinian National Immunization Program, all of the following are


correct EXCEPT:
A. Salk vaccine is given at one and two months of age.
B. Hep. B vaccine is given at birth, one month and six months of age.
C. A booster dose of DPT is given at six years of age.
D. MMR vaccine is given at age of 15 months.
E. Rubella vaccine is given to girls at age of 12 years.

4. Inn BCG vaccine all are correct EXCEPT:


A. It is live attenuated vaccine.
B. It is given after birth.
C. It has nearly 80% protection against pulmonary T.B.
D. It has nearly 60% protection against systemic T.B.
E. It can be given to asymptomatic HIV infant.

5. All of the followings are correct for live attenuated vaccines EXCEPT:
A. Contraindicated in immunodeficient patients.
B. Given only after 3 months of stopping immunosuppressive drugs or large
steroid dose therapy.
C. Not given within 3 weeks before or 3 months after giving immunoglobulins.
D. Two Live attenuated vaccines are not given together.
E. Measles vaccine is not given to a patient with T.B except after full treatment.

Pediatric M.C.Q’s ReviewDR/NABIL M.D.AL BARQOUNI


1. Measles
A. In countries with no immunization programs, the peak age of infection is
infancy
B. In GAZA immunization programs have reduced the incidence of infection by
80%
C. Usually spread by direct contact with infectious droplets
D. Patients become contagious when the rash appears
E. Incubation period is 4-5 days from exposure to onset of symptoms
Cough, coryza (inflammation of nasal mucosal membranes), fever,
conjunctivitis, exanthem of red macules and papules and Koplik spots

2. Young children have OM, pneumonia, croup and diarrhea


Acute encephalitis (1:1000) -->Permanent brain injury
In US death 1-3/1000 due to respiratory of neurologic complications
Transmitted by direct contact with infectious droplets or LESS COMMONLY
airborne spread Since vaccine use in 1963 there has been a 99% reduction in
incidence in US Children are contagious 4-5 days BEFORE rash appears to 4 days
after appearance of rash . Incubation period is 8-12 days from exposure to
onset of symptoms

Recognised complications of measles include all except :


A. Encephalitis.
B. Fébrile convulsions.
C. Diarrhoea.
D. Infertility in males.
E. Corneal ulceration.
All the following are recognized complications of chickenpox EXCEPT :
(A) Reyes syndrome
(B) Encephalitis
(C) Pneumonia
(D) Hemorrhagic vesicles
(E) Orchitis
3. Toxic shock syndrome (TSS) is associated with all of the following findings
EXCEPT :

(A) Vomiting and diarrhea


(B) Disorientation
(C) Increased platelet count
(D) Elevated liver function tests
(El Elevated blood urea nitrogen

4. A previously healthy 2 year old is brought to ED because her mom is unable


to awaken her for 45 minutes. She has not been ill. PE reveals an afebrile,
hypotonic child who withdraws her hand from painful stimuli but does not
spontaneously open her eyes. RR is 36/min, BP is 92/64. What is the next
best thing to obtain?
A. BUN
B. CXR
C. EKG
D. EEG
E. Toxicology screen

Pneumonic for Causes of Altered Consciousness: “Tips from the Vowels”


T-I-P-S-A-E-I-O-U
Trauma/Tumor
Infection/Inborn Errors/Insulin/hypoglycemia
Poisons
Shock
Alcohol/Abuse
Epilepsy/Encephalopathy
Intussusception
Opiates
Uremia

5. The MOST common site of Pseudomonas colonization in ICU is:


A. Central venous catheters
B. Foley catheters
C. Peripheral intravenous catheters
D. Surgical wounds
E. Tracheostomy tubes
6. You have intubated an 8 month old with sepsis and apnea with a 4.0
endotracheal tube. Proper placement is confirmed by observing chest rise
and auscultating symmetric breath sounds after bag and mask ventilation.
Perfusion is <3 seconds, and heart rate is 120/minute. Five minutes later the
RT tells you the oxygen saturation is 83% and the blood pressure and pulse
are dropping. Breath sounds are absent on the right and the right chest is
hypertympanitic.
What is the most likely diagnosis?
A. Esophageal intubation
B. Incorrect ET size
C. Obstructed ET tube
D. Right main stem bronchus intubation
E. Right pneumothorax

7. A 6 year old boy with severe factor VIII deficiency hemophilia develops
increased swelling of the right distal forearm. There is not history of trauma.
The peripheral circulation is normal and there is no joint involvement. The
MOST important complication of bleeding in this location is:
A. Blood loss
B. Muscular Damage
C. Neurologic Impairment
D. Tendon Shortening
E. Vascular Damage

8. The MOST common cause of acute laryngotracheobronchitis in children that


also causes many cases of bronchiolitis and pneumonia among young infants
is:
A. Adenovirus
B. Influenza virus
C. Parainfluenza virus
D. Respiratory syncytial virus
E. Rhinovirus

9. Which of the following laboratory findings is unusual in patients with simple


( nutritional ) rickets ?
(A) Aminoaciduria
(B) Hyperphosphaturia
(C) Elevated levels of serum alkaline phosphatase
(D) Hypercalciuria
(E) Hypophosphatemia

10. An ambulance team brings a 6 year old girl to the ED. She had been
unrestrained in the back seat. The girl is unresponsive on arrival and is
bleeding profusely from a scalp wound. Her Glasgow Coma Scale is 3.
What is the BEST initial step in evaluation and management?
A. Control profuse scalp bleeding
B. Establish IV access
C. Order portable cervical spine radiographs
D. Remove all clothing
E. Secure an adequate airway

11. Which of the following sets of blood gas values is most compatible with
acute aspirine poisoning in a 16 month old child:
a-PH 7.60, pco2 40 mmHg, Hco3 40 mmol/L
b-PH 7.50, pco2 40 mmHg, Hco3 30 mmol/L
c-PH 7.25, pco2 20 mmHg, Hco3 8 mmol/L
d-PH 7.20, pco2 45 mmHg, Hco3 20 mmol/L
e-PH 7.00, pco2 35 mmHg, Hco3 8 mmol/L

12. Accepted indication for the removal of adenoids in childhood is:


A. Mouth breathing
B. Serious post nasal obstruction
C. Nasal escape of air in speech
D. Inspiratory snoring
E. Recurrent otitis media

13. A 2 week old presents with tachypnea, poor perfusion, gallop rhythm,
diminished pulses, and hepatomegaly. ABG shows metabolic acidosis.
Echocardiography reveals critical Aortic Stenosis.
What intervention is most likely to stabilize the infant’s condition?
A. Dobutamine
B. Epinephrine
C. Nitric Oxide
D. 100% oxygen
E. Prostaglandin E-1
14. A male infant born at 36 weeks gestation had a left testicle palpable in the
inguinal canal. At 12 months of age, the left testicle has failed to descend into
the scrotum.
What is most appropriate care for this infant?
A. Observation until 2 years of age
B. Orchiopexy
C. Radionuclide scan of left testicle
D. Treatment with human chorionic gonadotropin
E. Treatment with testosterone

15. Intussuception in childhood:


A. Has as the earliest sign the passage of red current jelly stools
B. Has a peak incidence in the first 3 months of life
C. Requires operative reduction in the majority of cases
D. May be initiated by ameckel's diverticulum
E. It has association with umbilical hernia

16. Necrotizing enterocolitis is associated with all of the following except:


A. Congenital heart disease
B. Prematurity with respiratory distress syndrome
C. Maternal diabetes mellitus
D. Breast feeding
E. Polycythaemia

17. Medical indication for circumcision in children include:


Hypospadias
A. 3 month infantile colic
B. Recurrent ballanitis
C. Ambiguous genitalla
D. Cryptorchidism

18. In congenital diaphragmatic hernia:


A. Hernias most commonly occur on the left
B. Persistent fetal circulation occurs uncommonly
C. Associated congenital anomalies are common
D. Most present between 12 and 24 hours of age
E. Pulmonary hypoplasia is rarely the cause of death

19. The average time of closure of the anterior fontanel is:


A. 12 months
B. 14 months
C. 16 months
D. 18 months
E. 22 months

20. A newborn female has an open neural tube defect, low set ears, VSD, and
rib and vertebral column malformations.
Which of the following MATERNAL conditions was most likely present during
pregnancy?
A. Alcoholism
B. Diabetes mellitus
C. Hypothyroidism
D. Iodine deficiency
E. Syphilis

21. The arthritis of acute rheumatic fever usually:


A. Is monoarticular
B. Heals without deformity
C. Appears after the fever subsides
D. Is seen only in patients with concurrent carditis
E. Involves large and small joints equally

22. Which of the following support the diagnosis innocent murmur:


A. Present only in diastole
B. Fixed splitting of second heart sound
C. Is loud in high out put states
D. Heard allover the praecordium
E. The venous hum is best heard at the apex

23. A 4 month old with vitamin D deficiency rickets would be expected to


show all of the following except:
A. Craniotabes
B. Bow legs
C. Rosary
D. Low serum phosphate level
E. High alkaline phosphate levels

24. The following are causes of jaundice EXCEPT:


A. Congenital hypothyroidism.
B. Wilson's disease.
C. Pityriasis rosea.
D. Alagille Syndrome.
E. Alpha-1-antitrypsin deficiency.

25. All of the following are true about kwashiorkor except:


A. The presence of edema
B. Rash in sun exposed areas
C. Hypoalbuminemia
D. Weak muscles
E. An increased susceptibility to infection

26. The following are single gene disorders Except:


A. Cystic fibrosis.
B. Galactosemia.
C. Duchenne muscular dystrophy.
D. Marfan syndrome.
E. Turner syndrome.

27. The risk of neonatal jaundice is increase by all except:

A. Prematurity.
B. Trisomy 21.
C. Elective caesarean section.
D. Congenital hypothyroidism.
E. Cephalohaematoma.

28. All of the following are recognized causes of hypertension in childhood


except:
Chronic glomerulonephritis
Renal vein thrombosis
Neuroblastoma
Peripheral pulmonary artery stenosis
Congenital adrenal hyperplasia

29. A 5 year old male is hospitalized in January with fever and seizures. LP
reveals clear CSF with 47 WBCs/mm3 all of which are lymphocytes. On PE he
appears obtunded but arouses with painful stimuli. Neurologic exam reveals
no focal findings. Which diagnostic test is most likely to reveal this child’s
illness?
A. Bacterial culture of CSF
B. PCR test of CSF for HSV
C. Strepto.pneumoniae bacterial antigen test of CSF
D. Viral culture of CSF
E. Viral culture of nasopharyngeal and rectal swabs

30. A 11 year old girl complains of dysuria and abdominal pain for 2 days. She
denies nausea, vomiting, flank pain and vaginal discharge, mild suprapubic
tenderness, and otherwise normal findings.
What is the most likely diagnosis?
A. Bacterial vaginosis
B. Candidal vulvovaginitis
C. Chlamydia urethritis
D. Pelvic inflammatory disease
E. Urinary tract infection

31. A 3 year old girl comes to the ER with temperature of 40.0o C and acute
onset diarrhea. Stool is guaiac positive with leukocytes. There is no history of
foreign travel and the child has not received antibiotics recently.
What is the most likely organism?
A. Clostridium difficile
B. Giardia lamblia
C. Rotavirus
D. Salmonella enteritidis
E. Vibrio cholerae

32. You are evaluating a 4 week old boy for tearing of the right eye that has
worsened over the past week. Physical exam reveals slight tearing but no
evidence of purulent exudate or conjunctival erythema. All other findings are
normal.
The MOST appropriate initial management is…
A. Administration of amoxicillin
B. Endoscopic dacrocystorhinostomy
C. Instillation of silver nitrate in the eyes
D. Observation with intermittent massage of the duct
E. Surgical dilation of the nasolacrimal duct
33. The following can be the cause of a non-blanching rash except:
A. Idiopathic thrombocytopenic purpura.
B. Roseola infantum.
C. Henoch-Schönlein purpura.
D. Haemolytic uraemic syndrome.
E. Meningococcal sepsis.

34. Regarding nocturnal enuresis all true except:


A. Is more common in girls.
B. Can be a presenting feature of D.M.
C. Can be a presenting feature of D.I.
D. Emotional disturbance can be a cause.
E. Desmopressin is used in the treatment.
35. In acute bronchiolitis:
A. Ribavirin is the treatment of choice for hospitalized cases
B. Lung volume is usually decreased
C. Bronchodilators are usually effective
D. Feeding difficulties are common
E. Upper air way obstruction is a common feature

36. The following congenital heart lesions are "Acyanotic" except:


A. Ventricular septal defect.
B. Atrial septal defect.
C. Patent ductus arteriosus.
D. Aortic stenosis
E. Transposition of the great arteries

37. Hemolytic uremic syndrome is characterized by all of the following except:


A. Microangiopathic hemolytic anaemia
B. Familial cases
C. Thrombocytopenia
D. E-coli (0157:H7) infection
E. All of the above

38. A previously health 1-year-old infant who weighs 10 kg presents to your


office with a fever of 39° C (102.2° F). Her mother is very concerned about
the child’s intake and asks for guidance regarding caloric requirements during
this illness.
Of the following, the best estimate of the child’s caloric requirements at this
time is
A. 500 kcal/d plus 500 kcal due to the fever
B. 1,000 kcal/d plus 250 kcal due to the fever
C. 1,500 kcal/d
D. 1,500 kcal/d plus 250 kcal due to the fever
E. 2,000 kcal/d

39. Skin peeling occurs in all of the following conditions except:


A. Burns.
B. Kawasaki disease.
C. Scarlet fever.
D. Toxic shock syndrome.
E. Erythema infectiosum (slapped cheek disease).

40. In neonatal RDS (respiratory distress syndrome) all are true except:
A. Surfactant is useful in the treatment.
B. It is common in infants below 28 weeks gestation.
C. Antenatal steroids are beneficial.
D. Maternal opiate abuse increases the risk.
E. Maternal diabetes increases the risk.

41. In children with cerebral palsy all are true except:


A. Associated with a degree of learning impairment.
B. Birth asphyxia is the most common cause.
C. Hand preference before 12 months can be a useful sign.
D. The spastic form is the most common clinical type.
E. Management should adopt a multidisciplinary approach.

42. Which of the following condition is one type of haemoglobinopathies:


A. G6PD deficiency.
B. von Willebrand disease.
C. Spherocytosis.
D. Pyruvate kinas deficiency .
E. Sickle cell disease.

43. All of theses neuromuscular disorders are "genetic diseases" except:


A. Duchenne muscular dystrophy.
B. Myotonic dystrophy.
C. Spinal muscular atrophy.
D. Guillain Barre syndrome.
E. Frierich's Ataxia

44. A 5-year-old boy is brought to the emergency department after having


been struck by an automobile. Physical examination reveals facial abrasions,
abdominal tenderness, and gross blood at the urethral meatus. Pelvic
radiography reveals a left-sided fracture of the superior pubic rami.
Of the following, the best procedure for INITIAL evaluation of the urinary tract
in this patient is
A. Bladder catheterization via the urethra
B. Computed tomography of the abdomen
C. Intravenous pyelography
D. Renal untrasonography
E. Retrograde urethrography

45. In pyloric stenosis all are true except:


A. Is more common in boys.
B. Surgery is the usual treatment.
C. Infants often have a metabolic alkalosis.
D. Constipation can be a presenting feature.
E. A barium meal is usually performed to confirm the diagnosis.

46. The following can cause failure to thrive except:


A. Celiac disease.
B. Cystic fibrosis.
C. Emotional neglect.
D. Gastro-oesophageal reflux.
E. Otitis media.

47. The following are causes of polyhydramnios except:


A. Maternal diabetes mellitus.
B. Potter syndrome.
C. Anencephaly.
D. Oesophageal atresia.
E. Spina bifida.
48. The following are inherited in an X-linked manner all are true except:
A. Duchenne muscular dystrophy.
B. Haemophilia A.
C. Cystic fibrosis.
D. G6PD deficiency.
E. Hunter syndrome.

49. Concerning chickenpox all are false except:


A. The incubation period is 7 days.
B. Aspirin is a useful anti-pyretic.
C. Transmission is usually by contact or airborne.
D. Encephalitis not a recognized complication.
E. There is no effective vaccine.

50. Regarding Kawasaki's disease:


A. Purulent Conjunctivitis is a recognized
feature.
B. It is caused by a spirochaete organism.
C. It is a cause of coronary artery aneurysms.
D. Fever is not a typical feature.
E. Is associated with profound
thrombocytopenia

 A 1-year-old boy has been treated with a low-phenylalanine diet for the past
year after having been identified in infancy as having phenylketonuria.
Despite appropriate dietary restriction of phenylalanine, he has developed
neurologic symptoms.
Of the following, this child is MOST likely to be deficient in
Biotin
Cobalamin
Carnitine
Tetrahydrobiopterin
Thiamine

51. A 22-month-old girls is nonverbal. She sat alone at 7 months and walked
by 13 months, but now exhibits a wide-based stance, no longer ambulates,
and will not pick up or manipulate toys. Findings include: height and weight
at the 50th percentile; head circumference below the 5th percentile, with no
increase over the past 8 months; normal fundi, and no organomegaly.
Of the following, the MOST likely diagnosis is
Adrenoleukodystrophy
Cerebral palsy
GM2 gangliosidosis (Tay-Sachs disease)
Hypothroidism
Rett syndrome

52. Of the following, the MOST likely etiology of this decrease in hemoglobin is
Aplastic crisis
Folic acid deficiency
Hyperhemolytic crisis
Iron-deficiency anemia
Splenic sequestration

53. An 18-day-old term infant develops fever, lethargy, and focal seizures.
Findings include: an ill-appearing infant without exanthem, hepatomegaly, or
jaundice. Analysis of cerebrospinal fluid reveals white blood cells, 115/mm3;
45% neutrophils; 55% lymphocytes; red blood cells, 40/mm3; glucose, 45
mg/dL; protien 200 mg/dL; and negative Gram stain.
In addition to ampicillin and cefotaxime, the MOST appropriate treatment to
begin at this time is
Acyclovir
Amphotericin B
Dexamethasone
Metronidazole
vancomycin

54. A 10-year-old girl has complained of intermittent left lower abdominal


pain for 2 days. Previous evaluations, including a thorough physical
examination, urinalysis, and complete blood count, have not revealed the
cause. Tonight she is complaining of pain in the lower left abdomen that
radiates into her left leg. There is no history of fever, vomiting, or diarrhea.
The MOST likely cause of this girl’s pain is
Appendicitis
Intussusception
Malrotation of the intestine
Nephrolithiasis
Ovarian torsion
55. A 13-year-old boy has a congested, itchy, and runny nose accompanied by
itchy eyes. These symptoms usually occur in the fall, are unresponsive to
over-the-counter decongestants, and his father has the same problems.
Physical examination reveals pale, boggy nasal turbinates; clear nasal
discharge; and dark circles under his eyes.
These findings are MOST consistent with
Allergic rhinitis
Infectious rhinitis
Nonallergic rhinitis with eosinophilia
Rhinitis medicamentosa
Vasomotor rhinitis

56. A 5-month-old child is brought to the emergency department by her


mother because she has been “crying a lot” for the past 24 hours. The
mother denies any history of fever, trauma, or illness. Physical examination
reveals a lethargic toddler who is very irritable when examined and who has
mild tachycardia, scattered bruises over the chest, and ecchymosis behind
the left ear.
The MOST appropriate management is to
A. Administer intravenous naloxone
B. Administer a 20 mL/kg bolus of intravenous normal saline
C. Obtain an abdominal radiograph
D. Obtain a complete blood count and blood culture and administer
intravenous ceftriaxone
E. Obtain a computed tomographic scan of the head and a skeletal survey.

57. A 14-year-old boy who has allergic rhinitis reports that he frequently
develops coughing and wheezing after about minutes of playing soccer.
These symptoms improve after resting for 30 minutes. Of the following, the
drug that will give the BEST response in this patient if administered just prior
to exercise is
A. Inhaled beta2-agonist
B. Inhaled corticosteroid
C. Oral beta2-agonist
D. Oral corticosteroid
E. Oral theophylline

58. A term infant is delivered vaginally to a healthy 24-year-old primigravida.


Immediately after birth, the infant’s respiratory effort is vigorous, but
subcostal retractions and cyanosis persist. The abdomen is scaphoid in
appearance. Bag and mask ventilation is initiated. Auscultation reveals
decreased breath sounds on the left and heart tones that are louder on the
right.
The MOST likely explanation for these findings is
A. Congenital cystic adenomatoid malformation of the lung
B. Dextrocardia with situs solitus
C. Diaphragmatic hernia
D. Esophageal atresia with tracheoesophageal fistula
E. Pneumothorax

1. A 5-year-old girl with asymptomatic human immunodeficiency virus (HIV)


infection is being seen for a preschool evaluation.
Of the following, the vaccine that is CONTRAINDICATED is
A. conjugated Haemophilus influenzae type b
B. diphtheria-tetanus toxoids with pertussis
C. measles-mumps-rubella
D. oral poliovirus
E. split virus influenza

2. The average growth of head circumference in (centimeters/ month) in the


age from
8 – 9 months is as follows:
A. 2.00 cm /month.
B. 1.50 cm /month.
C. 1.00 cm /month.
D. 0.50 cm /month.
E. 0.25 cm /month.

3. A developmentally normal child who is just able to walk with one hand held
and says mama and dada with meaning is probably what age?
A. 8 months
B. 10 months
C. 12 months
D. 14 months
E. 16 months

4.All are correct for measles EXCEPT :


The incubation period is 8-12 days.
Infants are protected for at least 4 months after birth.
Koplik spots are pathognomonic for the disease.
Immunoglobulins are protective if given within 3 days of exposure.
Measles vaccine is NOT indicated for exposed children.

5. A 4-year-old boy develops fever, swelling of the parotid gland, and headache.
Of the following, which complication is MOST likely to occur in this patient?
A. Conjunctivitis
B. Deafness
C. Meningitis
D. Myocarditis
E. Orchitis
6. A full-term infant becomes cyanotic in the delivery room. After intubation
and attempts at stabilization, it is noticed that the infant has a scaffold
abdomen and decreased breath sounds over the left hemithorax. The most
likely diagnosis is
A. pneumothorax
B. cardiomegaly
C. diaphragmatic hernia
D. neuroblastoma
E. atelectasis

7. A newborn female has a tracheoesophageal fistula. All of the following would


be appropriate EXCEPT
A. renal ultrasonography
B. cardiac ultrasonography
C. brainstem auditory evoked response testing
D. evaluation for anal atresia
E. vertebral radiographs

8. The metabolic alteration found in infants with pyloric stenosis is


A. hypochloremic acidosis
B. hypochloremic alkalosis
C. hyperchloremic acidosis
D. hyperchloremic alkalosis
E. None of the above.
9. Of the following, the MOST common feature of chronic, nonspecific diarrhea
of infancy and childhood (toddler's diarrhea) is:
A. guaiac-positive stools
B. intermittent fever
C. lactose intolerance
D. significant weight loss
E. unimpaired growth

10. A 6-year-old boy with absence seizure . Of the following pairs of drugs
MOST likely to be effective in the treatment of his condition:
A. ACTH or corticosteroid.
B. Amantadine or bromide.
C. Carbamazepine or gabapentin.
D. Ethosuximide or valproic acid.
E. Phenobarbital or phenytoin.

11. High risk lesions of bacterial endocarditis include all of the following EXCEPT:
A. PDA.
B. VSD.
C. ASD.
D. Blalock Taussig shunt.
E. Left-sided valvular lesions.

12. In infant with UTI , the most common manifestation is:


A. Fever.
B. Dysuria.
C. Frequency.
D. Costovertebral angle tenderness.
E. Incontinence.

13. Which of the following vitamins is in higher concentration in cow's milk than
in human milk?
A. A
B. C
C. E
D. K
E. B6

14. Risk factors for fatal asthma include all of the following EXCEPT:
A. sudden severe obstruction
B. an allergic component
C. underuse of steroids
D. underestimation of the severity
E. poor compliance

15. Long-term complications of type I diabetes include all of the following


EXCEPT:
A. hypoglycemia
B. hypocalcemia
C. neuropathy
D. nephropathy
E. arthropathy

19. In chronic granulomatous disease, the neutrophils are:


A. It is of autosomal dominant inheritance.
B. Unable to increase their numbers in response to bacterial infections.
C. Leukemoid in number.
D. Markedly decreased in number.
E. Incapable of intracellular killing of certain bacteria.

20. A 4-month-old boy weighed 3500 g at birth. He now weighs 4.5 kg. He has
been formula fed since 1 week of age.
The MOST likely reason for this child's failure to thrive is:
A. absence of solids in his diet
B. exaggerated parental concern about overfeeding
C. improper feeding technique
D. omission of supplemental vitamins
E. withholding of nighttime feedings

22. An infant with eczema, thrombocytopenia, and recurrent otitis media is


most likely to have which one of the following disorders?
A. DiGeorge syndrome
B. Bruton disease
C. Graft-versus-host disease
D. Wiskott-Aldrich syndrome
E. Nezelof syndrome
1. The median age of an infant who can first sits with support, turns
over, bears weight on legs when held on standing position, and
reaches for an object but can not stand on furniture is:
A. 4-month-old
B.5-month-old
C.6-month-old
D.7-month-old
E.8-month-old

2. The mother brought her 10-month-old son complaining that he is not thriving
well for the last 4 months although he did not have any disease, you find his
weight is just below -2 Z-score, his length is below -1 Z-score, and his head
circumference is below 0-Z-score, your correct comment will be as follows:
A. he has microcephaly
B. he has short stature
C. his feeding and weaning is proper
D. he has chronic problem since birth
E. he has recent improper feeding and weaning.

3. A 14_ month old boy is brought to your office because a visiting relative
noted that he appeared pale, he drinks 40-50 OZ of cow milk daily (1000-
1200 ml). He looks well except for pallor. WBC of (6.9× 109/L) ,hemoglobin of
5.9 g\dL ,MCV of 57 fL and platelet count of 775× 109/L .The most
appropriate next step is to:

A.Give a blood transfusion


B.Give a single dose of intravenous iron
C.Initiate a trial of oral ferrous sulfate
D.Obtain hemoglobin electrophoresis
E.Obtain serum ferritin, iron , and total iron binding capacity measurements

4. All are findings in Rickets EXCEPT:


A. Bow legs.
B. Cubitus vulgaris.
C. Wide lower end of radius.
D. Craniotabes.
E. Rachitic rosary.

5. Pellagra is associated with the following vitamin deficiency:


Vitamin A
Vitamin B1
Vitamin B2
Vitamin B6
Niacin

6. Regarding Down syndrome ;


A. the incidence is 1 in 1500.
B. upward sloping of palpebral fissures.
C. Brushfield spots are found in the sclera.
D. microglossia causes speech delay,
E. a single palmar crease is pathognomonic of the syndrome.

7. Regarding raised intracranial pressure the following is true except:


A. Drowsiness is seen with a rapidly increasing pressure
B. Convulsions are a common presentation
C. The headache is worse on movement
D. There may be a sixth nerve palsy
E. Signs take longer to develop in young children than in adults

Thrombocytopenia with abnormally shaped large platelets is:


A. Wiskott – Aldrich syndrome
B. TAR syndrome (thrombocytopenia with absent radii )
C. Bernard – soulier syndrome
D. Immune deficiency
E. Idiopathic thrombocytopenic purpura

Pallor, hemosideosis, jaundice and extramedullary hematopoiesis are


characteristic of which one of the following disorders :
A. Sickle cell trait
B. Sickle cell anaemia
C. Thalassemia minor
D. Thalassemia major
E. Hemoglobin C – thalassemia

A 2-year-old boy is hospitalized with a history of fever for seven days,


strawberry tongue, bilateral conjunctival injection without exudate, and a
generalized morbilliform rash. An echocardiogram shows dilatation of the left
main coronary artery. Administration of which of the following is the most
appropriate next step in management?
A. Acetaminophen
B. Coumadin
C. IVIG
D. Low-dose aspirin
E. Pulse steroids

A 3-yr-old boy presents to an urgent care clinic with a-3 day H/O abdominal pain
and difficulty walking. Abnormal findings include BP 120/80 mmHg, diffuse
abdominal tenderness, purpuric rash of the hands, and diffuse periarticular
tenderness and swelling of the ankles.
The most likely diagnosis is :
A. Systemic lupus erythematosus
B. Kawasaki's disease
C. Juvenile rheumatoid arthritis
D. Henoch-Sch nlein purpura
E. Stevens-Johnson syndrome

The smear shows a microcytic, hypochromic anemia. Iron supplementation


therapy is started. When will the reticulocyte response be maximum?
A. 1 to 2 days
B. 5 to 7 days
C. 14 to 21 days
D. 3 to 4 weeks
E. Above 6 weeks

Characteristic feature of UMNL include all Except :


A. Increase in the tone
B. Wasting of the affected muscle
C. Loss of the abdominal reflexes
D. Clonus
E. An extensor planter reflex

Wheezing may be found in children affected by:


A. Inhaled foreign body
B. Aspiration of gastric contents
C. Viral bronchiolitis
D. Bronchopulmonary dysplasia
E. All of the above

Which of the following is false Tetravalent meningococcal vaccines composed of


capsular polysaccharides of meningococcal groups:
A- A
B- B
C- C
D- Y
E- W135

What is the treatment of choice for epiglottitis?


A- Supportive only
B- IV epinephrine
C- IV prednisone
D- IV amoxicillin
E- Endotracheal intubation

A 4-year-old boy develops fever, swelling of the parotid gland, and headache. Of
the following, which complication is MOST likely to occur in this patient?
A. Conjunctivitis
B. Deafness
C. Meningitis
D. Myocarditis
E. Osteomyelitis

In minimal lesion nephrotic syndrome all are true except:


A. There is proteinuria without gross hematuria
B. Hypertension is unusual
C. Compliment (C3) level is depressed
D. Serum BUN and creatinine are normal
E. Bacterial peritonitis may occure

Transient proteinuria is usually found in all of the following except :


A. Vigorous excersise
B. Acute glomerulone phritis
C. Fevers exceeding 38.5Co
D. Orthostatic
E. Dehydration
Which of the following procedures is reliable in the diagnosis or
exclusion of a foreign body in the airways?
A- A physical examination
B- A thorough history taking
C- Bronchoscopy
D- Chest transillumination (Holzknecht's sign)
E- Chest x-ray

A 2-year-old child is noted to have an erythematous, bulging right tympanic


membrane. The two most likely bacterial causes of this illness are:
A. Streptococcus pyogenes and Staphylococcus aureus
B. Haemophilus influenzae and Staphylococcus aureus
C. Haemophilus influenzae and Streptococcus pneumonia
D. Streptococcus pneumonia and Staphylococcus aureus
E. Moraxella catarrhalis and Streptococcus Pyogenes

A 10-year-old child complains of episodic abdominal discomfort; the child's


mother says that these episodes are associated with periods of staring and
followed by a brief period of lethargy. Which of the following disorders is most
likely to be responsible for the child's symptoms?
A. Complex partial seizures.
B. Migraine.
C. Absence.
D. Conversion reaction.
E. None of the above

In Breath holding attacks all of the following true except:


A. Are commoner in infants and younger children
B. Can be confused with a generalised convulsion.
C. May be precipitated by a minor injury.
D. Should be treated with sedatives.
E. Are never fatal.

An ambulance team brings a 6 year old girl to the ED. She had been
unrestrained in the back seat. The girl is unresponsive on arrival and is bleeding
profusely from a scalp wound. Her Glasgow Coma Scale is 3.
What is the BEST initial step in evaluation and management?
A. Control profuse scalp bleeding
B. Establish IV access
C. Order portable cervical spine radiographs
D. Remove all clothing
E. Secure an adequate airway

Concerning urinary tract infection in children all of the following is correct


except:
A. Has aprevalence of 2 – 3 % in girls
B. UTI in children 2 – 6 years of age is often associated with bacteraemia
C. Is linked to constipation
D. Blood in urine may be present
E. Common causative organisms are gram negative organisms

The following are relevant in megaloblastic anaemia except:


A. History of intestinal resection
B. Goat’s milk feeding
C. History of celiac disease
D. History of Giardia lambia infection
E. History of epilepsy

Idiopathic thrombocytopenic purpura in children


A. Diagnosis usually required bone marrow examination
B. Often follows aviral infection
C. Is characteristically associated with moderate splenomegaly
D. Is associated with areduction of megakaryocytes on bone marrow
examination
E. Requires splenectomy in more than 20% of cases

The cranial nerves . all of the followings are true except:


A. The trigeminal nerve injury will lead to loss of corneal reflexes
B. Facial nerve injury can cause Bell’s palsy
C. The oculomotor nerve injury causes pupil constriction
D. Vagus nerve injury is associated with palate weakness
E. Hypoglossal nerve injury is associated with tongue atrophy

Absence epilepsy is characterized by :


A. Duration of absence of 1 -2 minutes
B. A postictal phase
C. Hyperventilation usually precipitating typical absence
D. Ictal EEG showing 3HZ slow wave generalized activity without spikes
E. Closed eyes during attack with loss of tone

In simple febrile convulsion


A. Lasting between 15 – 20 minutes
B. Occurs usually in less than 6 months old infants
C. Can be repeated in 24 hour period
D. The prognosis of children with febrile seizures is excellent
E. The risk of epilepsy is more than 2%

A 9-yr-old with vomiting and diarrhea has a systolic blood pressure of 75 mm


Hg. You should:
A. Check the fundi for papilledema
B. Administer 20 mL/kg of normal saline
C. Administer 20 mL/kg of lactated Ringer solution in 5% dextrose in water
D. Obtain upper and lower limb blood pressure readings
E. Begin administration of epinephrine or atropine

A 12-yr-old-girl is admitted with a 5-day history of worsening fever, left flank


pain, and vomiting. Physical examination reveals an ill-appearing, dehydration.
Pulse rate is 110/min; temperature is 40°C. Left costovertebral angle tenderness
is present. Serum sodium is 131 mmol/L, potassium 6.7 mmol/L, chloride 108
mmol/L, and bicarbonate 15 mmol/L. The most likely diagnosis is:
A. Addison's disease
B. Ingestion of high potassium-containing foods
C. Distal renal tubular acidosis
D. Acute pyelonephritis
E. Congenital adrenal hyperplasia

Acute rheumatic fever is associated with all of the following except:


A. Acute rheumatic fever is a systemic disease of childhood.
B. Often follows group A beta hemolytic streptococcal infection( GABHS)
C. Patients with acute rheumatic fever almost always have serologic evidence of
a recent GAS infection.
D. It is delayed suppurative sequelae to URTI with GABH streptococci.
E. It is a diffuse inflammatory disease of connective tissue, primarily involving
heart, blood vessels, and joints, subcutaneous tissue and CNS.

In meningococcal septicaemia:
A. Clinical features of meningitis are usually present .
B. Amikacin is a suitable choice for prophylaxis .
C. Prophylaxis does not reduce nasal carriage .
D. Treatment should await bacteriological confirmation .
E. A petechial rash is associated with DIC .

Treatment of acute gastroenteritis including all except:


A. Probiotic to be helpful only in mild cases
B. Avoiding and treating dehydration are the main goals in treatment of
viralenteritis.
C. A secondary goal is maintenance of the nutritional status of the patient.
D. Immunoglobulin can be given for patients.
E. There is routine role for antiviral, antiemetic or antidiarrheal drugs,
Antibiotics.

The most common cause of acute renal failure in children is :


A. Hemolytic-uremic syndrome.
B. Poststreptococcal glomerulonephritis
C. Prerenal acute renal failure.
D. Severe dehydration.
E. Nephrolithasis and urolithaiasis

Which of the following is not one of the criteria for diagnosis of Kawasaki
Disease?
A. fever >5 days
B. bilateral conjunctivitis
C. polymorphous rash
D. hepatosplenomegaly
E. cervical lymphadenopathy

All of the following are causes of acute gastroenteritis except:


A. Rotaviruses.
B. Caliciviruses (including the noroviruses).
C. Human metapneumovirus.
D. Astroviruses
E. Enteric adenoviruses (serotypes 40 and 41).

An atrial septal defect in a 7-year-old boy may be associated with which one of
the following?
A. A collapsing pulse.
B. Wide and fixed splitting of the second heart sound.
C. Clubbing of the fingers.
D. A pansystolic murmur of grade 4/6 in intensity.
E. Splenomegaly.

A newborn baby, one of twins born at term, is noted to be centrally cyanosed


soon after birth. Which of the following is the most likely cause:
A. Ventricular septal defect
B. Transposition of great arteries
C. Patent ductus arteriosus
D. Coarction of the aorta
E. Tricuspid atresia

All of the following findings suggest that a murmur is innocent EXCEPT :


A. It is grade 1-2/6.
B. It has a vibratory quality.
C. It is loudest at the apex.
D. It is only heard when the patient is febrile.
E. It changes in intensity with posture.

Which one of the following statements applies to congenital adrenal


hyperplasia:
A. It always presents shortly after birth
B. Boys more commonly present at an earlier age than girls
C. It is most frequently caused by 11 beta hydroxylase deficiency
D. It is commonly associated with genital pigmentation
E. Hypotension is invariably present

All of the following are true regard Shigella dysenteriae except:


A. It may cause disease by producing Shiga toxin, either alone or combined
with tissue invasion.
B. The incubation period is 1 to 7 days.
C. Infection is spread by person-to-person contact or by the ingestion of
contaminated food.
D. High fever and seizures may occur, in addition to diarrhea.
E. The small intestines are selectively affected.

Most cases of hemolytic uremic syndrome (HUS) are caused by:


A. Campylobacter jejuni .
B. Yersinia enterocolitica.
C. Entero-hemorrhagic E. coli (EHEC) especially the E. coli O157:H7 strain.
D. Clostridium difficile.
E. Entamoeba histolytica .

Overly rapid correction of hyponatremic dehydration should be avoided


because of:
A. The risk of developing brain edema.
B. The risk of renal failure.
C. The remote risk of central pontine myelinolysis.
D. The risk of developing cardiac arrhythmia.
E. Impaired response to catecholamine.

The following are causes of true hyponatremia except:


A. Secretion of inappropriate antdiuretic hormone( SIADH).
B. Gastrointestinal losses ( emesis & diarrhea).
C. Third space loss.
D. Nephrogenic diabetes insipidus.
E. Thiazide or loop diuretics.

All of the followings are causes of hypernatremia except:


A. Excessive sodium bicarbonate.
B. Central diabetes insipidus.
C. Ineffective breast feeding.
D. Phototherapy.
E. Cystic fibrosis.

All of the followings are causes of hypokalemia except:


A. β - Adrenergic agonists.
B. Proximal renal tubular acidosis.
C. 21-hydroxylase deficiency
D. Bartter syndrome.
E. Pyloric stenosis.

Viral bronchiolitis characterized by all of the following except:


A. Decreased in functional residual capacity.
B. Air trapping in chest X-ray.
C. Hypoxia.
D. Inflammation and partial occlusion of the bronchioles.
E. Wheezing and tachypnea.

All of the following statements are true of acute laryngeotracheobronchitis(


croup) except:

A. Barking cough and hoarse voice are early symptoms.


B. Boys are more likely to be admitted to the hospitals than girls.
C. The symptoms are frequently respond to salbutamol.
D. The adrenaline nebulization can be used.
E. Parainfluenza virus is the predominant etiological agent.

Pediatric Final exam 2/5/2013


Normal child wieght
At birth 3.5kg
Dublication at 5 month
Triplication at 12 month
Tetraplication at 36 month
10 % loss of body weight at first week

Duplication of height at age


12 month
2 year
3 year
4 year
5 year
Normal H.C at 1 year age 47 cm

Child at 5 year age can do except


Skipping
Steps 10 cubes
Count 10
Dress and undress
Draw triangle

Preschool children have 2000 word

15 Mo bow leg and other normal …. Reassurance


Rechets except
Bow leg
Craniotabes
Rosary
Cupitusvalugus
Swelling proximal radius

 All these association true except…… pellagra.. thiamine


 Cause of pellagra niacin
 Breast milk rich in carbohydrate
 Dealiyrecoiment of vitamin except vitamin A 400 IU(1500-5000IU)
 In Palestinian vaccination schedule all give at 4 mo except IVP
 Case… Mumps then fever headache…. Meningitis
 Case HIV pneumocystis juveri not give Varicella vaccine
 Child one word, bye … 10 mo
 All true about live attenuated vaccination except …. Not give together
 All true about kawshikwr except ….. Calori loss
 About down which true ….. upward slanting of eyelid
 All true about incubation period except…. Bacterial meningitis 10-14 day (2-
10d)
 Case history of OM, CSF low glucose, high protein, polymorphonucler cell
with seizure…. . partial treated meningitis
 All of this cause bronchiolitis except…… H. influenza
 All use in treatment of CROUP except… b agonist
 Mild persistent asthma true except …. Initial treatment salbutamol
 All of these true except….. Occulomotor nerve injure… meiosis
 CSV leakage risk for……S.pnumonia meningitis
 All true about ORS except….. K 100
 All true about ORS exceptGlucose + K
 All cause of hypernatremia except CF
 Euvelomichyponatemia except….. CHF
 Hyponatremia except…… nephrotic DI
 All true about hypocalcaemia except….. decrease reflex
 All viral causes gastroenteritis except….meta pnumato virus
 About shigla except… Small intestine
 Case about hemorrhagic E.coli
 TTT mycoplasma pneumonia Erythromycin
 About Typhoid bacillus except…. Human and animal
 CMV… periventricular calcification
 Rubella… PDA
 Toxo….. Hydrocephalus
 Varicella ….. limb hypoplasia
 Pavo….. anemia
 Case well baby … hemolytic anemia except… DIC
 Asthma….. no clubbing
 ITP…. Often follows viral infection
 Fanconi except….. microcytosis
 About G6PD….. x-linked r
 Uncomplicated PDA…. Plethoric lung
 Infective endocarditis causes
 VSD… at apex diastolic murmur
 Von will brand except decreas platelet
 Congenital rubella cataract
 Case Z score Ht -1, Wt -2 … Improper feeding
 About innocent murmur …. Except at apex loud
 Encephalitis measles 1-1000
 Enanthema pathognomonic measle
 Infantile spasm… most common mixed type
 Absence seizure … Hyperventilation
 TOF systolic murmur Pulmonary
 MCGN except… decrease C3
 PSGN …. Hypertensions encephalopathy
 Orthostatic protein urea except..ASGN
 B12 anemia except… Gardia
 Complication of rabid correction of hypornatremia
 DM type 1… acute onset
 Except… DM 1 follows islet antibodies
 Ketotic hypoglycemia …. Alanine deficiency
 DM diet….. High fiber
 Non ketosis hypoglycemia…. Precede by hypoglycemic convulsions
 RF…Erythema marginatum
 Kawasaki….. not spread from one to one
 Most common of congenital anomalies in aorta… …
 Facial nerve injury… LMN
 Necrotizing enterocolitis…. Pnumatic wall
 All true except Factor VII…. PT
 All About immunity except…. Thymus humeral….immunity
 About Thalasemia A2
 HF except Alkaline resistant
 Sickle cell anemia except … renal stone
 TOF spells except prostaglandin
 Hyperkalemia treatment except…. Arginine
 Hypokalemia except ….. Tall t wave
 HCP < 1 % serious complications
 Not complication of HCP…. Aneurysm
 Not common cause of emesis …. Crohns
 New porn twin second cyanotic most cause TGA
 Coarctation of aorta decrease vascular resistant in lower limb
 Hemolytic anemia except …. Increase haptoglobulin
 Teething except…Upper central insecior first
 Femal..Thelarch ..Rabid growth..Menarch
 Pott syndrome …. Renal agenesis
 Female with gestational diabetes the next pregnancy …..control of glucose
 Cow milk cause microcytic anemia ……. Therapeutic trail of ferrous sulfate
 Most common cause of abdominal distention is … Hydronephrosis/ dysplastic
kidney
 Febrile convulsion ….. Excellent prognosis
 About UTI except..2-6 year often have bacteremia
 Criteria of Kawasaki except … Hebatosplenomegaly
 TEF except …. Commonest distal and proximal fistula
 All about viral chest infection are true except….. decrees FRC
Quiz No 3... Dr.Anwar
Infectious Diseases
1- The incidence of post measles encephalitis is
1\100
5\100
5\1000
1\1000
1\100,000 I’m not sure
***
2- Which statement is incorrect about neonatal chicken pox
*VariZIG is indicated for newborn whose mother has chicken pox at delivery
*variZIG is given to all premature babies <28 weeks whose mothers have
chicken pox at delivery
*still half of the patients given VariZIG in the previous situations will have severe
disease
*acyclovir is also given when rash appears after delivary
*I.V acyclovir is given for all cases of community acquired varicella
***
3- HSV and VZV are both human herpes viruses. Both can cause congenital
infections especially when a pregnant woman has primary infection during the
1st trimester. The stigmata can be similar enough that the diagnosis is
occasionally confusing. Which of the following stigmata strongly suggests
congenital varicella only?
*Encephalitis
*Chorioretinitis
*Microcephaly
*Limb hypoplasia
*Vesicular skin lesions
***
4- A mother calls to report that her child was exposed to chicken pox in
preschool. The girl is healthy and has no history of varicella. The parents have
refused varicella vaccine. Which of the following will be your best advice is that
the child will not attend preschool
*For the period of 10-28 d after exposure
*For the period of 10-21 d after exposure
*For 14 d after exposure
*if she develops chicken pox lesions
*untill the chicken pox is no longer circulating in the classroom
***
5- You are ealuating an ill appearing 9 mo old hild whose father is a farmer.
Findings on PE inlude a brassy ough, onjuntiitis, and oryza. The hild's temp is 39
and there is red maulopapular rash on his head, trunk and proximal
extremeties. What is the M of mortality in this ondition
*encephalitis
*hemorrhagic shock
*hepatitis
*myocarditis
*pneumonia
***
match the correct HSV
A- HSV-1
B- HSV-2
C- BOTH OF THEM
D- NON OF THEM
*Causes more concurrent genital infection B
*Can cause initial infection at any anatomical site A
*Prevented by routine vaccine at developed countries D

" ‫" أسئلة امتحان األطفال‬


‫كم بزيد وزن الطفل " يف اليوم " يف أول شهرين ؟‬
‫ف سؤال توصيل كان من ضمن أسئلة دكتور أنور عن ال‬Infectious
‫ي‬ ‫كان يف‬mumps – rubella-
measeles
All are symptoms and signs of IgA nephropathy except
All are causes of trisomy 21 except ‫ وكان من ضمن الخيارات‬translocation – deletion
‫خصائص‬patau
‫خصائص‬prader willi
‫بايش بتمي ال‬cow milk ‫عن ال‬breast milk ? ‫باليوتي‬ ‫؟ ر‬
‫ال‬breast milk ‫بقلل نسبة اإلصابة بأي‬infection ? ‫ ؟‬HIV-E COLI - -- ? ! ‫؟ ما عرفناه‬
‫من خصائص ال‬TETRALOGY ‫ ؟ هادا بأسئلة سمي و الجواب انو بكون فيه‬ejection systolic
murmur in 3rd space
‫لو كان طفل‬preterm ‫و شوفت ال‬meconium ‫ أثناء الوالدة ؟ التفسي انو بكون‬listeria
infection
‫سؤال عن خصائص ال‬ORS >> ‫ أحمد‬.‫نسخ لصق من محاضة د‬
‫سؤال عن عالج ال‬Hypernatremic dehydration
‫خصائص ال‬life threatening asthma
‫خصائص ال‬severe asthma
‫سؤال بطلب فيه شو ال‬stage of asthma
‫واحدة حامل و اجاها‬chickenpox ‫ شو العالج ؟‬.. ‫قبل بيومي من الوالدة‬
‫كان يف سؤال عن‬sudden infant death syndrome ‫ شيين‬.‫مباش من محاضة د‬ ‫ ر‬..
‫يل من خصائص ال‬ ‫ كل ما ي‬: ‫سؤال‬acute hepatic failure except ! ‫و ما عرفناه‬
‫سؤال عن ال‬child abuse >> ‫ شيين‬.‫مباش من محاضة د‬ ‫برضو كان ر‬
‫من أسباب ال‬neonatal hypocalcemia ?
‫وحدة ولدت بالبيت و بنتها عندها‬bruising ‫ وكان عندها‬PT ‫و‬PTT ‫عاليي و‬PLT ‫قليلة شو السبب؟‬
vitamen K deficiency
‫سؤال عن الفاكتورز شو الغلط ؟ الجواب انو‬factor 7 prolong PTT
‫سؤال عن يم ال‬type of muccopoltsccharidoses ‫ا يل ما بعمل‬corneal cloudening ?
‫البيب عندو‬
‫بتشك أنو ر ي‬inborn
‫ي‬ error of metabolism ‫يل ما عدا ؟‬‫ازا كان عندو كل ما ي‬
respiratory acidosis
‫من خصائص ال‬congenital hypothyroidism ‫؟‬hypothermia
‫شو الصح‬about congenital adrenal hyperplasia ‫؟ أنو‬17-hyroxy ‫بيحدد ال‬ttt ‫و هادا برضو‬
‫ سمي‬.‫بأسئلة د‬
About asthma ‫شو الصح ؟أنو‬wheeze not diagnostic
‫اجاك ب‬‫ي‬ ‫واحد‬coma ‫ شو بيساعد ع التشخيص؟‬smell of ketones
‫ سنوات ماعدا ؟ بعملش مثلث‬4 ‫يل بيصي عل عمر‬ ‫كل ما ي‬
‫ر‬
About oral exam in newborn ‫ شو الغلط ؟ أنو‬ranula ‫ه نورمال‬ ‫ ي‬.. ‫اش خطي‬ ‫ي‬
‫من خصائص ال‬JRA ? ‫ انو فيها‬erythema marginatum
‫من خصائص ال‬rheumatic fever ‫؟ انو بكون فيها‬sore throat
‫تان عنها من خصائصها ؟ بكون فيها‬ ‫برضو سؤال ي‬aortic / mitral valvitis
About endocarditis ‫؟ انو بكون فيها بال‬blood culture – sterp viridians
‫ايش الصح عن‬VSD ‫؟ انو‬small opening causes loud murmur
‫طفل كل ما نعطيه‬cow milk ‫بصي معاه اسهال شو السبب؟‬lactose intolerance
‫من خصائص ال‬kwashiorkor ‫؟ انو ما بصيفيها‬spasticity
‫يل من خصائص ال‬ ‫كل ما ي‬CP ‫ما عادا ؟ انو بعملش‬blindness
‫شو الغلط‬about beriberi ? ‫ انو مش سببو‬B6
About B12 ‫ شو الغلط ؟‬extrinsic factor increase absorption
‫شو الغلط يف‬diamond-blackfan! ‫؟ أنو يكتشف بعد سنة من الوالدة‬
‫مي بعملش‬microcytic anemia ‫؟‬G6PD
‫سؤال عن ال‬antidote ‫و مي الغلط ؟‬atropine-glycogen
‫ف ال‬klumpke
‫ي‬ ‫ شو الغلط ؟ انو ما بكون بعضالت ال‬intrinsic
‫مي بعملش‬jaundice ‫ف أول يوم ؟‬breast‫ي‬ milk
‫مي‬good prognosis ‫ف ال‬acute ‫ي‬ lymphocytic leukemia ‫؟ موجودين بجدول بالكتاب‬
‫شو الغلط‬about neuroblastoma ‫؟ انو بتكون‬poor prognosis .. ‫يف عمر أقل من سنة‬
‫من خصائص ال‬recurrent functional abdominal pain ‫؟ أنو بكون حوالي ال‬umbilicus
‫من أسباب ال‬botulism ???
About constipation ‫؟ شو الغلط ؟؟ أنو بييد الوزن‬
‫واحد عندو‬tender lymph nodes ‫؟ شو التشخيص‬acute bacterial lymhadenitis
About absence ‫شو الغلط ؟ أنو ال‬EEG ‫ملوش أهمية ! بالعكس الو خاصية معينة‬
‫طفل كلو نورمال و صار عندو‬SEZIURE ‫ شو بتتوقع السبب ؟‬FEVER
‫واحد اجتو‬febrile convulsion ‫مرة واحدة أديش نسبة يصي معاه‬epilepsy 1 ‫لقدام ؟ أقل من‬
‫بالمية‬
‫شو الغلط‬about bacterial diarrhea ‫؟ انها مش‬more common than viral
Male ‫و عندو مرض‬x-linked ‫مي بكون المصاب بالعيلة ؟‬mother's uncle
‫شو الصح‬about hypoxic ischemic encephalopathy ‫؟ انو‬prognosis ‫أسوأ يف ال‬preterm
‫كم جرام بيحتاج ال‬premature 3 ‫اليوتي ؟‬ ‫من ر‬
‫شو الصح‬about premature formula ‫؟ انو فيها بروتي أكي‬
About familial short stature ‫؟ أنو‬bone age ‫مش‬delayed ‫هادا الجواب‬
‫سؤال معطيك فيه‬z-score : ‫و بقول فرسو بيطلع الجواب‬improper feeding recently
‫يل من خصائص ال‬ ‫كل ما ي‬proximal RTA ‫ما عادا ؟‬NEPHROCALCINOSIS
‫شو أحسن وسيلة لتشخيص‬VESICOURTREC REFUX ‫؟‬MCUG
‫شو تعريف ال‬UTI ‫؟ هو وجود ال‬105 ‫باف التعريف‬ ‫و ي‬
About post-strep ‫ ؟ انو ال‬c3 ‫ اسابيع من العالج‬8 ‫بتحسن بعد‬
‫ايش بعملش‬macrocephaly ‫؟‬CMV
‫مي مش‬RISK FOR CANCER ‫؟‬achondroplasia
‫ف ال‬uncomplicated
‫ي‬ PDA ‫ شو ما بصي ؟ الجواب‬plethora in lung
‫ال‬extensor plantar flexion ‫وين مش موجود ؟ يف ال‬lower motor neuron lesion
‫مي من هادي األمراض‬autoimmune ‫؟‬DM type 1
‫ف أي‬stage ‫ بكون ال‬growth spurt ‫ش ؟‬ ‫تبع الفيميل أشع ر‬
‫ي‬ ‫ر‬ ‫ي‬
‫أكي مضاعفات ال‬meningitis ‫؟‬hearing loss
‫مي أكي مسبب لل‬menigeoencephalitis ‫؟‬enterovirus
‫ال بعمل‬ ‫مي ي‬cerebral abcess ‫؟‬vsd – asd- tet ?
‫بالنسبة لل‬neonatal anemia ‫شو تفسيها الصح ؟ أنو‬mainly ‫ناتجة عن تكرس ال‬RBCS
‫مش من أسباب ال‬extracorpuscular hemolysis ‫؟‬autoimmune hemolytic anemia
‫ال بيصي يف ال‬ ‫شو تفسي ي‬HUS ‫؟‬damage in vascular endothelium
‫شو بيصي مع ال‬goat milk ‫؟‬folate deficiency
‫يل يسبب قلة ال‬ ‫كل ما ي‬neonatal platelet ‫ما عادا ؟ أنو األم بتاخد‬warfarine
‫شو بيعملش‬anemia in neonate ‫؟ ال‬sickle
‫ال بقلل ال‬ ‫شو ي‬growth in cystic fibrosis ‫؟ انو بكون عندهم‬pancreatic enzyme
deficiency
‫ف سؤال انو األزما ما بتعمل‬clubbing
‫ي‬
‫واحد عندو‬clubbing + recurrent pneumonia ‫؟ بكون عندو‬cystic fibrosis
‫يل من أسباب ال‬ ‫كل ما ي‬pathological proteinuria ‫ما عادا ؟ ال‬orthostatic
‫يل بخصوص ال‬ ‫كل ما ي‬necrotizing enterocolitis ‫غلط ما عادا ؟ أنو ما بنادي السيجون وقت ال‬
perforation ‫ الزم أناديه من قبل‬.. ‫فقط‬
‫ف سؤال كان عن ال‬sturge‫ي‬ –weber
‫و كان يف سؤال عن‬rett syndrome !!
‫و كان يف سؤال عن‬gingivostomatitis ‫أنور‬.‫مباش من محاضة د‬ ‫كان ر‬
‫شو ال‬pathognomonic of measles ‫؟ هو ال‬enanthem
‫ سؤال عن ال‬pethargy test in TB ‫ برضو جدول‬.. ‫ شهور‬6 ‫امب بكون بوزيتيف يف أكي من‬
‫أنور ر‬.‫بمحاضة د‬
‫مباش‬

o Factor 8 deficiency, Hemophilia A, What is the most common complication


 Neurological defect
 Nasal damage
 Local bleeding blood loss
 Short tendon Muscle

 African-American, High MCV, Other things are normal…….diamond-Blackfan


syndrome
 Hb in newborn….. Fetal Hb is predominant
 IDA………… Never treat before 6 month

 RA….ESR not elevated (F)


 Abdominal pain, Fever, petechial rash on buttocks and lower extremities….
HSP
Arthritis in RF heal without complication.
 Oligomic in CXR filed due to decrease pulmonary blood flow….. TRICUSPID
ATRESIA
 Congenital adrenal hyperplasia…… genitalia
 DKA and Hyperglycemias other electrolyte ….. Hypernatremia.
 DM in young avoid competitive game …….oral hypoglycemic drugs.
 All causes hypochromic microcytic anemia except…G6PD
 Thalassemia minor
 Thalassemia major
 IDA
 G6PD
 Exclusive breast feeding 16 moths baby risk for iron deficiency
 Tracheoesophageal Fistula (TEF) Except proximal type more common ( distal
type more common)
 CF except acts on K channel ( on cl channel)
 Causes of recurrent pneumonia Except
 Kertagner
 Lobar emphysema
 Asthma
 Brutton agammaglubenemia
 Sickle cell anemia
 As above

 Non infective diarrhea except ……..hurshbring ds


 Von will brand ds
 Causes of microcephaly except….
 Causes of macrocephaly except ……..rubella
 Most common cause of AGE….. Rota virus
 Palivizumab, an RSV-specific monoclonal antibody.
 Alport syndrome deafness and hematuria IgA nephropathy.
 PSGN
 Cromolyn sodium mast cells stabilizer
 Infantile spasm Cryptogenic type better prognosis and treatment better than
systemic causes
 Mycoplasma treatment …. Erythromycin
 Pnumatocele the cause of it……. S. aureus

 Two vaccine that does not give together 1 month apart MMR+ varicella
 Encephalitis in Measles 1/1000
 HUS defect in vascular damage
 Wiskot aldor syndrome except neutropenia
 ITP
 Teeth eruption Except … Upper central 1st erupt.
 Vitamin B12 except Intrinsic factors promote absorption

 Hemolytic anemia except


 Increase conjugated bilirubin in blood
 Urobilinogen in urine
 Haptoglobin increase
 Excudate plural effusion except protin fluid/serum < .5

Causes of croup except


RSV
Adenovirus
Parainflunza
Mycoplasma
Adenovirus

 Epiglottitis except Braking cough


 Epiglottitis ….. oxygen and call ENT and anesthesia
 Epiglottitis except family of other member
 Supraglotic disease except….. barking cough

 Chroinc cause of wheezing in child except


 Foreign body
 GERD
 Asthma
 Congenital heart disease
 Bronchiolitis

Female with loos skin in the neck what is investigation…..


 Chromosomal analysis (Turner syndrome)
 Pneumothorax need immediately needle insertion.
 Innocent murmur change with change the position
 Asthma mild persistence

High anion gap except……


 DKA
 RF
 IMD
 Salcylate

Decrease late complement ….. meningococcemia … gonococci infection

Last thing occurs in dehydration


 Wt lost
 Decrease urine output
 Hypotension
 No tears when cry
 Tachycardia

 CXR Bilateral Hyperinflation, no wheezing……. Asthma


 Foreign body in the Rt Bronchus what are the findings in CXR…………… Rt lung
atelectasis and heart shift to left.
 UTI
 Nephrotic syndrome in children proteinuria > 35 g/l
 Minimal change GN
 Coarctation of the aorta ………. Decrease of femoral pulse
 Clubbing except …. Bronchial asthma
 Handedness 3y
 Circle, bridge, 10 cubes…. 3y
 6 Month except stand on furniture
 Familial short stature except….. Delay bone age

Hyponatrimic dehydration except


 Volume depletion stimulate pituitary to produce ADH, which prevent H2O
secretion so correct hyponatremia
 Shift fluids from extracellular to intracellular
 Hypertonic NaCl 3% may use for rapid correction

 Causes of euvolemic hyponatremia except


 Emesis’s cause metabolic alkalosis and hypokalemia… loss of HCL….metabolic
alkalosis….. depletion of volume….. kidney loss K….. hypokalemia

Pattern of respiration except


 Restriction … rapid rate, increase radial volume
 Kussmaul breathing
 Cheyne–Stokes respiration
 Gasping
 Biot's respiration

Lung volume and capacity except

Causes of upper air way obstruction in newborn except


 Choanal atresia
 Viral croup
 Nasal encephalocele
 Laryngeal web
 Peri robin syndrome
 Symptoms of anemia all of the above.
 Hereditary spherocytosis
 Pure RBC aplasia congenital < 1y
 Congestive heart failure except …. Ascites
 Mid parental height = (maternal + paternal +13)/2
 Answer parainfluenza virus
 Wide base gate 15 month reassure the patient
 Question about decrease factors
 Hemophilia A
 Proteinuria in any state Fever, exercise…
 Delay in separation of umbilical cord leukocyte adhesion deficiency (LAD)
phagocytosis
 Answer Enterohaemorrhagic E. coli (EHEC)
 Aplastic anemia
 Congenital anemia more common than acquired
 Fanconi anemia is most common
 Answer poor feeding ( history FTT failure to thrive)
 oculomotor nerve palsy except

Pediatrics midterm exam, 6th year, 2012-2013


50 MCQs
 About HIE all true except: HIE has a poorer prognosis in full term infants than
preterm infants.
 The following can improve kidney function in CKD except: ?growth hormone
Rx?
 Two vaccines if not given at same time should be given at least 1 month
apart: MMR,varicella
 Case ricketssia >>>doxycycline
 True except: preterm formulas have lower protein content
 Regarding APSGN what is true: hypertensive encephalopathy is a recognized
complication
 Scarlet fever vs. Kawasaki:? pharyngeal swab useful in scarlet fever but not
Kawasaki
 Delayed bone age, short, otherwise normal: constitutional
 Minimal change nephritic true except: cyclophosphamide useful to induce
remission in recurrent cases/ oral diuretics for edema, proteinuria,RBC casts
absent, HTN rare.
 False combination: photosensitivity & riboflavin
 Planter reflex absent in: LMNL
 About SIDS: boys>girls
 No corneal cloudness in: Hunter
 False about TB effusion: polymorphs present in fluid
 A –ve mother, jaundice baby, past similar hx: RH incompatibility
 Case Prader willi synd.: paternal chr 15 deletion
 Causes of ataxia except: ?folic acid deficiency
 Ride tricycle, draw circle, mimic bridge: 36 mo.
 Waves bye, one word without meaning: 10mo.
 ORS true except: depends on coupled transport of potassium-glucose
 Spastic diplegia> periventricular leukomalacia
 TPN true except: dextrose 20% in peripheral line
 About primitive reflexes true except: when rotating head to left side in tonic
neck reflex the right arm extends..
 False: erythma toxicum rare….
 Term baby, CS, fluid in transvers fissure, prominent pulmonary vasculature:
transient tachypnea of newborn
 About neurofibromatosis 1 what is true: AD/ mutation 50%, optic glioma
rare/ children usually mentally retarded
 Insulin dependent DM what is false:? disease rapidly occurs after onset of
islet cell antibodies
 Tubular proteinuria except: DM
 Glomerular proteinuria: polycystic kidney
 Girl in ICU with congenital heart disease developed acute renal failure, the
following can be causes of her ARF except: Ecoli cystitis
 Case sudden abdominal pain, bloody diarrhea..next step: ?air contrast enema
 Hepatitis viruses true except: ????chronicity in children 90% and adults
10%????
 Child with meningomyelocele, parents asked about risk of hydrocephalus:
risk higher with higher lesions, risk lower with higher lesions, according to the
cause, risk can’t be determined at birth
 Hypotonia except: ??glioma of motor cortex, cerebellar, syringomyelia
 Congenital toxoplasmosis what is true: microcephaly&hydrocephalus can
occur/ mother ill, headache, fever/ recurrent in subsequent pregnancies
 CAH true except: low 17 hydroxyprogesterone..
 Hypokalemia except: 3 B hydroxysyeroid dehydrogenase deficiency
 Asthma true except: in mild persistent asthma day time attacks up to 2
times/week, night time up to 2 times/month
 JRA what is false: ???association with HLA DR3…/cervical spine involvement
rare??
 Portal HTN mechanisms true except: collaterals lead to portal HTN
 Measles true except: ?morbidity more in 2-20 yrs
 Poor prognosis in leukemia: mediastinal mass
 Low ferritin, irritability…except: B thalassemia
 In hemolytic anemia: high urine urobillin
 Newborn examination true except: dull tympanic membrane indicates AOM
 True except: in klumpke palsy elbow not functioning
 False about HIE: increased intracellular Na, Ca have protective effect
 Complex partial seizures true except: dream like state/ visual hallucination,
loss of consciousness, oculogyric reflex, chewing and swallowing
 Q about NTD???
 Q about acute fulminant hepatic failure????

Pediatrics midterm exam, 6th year, 2012-2013

Which of the following support the diagnosis innocent murmur:


A. Present only in diastole
B. Fixed splitting of second heart sound
C. Is loud in high output states
D. Heard all over the praecordium
E. The venous hum is best heard at the apex

A 4 month old with vitamin D deficiency rickets would be expected to show all
of the following except:
A. Craniotabes
B. Bow legs
C. Rosary
D. Low serum phosphate level
E. High alkaline phosphate levels

3- A 5 year old male is hospitalized in January with fever and seizures. LP


reveals clear CSF with 47 WBCs/mm3 all of which are lymphocytes. On PE he
appears obtunded but arouses with painful stimuli. Neurologic exam reveals no
focal findings. Which diagnostic test is most likely to reveal this child’s illness?
A. Bacterial culture of CSF
B. PCR test of CSF for HSV
C. Strepto.pneumoniae bacterial antigen test of CSF
D. Viral culture of CSF
E. Viral culture of nasopharyngeal and rectal swabs

4- A previously healthy 2 year old is brought to ED because her mom is unable


to awaken her for 45 minutes. She has not been ill. PE reveals an afebrile,
hypotonic child who withdraws her hand from painful stimuli but does not
spontaneously open her eyes. RR is 36/min, BP is 92/64. What is the next best
thing to obtain?
A. BUN
B. CXR
C. EKG
D. EEG
E. Toxicology screen

5- A 2 week old presents with tachypnea, poor perfusion, gallop rhythm,


diminished pulses, and hepatomegaly. ABG shows metabolic acidosis.
Echocardiography reveals critical Aortic Stenosis.
What intervention is most likely to stabilize the infant’s condition?
A. Dobutamine
B. Epinephrine
C. Nitric Oxide
D. 100% oxygen
E. Prostaglandin E-1

6- In congenital diaphragmatic hernia:


A. Hernias most commonly occur on the left
B. Persistent fetal circulation occurs uncommonly
C. Associated congenital anomalies are common
D. Most present between 12 and 24 hours of age
E. Pulmonary hypoplasia is rarely the cause of death

7- The average time of closure of the anterior fontanel is:


A. 12 months
B. 14 months
C. 16 months
D. 18 months
E. 22 months

8- A newborn female has an open neural tube defect, low set ears, VSD, and rib
and vertebral column malformations.
Which of the following MATERNAL conditions was most likely present during
pregnancy?
A. Alcoholism
B. Diabetes mellitus
C. Hypothyroidism
D. Iodine deficiency
E. Syphilis

9- The arthritis of acute rheumatic fever usually:


A. Is monoarticular
B. Heals without deformity
C. Appears after the fever subsides
D. Is seen only in patients with concurrent carditis
E. Involves large and small joints equally

Intussusception in childhood:
A. Has as the earliest sign the passage of red current jelly stools
B. Has a peak incidence in the first 3 months of life
C. Requires operative reduction in the majority of cases
D. May be initiated by ameckel's diverticulum
E. It has association with umbilical hernia

10- Intussusception in childhood except:


A. Is characterized by screening attacks with pallor
B. Has ,as the sign late, the passage of red-current jelly stools.
C. Has a peak incidence in the first two months of life .
D. Is accompanied by enlargement of mesenteric lymph nodes.
E. May be caused by an adenovirus infection.

11- The following statement about hepatitis are true except


A. Hepatitis c rarely leads to chronic conditions
B. One single raised antibody titre confirms the diagnosis of hepatitis A
C. Hepatitis A vaccine is very effective
D. Hbs AB persistent for more than 1o years
E. Hbe Ag is associated with increased infectivity

12- Useful indication of dehydration in infants include :


A. Assessment of intraocular tension by palpation
B. Skin turgor over the dorsum of hand
C. Palpation of the anterior fontanelle
D. Blood pressure
E. Serum sodium concentration

13- A 2 year old boy with spastic diaplegia from of cerebral palsy is being
evaluated. MRI for his brain is most likely to show :
A. Multicyctic encephalomalacia
B. Periventricular leukomalacia
C. Normal anatomy
D. Basal ganglia abnormalities
E. Agenesis of the corpus callosium

14- Acute fulminant hepatitis failure ,all are true except:


A. The commonest causes are viral hepatitis and medications-induced
B. The hallmark is the presence of acute hepatic encephalopathy
C. The absence of jaundice is against the diagnosis
D. There are long listed complications and those usually complicates the
picture further
E. The patient should be managed in an intensive care unit or a high
dependency unit once the prolonged
15- 3 year old boy with rash and fever He has a 2 day history of headache
………….. and is being treated with amoxicillin for otitis media , finding include
………. Alert child :temperature 39,5C :injected conjunctiva, a red nonpetechial
maculopapular rash on his wrists and ankles ; hyponatremia and mild ……… of
the following the most appropriate treatment is
A. Atropine
B. Induction of amoxicillin
C. Doxycycline
D. Immnoglobin
E. Trimethoprim sulfamethoxazole

16- The following are poor prognosis sign of acute leukemia except :
A. Male child
B. Child more than 10 years
C. White blood cell count less than 50 000
D. Presence of philadephia chromosome
E. Child with mediastinal mass

17- a 11 month old child presents with irritability and pallor, he has Hbg level of
7 mg/dl and hypochromic microcytic blood film, serum ferritin is below normal,
the following state event are true except:
A. He probably has beta thalassemia major
B. A dilatory history is important
C. Occult blooding should be excluded
D. Low serum ferritin depletion of iron stores

18- Which of the following is a feature of hemolytic anemia


A. Raised haptoglobin
B. Raised conjugated bilirubin
C. Hypochronic erythrocytes
D. Urobilin in urine
E. Low reticulocytes

19- which of the following is true of post streptococcal glomerulonephritis :


A. Usually associated with hypoabuminemia
B. Life long penicillin prophylaxis is recommended
C. Hypertensive encephalopathy is recognized complication
D. Abnormalities of serum complement usually persist for more than 3
months
E. ESR is the most useful marker of post streptococcal infection

20- a 12 year old girl is at the 5th percentile for height .she is an otherwise
healthy child. The bone age is greater than two and half standard deviation
below the chronologic age. Of the following, the most likely cause of patient’s
short stature is
A. Constitutional growth delay
B. Genetic short-stature
C. Growth hormone deficiency
D. Hypothyroidism
E. Ulcerative colitis
21- all are correct for measles except
A. Can cause activation of pulmonary T.B
B. Can cause suppression of purified protein derivative test
C. Morbidity is more in the age 2-20 years
D. Fever subsides after onset of rash
E. Enanthem is pathognomonic for the disease

22- all of the following are cause of tubular proteinuria except:


A. Lead poisoning
B. Juvenile D.M
C. Reflex nephropathy
D. Wilson disease
E. Interstistial nephritis

23- all the following viruses have been associated with congenital infection
except :
A. Cytomegalovirus
B. Rubella virus
C. Hepatitis B virus
D. Herpes simplex
E. Rotavirus

24- the most serious acute medical out come of the drowning is
A. Pulmonary edema
B. Acute renal failure
C. Hypoxic brain injury
D. Blood loss
E. Cardiac arrhythemia

25- to be effective for preventing mylomeningocele ,administration of folic acid


needs to begin
A. By 3 months of gestation
B. At the first missed period
C. Before conception
D. By 30 days of gestation
E. At 3 months prior to delivery
26- neonatal hypocalcaemia all of the following are causes except
A. Birth asphyxia
B. Infant of diabetic mother
C. Intracerbral hemorrhage
D. Sepsis
E. Hypophosphatemia
27-which of the following chemotherapy is not consistent with its side effect
A. Vincristine – peripheral neuropathy
B. Bleomycin – pulmonary fibrosis
C. Methotrexate- muccositis
D. Adrenomycin – nephropathy
E. Cyclophosphamide- hemorrhagic cystitis
28- the following are true regarding oral rehydration solution (ORS) except
A. The ORS relies on the coupled transport of potassium and glucose in the
intestine
29- the following are true regarding TPN except
A. The dextrose content of peripheral TPN cannot be greater than 20%

30- A 3-yr-old girl with congenital heart disease develops acute renal failure
during a prolonged stay in an intensive care unit. Possible contributory factors
include all of the following except:
A - E. coli cystitis
B - Nephrotoxic antibiotics
C - Hypotensive episodes
D - Contrast agent used for cardiac catheterization
E - Congestive heart failure

31- which of the following is abnormal in cardiovascular examination of


newborn4th heart sound

32- all of the following are true regarding breast feeding except:
A. Milk contain free oxygen radicals, which kill gardia and entamebia

33- all of the following are true regarding HIE except


A. HIE has a poorer prognosis in full term infants than preterm infants.

Pre-test in Pediatrics

1.Autosomal dominant inheritance is present in all of the following Except:


A. von willebrand's disease.
B. Color blindness.
C. Dystrophia myotonia.
D. Hereditary spherocytosis.
E. Polyposis coli.

2.All of the following statements about DNA structure are correct Except:
A. there are two purine bases called adenine and Thymidine.
B. adenine always pairs with thymidine.
C. an amino acid codon consists of three bases.
D. each strand of DNA has a sugar-phosphate backbone with projecting bases.
E. there are 64 possible codons.

3.All are correct for nitric oxide Except:


A. synthesis is stimulated by TNF( tumor necrosis factor).
B. it has an inducible form in mononuclear phagocytes.
C. is inherited by the lipopolysaccharide of gram negative bacteria.
D. is a neurotransmitter.
E. exists in a free radical form which can be neurotoxic.

4.The following is correct for interleukin-1:


A. can not be detected in individuals in the absence of inflammation.
B. Blood levels are a usuful index of disease activity in rheumatoid arthritis.
C. Infused intravenously leads to hypotension.
D. Is a neurotransmitter.
E. Antagonizes the effect of tumor necrosis factor.

5.All are correct for polymerase chain reaction (PCR) Except:


A. used in diagnosis of specific mutations in genetic diseases.
B. Guthrie blood spot cards can be used to perform retrospective genetic
diagnosis.
C. DNA fragments up to one megabase can be successfully amplified.
D. RNA as well as DNA can be used in PCR.
E. Analysis of genetic abnormalities using a single cell can be performed.

6.All of the following infections may be aquired from contact with animals or
animal products Except:
A. leptospirosis.
B. Tularaemia.
C. Plague.
D. Typhoid fever.
E. Brucellosis.
7.The following is correct for Hepatitis C virus:
A. is a DNA virus.
B. Has a seroprevalence rate of 60-90% in haemophiliacs.
C. Transmission is unknown in 70% of patients.
D. Rarely leads to chronic liver disease.
E. May be treated effectively with high dose acyclovir.

8.HIV is reliably inactivated by all of the following Except:


hot air oven.
Glutaraldehyde.
Hypochlorites.
Autoclave.
Chlorhexidine.

9.The following about spinal cord is correct:


the subdural space contains the CSF.
Segment T12 lies at the level of vertebral body T11.
It transmits two-point discrimination sensations in the lateral spinothalamic
tract.
Hemisection results in contralateral upper motor neuron paralysis below the
level of the lesion.
Hemisection results in contralateral loss of pain and temperature sensation
below the level of the lesion.

10.The following is correct about vasoactive intestinal polypeptide (VIP):


is secreted by beta islets cells in the pancreas.
May be secreted by a bronchogenic carcinoma.
Stimulates gastric acid secretion.
Enhances small intestinal reabsorption of water and electrolytes.
Normal serum level is 3000pg/ml.

11.Positive clinitest reaction may occur with all Except:


L-dopa therapy.
Salicylate therapy.
Alkaptonuria.
Galactosaemia.
Aminoaciduria.
12.The simultaneous administration of the following pairs of drugs are
potentially hazardous Except:
oral contraceptives and rifampicin.
Warfarin and chloral hydrate.
Digoxin and erythromycin.
L-dopa and phenelzine.
Digoxin and propranolol.

13. The data generated from observational assessment of a child's overall state
of well-being are greatly influenced by the child's developmental stage. This is
most importantly shown in which observational data?
A. Hydration
B. Skin color
C. Visual response
D. Respiratory effort
E. Motor tone

14. The approach taken by a physician to eliciting data during a physical


examination is linked to a child's stage of development. This is importantly
demonstrated in the examination of
A. the pharynx
B. the ears
C. arterial pulsations
D. the chest
E. the lumbrosacral spine

15. Behavioral states in the newborn period include


A. quiet and active sleep
B. drowsy and alert
C. fussy and crying
D. all of the above
E. none of the above

16. A developmentally normal child who just hops on one foot, copies a cross
and square, tells a story, and goes to the toilet alone is probably what age?
A. 24 months
B. 36 months
C. 48 months
D. 60 months
E. 72 months

17. Handedness is usually determined by what age?


A. 2 to 4 months
B. 6 to 12 months
C. 15 to 18 months
D. 20 to 24 months
E. 36 to 48 months

18. The most rapid increase in height in girls is found in which Tanner SMR
stage?
A. 1
B. 2
C. 3
D. 4
E. 5

19. All of the following are diagnostic soft signs of ADHD EXCEPT
A. mixed hand preference
B. impaired balance
C. dysdiadochokinesia
D. astereognosis
E. none of the above

20. Separation anxiety disorder is most likely to be associated with


A. aggressive behaviors
B. poor academic performance
C. school refusal
D. increased risk of schizophrenic outcome
E. maternal neglect

21. Which of the following statements correctly describes children with


attentional dysfunction?
A. They are always hyperactive.
B. Their problems are mainly behavioral.
C. They often have additional neurodevelopmental dysfunctions.
D. They all respond to treatment with methylphenidate.
E. Their difficulties lessen significantly after puberty.

22.All of the following are correct except:


Night terrors occurs during REM sleep.
Night terrors are associated with autonomic activity.
Night terrors are associated with somnambulism.
Narcolepsy is associated with sleep paralysis.
Narcolepsy is associated with daytime naps.

23.All of the following are conduct disorders EXCEPT:


Continuous arguing.
Chronic stealing.
Fire setting.
Rape.
Property destruction.

24.All are correct about bruise appearance and its age in days EXCEPT:
Yellow-brown (6-7 days).
Green (8-10 days).
Red-blue (1-2 days).
Blue-purple (4-5 days).
Resolved (10-15 days).

25.All of the following are true association with the corresponding vitamin
deficiency EXCEPT:
Photosensitivity – Riboflavin.
Bitot spots – A.
Tender nerves – Thiamine..
Alpoecia – Biotin
Cerebellar ataxia – E

26.All are correct associations with the corresponding mineral deficiency


EXCEPT:
Zinc – Dwarfism.
Chloride – Alkalosis.
Selenium - Cardiomyopathy.
Iodine – Caries.
Phosphorus – Rickets.

27. Which of the following vitamins is in higher concentration in cow's milk than
in human milk?
A. A
B. C
C. E
D. K
E. B6

28. The normal fractional excretion of sodium in children older than 1 year on a
diet containing regular amounts of sodium is
A. 1%
B. 7%
C. 10%
D. 15%
E. 25%

29. A 10-year-old has a Glasgow Coma Scale score of 4 and develops irregular
respirations after head trauma. The next important step in the care of this
patient is to
A. perform endotracheal intubation
B. administer 20 mL/kg of lactated Ringer solution
C. administer naloxone
D. administer mannitol
E. obtain a head CT scan

30. Sea-water drowning can lead to all of the following Except:


Inactivates surfactant.
Hemolysis.
Pulmonary aspiration.
Hypothermia.
Ventilation-perfusion mismatch.

31.In Trisomy 13 all are correct Except:


Hypotonia.
Cleft lip.
Hypoplastic ribs.
Risk of abortion.
Meiotic nondysjunction.

32.All of the following diseases are transmitted by X-linked recessive inheritance


Except:
Hemophilia B.
Hemophilia A.
Vitamin D-resistant rickets.
Bruton hypogammaglobulinemia.
Color blindness.

33. The corresponding urine odor is correct in all of the following inborn errors
of metabolism except:
Glutaric academia (type II) – sweaty feet.
Phenylketonuria – mousy.
Methionine malabsorption – cabbage.
Trimethylaminuria – Tomcat urine
Hawkinsinuria – swimming pool.

34.Heparan sulfate is found in the urine of the following syndrome:


Scheie syndrome.
Sanfilippo syndrome A.
Morquio syndrome A.
Morquio syndrome B.
Maroteaux-Lamy syndrome.

35.The following is correct for disruption dysmorphology:


Multifactorial recurrence risk of 2-5%.
Error in morphogenesis.
Congenital hip dislocation.
Talipes foot disorder.
Porencephaly.

36. A 1700-g infant is born at 36 weeks' gestation complicated by severe


oligohydramnios. The Apgar scores are 3 at 1 minute and 5 at 5 minutes. The
infant requires endotracheal tube placement as part of the resuscitation and
continued mechanical ventilation to improve the arterial blood gases. At 1 hour
of age, the infant shows acute deterioration with cyanosis, bradycardia, and
hypotension. The most likely diagnosis for this acute change is
A. patent ductus arteriosus
B. intraventricular hemorrhage
C. hypoglycemia
D. pneumothorax
E. respiratory distress syndrome

37.The following features are related to the corresponding congenital infection


Except one:
Myocarditis – enteroviruses
Cerebral periventricular calcifications – toxoplasmosis
Patent ductus arteriosus – rubella
Limb hypoplasia – varicella
Fetal anemia – parvovirus

38.All of the following signs and symptoms are correct for bulimia Except:
frequent weight fluctuations.
Esophagitis.
Amenorrhea.
Recurrent episodes of binge eating.
Onset at puberty.

39.Lupus-like syndrome is related to the following complement deficiency:


C3.
C1q.
C1 inhibitor.
Properdin.
C5-9.

40.All of the following are matching with the corresponding immunodeficiency


Except:
Fatal echovirus encephalitis – severe combined immunodeficiency(SCID)
Splenomegaly – common variable immunodeficiency
High frequency of autoimmune disorders – hyper IgM
Rib cage anomalies – Adenosine deaminase (ADA) deficiency
Elevated IgE levels and eosinophilia – Omenn syndrome

41. Eosinophilia is observed in all of the following EXCEPT


A. Giardia infection
B. Toxocara infection
C. drug hypersensitivity
D. periarteritis nodosa
E. allergy

42. Associated epidemiologic facts about asthma include all of the following
EXCEPT
A. another name could be chronic desquamating eosinophilic bronchitis
B. a child with two affected parents has a 50% risk of asthma
C. asthma is universally present in monozygotic twins
D. asthma is transferred with lung transplantation
E. both large and small airways are affected

43.All of the following are part of the revised diagnostic criteria for SLE EXCEPT
A. nonerosive arthritis
B. lymphopenia
C. Raynaud phenomenon
D. pleuritis
E. oral ulcers

44.Spondyloarthropathy is a feature in the following type of rheumatoid


arthritis:
A. Pauciarticular JRA type I
B. Pauciarticular JRA type II
C. Systemic-onset JRA
D. Polyarticular rheumatoid factor positive
F. Polyarticular rheumatoid factor negative

45. All of the following are diagnostic features of Kawasaki disease EXCEPT
A. generalized lymphadenopathy
B. fever for at least 5 days
C. nonpurulent conjunctivitis
D. desquamation of the fingers
E. polymorphous rash

46.Painful red nodules in the thighs is a feature of the following disease:


fasciitis.
Relapsing panniculitis.
Behcet syndrome.
Mixed connective tissue disease.
Erythema nodosum.

47. Therapy of human herpesvirus 6 (HHV-6) infection is associated with all of


the statements EXCEPT
A. therapy is generally symptomatic with antipyretics
B. therapy may involve anticonvulsants for recurrent febrile seizures
C. in life-threatening cases, in vitro data would suggest a trial of ganciclovir
D. there are no controlled trials of treatment of HHV-6 in immunocompromised
persons
E. Oral acyclovir is indicated to shorten the duration and prevent complications
of roseola

48. After diagnosis of pertussis in a toddler, erythromycin should be given to the


patient and to which family members?
A. Only those with a cough
B. Only those younger than 7 years
C. Only those who are incompletely immunized
D. Only those with compromised immunity
E. All regardless of age, symptoms, or immunization status

49. The differential diagnosis of diarrhea that persists for more than 2 weeks
includes
A. S. dysenteriae serotype 1, V. cholerae, Pleisomonas shigelloides
B. enterotoxigenic E. coli, enterohemorrhagic E. coli
C. enteropathogenic E. coli, enteroaggregative E. coli, nontyphi salmonella,
Aeromonas caviae
D. enteroinvasive E. coli, Rotavirus, Norwalk virus
E. C. jejuni, S. flexneri, Helicobacter pylori

50. Positive fecal leukocyte findings in a child with diarrhea allows the clinician
to make a presumptive diagnosis of
A. shigellosis
B. S. typhi
C. Entamoeba histolytica
D. Yersinia enterocolitica
E. colitis

51.All of the following are causing mixed (cholestatic) jaundice Except:


Cystic fibrosis.
Gilbert disease.
Zellweger syndrome.
Galactosemia.
Biliary atresia.

52.In malabsorption all of the following are correctly matching with the
corresponding disease Except:
A. Acanthocytes - Abetalipoproteinemia
B. Pellagra-like rash - Hartnup disease
C. Lymphopenia - Shwachman-Diamond syndrome
D. Chronic sinopulmonary disease - Cystic fibrosis
E. Responds to oral zinc sulfate -Acrodermatitis enteropathica

53.In Crohn disease all are correct Except:


fever, weight loss, no diarrhea.
Ileal involvement.
Cured by colectomy
Fistula formation.
Risk of cancer.

54.All are correct for epiglottitis Except:


A. caused by Haemophilus influenzae type B
B. Barking cough
C. Drooling
D. Thumb sign in x-ray.
E. Requires urgent endotracheal intubation

55. A male born at term after an uncomplicated pregnancy, labor, and delivery
within a few hours develops severe cyanosis requiring supplemental oxygen and
supported ventilation. Results of routine cultures are negative. The chest
roentgenogram reveals a normal heart shadow and a fine reticulonodular
pulmonary infiltrate radiating from the hilum. Family history reveals that a male
and a female sibling with a similar clinical course died at 2 and 4 months of age,
respectively. What is the most likely diagnosis?
A. Neonatal alveolar proteinosis
B. Neonatal herpes simplex infection
C. Type II glycogenosis
D. Meconium aspiration syndrome
E. Carnitine palmityltransferase deficiency
56.Ebstein anomaly is associated with all of the following EXCEPT
A. dysrhythmias
B. patent ductus arteriosus (PDA) dependence in neonates
C. massive heart size on radiographs
D. VSD
E. tall, broad P waves

57.All are correct for asplenia syndrome Except:


A. Severe cyanosis
B. Biliary atresia.
C. Decreased pulmonary blood flow
D. Malrotation (intestinal)
E. Howell-Jolly bodies

58.The type of anemia is correct in all of the following diseases Except:


1. Thalassemia - microcytic
2. Aplastic anemia - normocytic
3. Copper deficiency- microcytic
4. Hypothyroidism - normocytic
5. Orotic aciduria – macrocytic

59.Thrombocytopenia is found in all of the following Except:


A. Aspirin toxicity
B. ITP
C. Wiscott-Aldrich syndrome
5. Kasabach-Merritt syndrome
6. Disseminated intravascular coagulation (DIC)

60.The following findings are correct for neuroblastoma Except:


A. Hypertension
B. Ataxia
C. Diarrhea
D. Opsoclonus
E. Bilateral disease.

61. A 5-year-old presents with a 1-month history of bone pain, increased


bruising, swollen lymph glands, and recurrent fever. Laboratory studies reveal a
hemoglobin value of 6.8 g/dL, platelet count 60,000/mm3, and leukocyte count
3500/mm3 with 2% neutrophils and atypical cells described. The most likely
diagnosis is
A. acute myelocytic leukemia
B. Wilms' tumor
C. Hodgkin disease
D. aplastic anemia
E. acute lymphoblastic leukemia

62. The most frequent cause of graft loss in pediatric renal transplant recipients
is
A. trauma to the graft
B. recurrence of the original renal disease in the graft
C. technical difficulties
D. infection
E. rejection reaction

63. A 4-year-old male developed an upper respiratory tract infection that was
followed in 2 weeks by general edema. His blood pressure is normal. Urinalysis
reveals 2 to 5 red blood cells per high-power field and 4+{plus} protein. His BUN
is 19 mg/dL, creatinine 0.6 mg/dL, cholesterol 402 mg/dL, serum albumin 0.9
g/dL, antistreptolysin O titer 1:16, and C3 92 mg/dL. The most likely diagnosis
would be
A. poststreptococcal glomerulonephritis
B. membranous glomerulonephritis
C. minimal lesion nephrotic syndrome
D. membranoproliferative glomerulonephritis
E. focal sclerosis

64. The presence of renal parenchymal scarring due to vesicoureteral reflux is


best determined by
A. DMSA scan
B. renal ultrasonography
C. VCUG
D. CT scan
E. intravenous pyelography

65.Which of the factors listed below is most likely to contribute to renal


deterioration in an infant with myelomeningocele?
A. The level of the lesion
B. The child's gender
C. High intravesical pressure
D. The presence of hydrocephalus
E. All of the above

66. A 12-year-old female has a hypoglycemic seizure, weakness, and increased


cutaneous pigmentation. She is also noted to have a chronic history of
mucocutaneous candidiasis, which is especially severe on her nails. In addition,
she has been on thyroid replacement medication since the age of 9 years. The
most likely diagnosis is
A. insulinoma
B. growth hormone deficiency
C. AIDS
D. autoimmune polyendocrinopathy
E. DiGeorge syndrome

67.. Male pseudohermaphroditism in neonates is associated with all of the


following syndromes EXCEPT
A. camptomelic
B. WAGR
C. Denys-Drash
D. Swyer
E. 11-hydroxylase deficiency

68. Physical findings in Graves disease include all of the following EXCEPT
A. motor hyperactivity
B. cold intolerance
C. tremor
D. smooth, flushed, warm skin
E. tachycardia

69. A child is below the third percentile for height. Growth velocity is normal,
but chronologic age is greater than skeletal age. This condition is called
A. primary hypopituitarism
B. secondary hypopituitarism
C. constitutional delay in growth
D. genetic short stature
E. primordial dwarfism
70. Causes of megalocephaly include all of the following EXCEPT
A. thalassemia
B. chronic subdural effusions
C. hydrocephalus
D. Canavan disease
E. congenital CMV

71. A 4.3 kg infant is delivered to a woman whose diabetes mellitus is poorly


controlled.
Of the following, the MOST likely neonatal manifestation of maternal diabetes is
A. diabetic ketoacidosis
B.Hirschsprung disease
C.hypercalcemia
D.polycythemia
E.renal vein thrombosis

72. Of the following, the condition that is MOST likely to present with seizures
during the first 24 hours of life is
A .fetal alcohol syndrome
B .herpes simplex infection
C. hypoxic-ischemic encephalopathy
D. organic acidemia
E. urea cycle defect

73.Characterized by significant hypotonia in infancy, which evolves to a pattern


of overeating and obesity by
six months to a year, with variable mental retardation, characteristic facies,
small hands and feet, and
hypogonadism; this defect is a result of abnormalities on:
a) maternal chromosome 15q11-15q13
b) paternal chromosome 15q11-15q13
c) maternal chromosome 7p11-7p13
d) paternal chromosome 7p11-7p13
e) maternal chromosome 17q13-17q15

74. Patients deficient in early complement elements have a susceptiblilty to:


a) viruses
b) fungus
c) encapsulated bacteria
d) intracellular bacteria
e) gonococcus and meningococcus

75. A patient presents with recurrent infections with S. aureus and Serratia
marcescens in the form of
cutaneous abscesses, lymphadenitis, pneumonia complicated by empyema, and
peri-rectal abscess. His
defect is in which enzyme:
a) Super-oxide dismutase (SOD)
b) NADPH oxidase
c) catalase
d) G6PD
e) phosphofructokinase (PFK)

76. A nine year old boy presents to your office after a camping trip with his boy
scouts group one week ago. He looks fairly toxic with fever of 39.5C, headache,
myalgia, nausea, vomiting, and an
erythematous macular rash on the wrists, ankles and proximally on the trunk.
The best treatment for this illness is:
a) Ceftriaxone
b) Gentamycin
c) Acyclovir
d) Tetracycline
e) Aztreonam

77. A teenage boy presents to your office with a history of decreasing school
performance and increasingly
labile mood. Mom states he seems “confused” much of the time. On Physical
Exam he is jaundiced, tall
and thin, has mild hepatomegaly and slit-lamp exam shows Kayser-Fleisher rings
in both eyes. Laboratory
evaluation would most likely show which abnormality:
A. Absence of albumin
B. Excessive albumin
C. Decreased ceruloplasmin
D. Increased ceruloplasmin
E. Hepatic cell in the blood

78. Which of the following is not associated with Neurofibromatosis-1?


a. lisch nodules
b. axillary freckling
c. ash-leaf spots
d. optic gliomas
e. all of the following are associated with NF-1

79. What is the typical order of events in puberty for females?


a. thelarche, adrenarche, growth spurt, menarche
b. adrenarche, thelarche, growth spurt, menarche
c. thelarche, adrenarche, menarche, growth spurt
d. thelarche, growth spurt, adrenarche, menarche
e. none of the above is correct.

80. The parents of a healthy term baby ask you at the 12-month health
supervision visit what they should expect of the baby developmentally by the
15-month visit.
Of the following, the milestone MOST likely to be met by 15 months of age is
A. drawing a circle
B. drinking from a cup
C. having a vocabulary of at least 50 words
D. throwing a ball overhand
E. walking well without tripping

81.A 6-year-old girl can write her name and can count 10 objects.
Of the following, the MOST likely additional activity of which she is capable is:
A. hitting a baseball
B. knowing her right hand from her left
C. making a simple meal
D. printing neatly in small letters
E. sounding out words while reading

82.A 12-year-old girl is at the 5th percentile for height. She is an otherwise
healthy child. The bone age is greater than two standard deviations below the
chronologic age.
Of the following, the MOST likely cause of this patient's short stature is:
A. constitutional growth delay
B. genetic short stature
C. growth hormone deficiency
D. hypothyroidism
E. ulcerative colitis

83. The decreased incidence of enteric infections noted in breastfed infants


compared with formula-fed infants is MOST likely due to the
A. more alkaline stool pH in breastfed infants
B. nutritional benefits of human milk on the infant's immune system
C. predominance of Bacteroides and Clostridium in the gut of breastfed
infants
D. presence of protective antibodies against enteric infection in human milk
E. sterility of human milk

84. You note that a previously healthy 15-month-old boy appears pale. He has
been fed goat milk exclusively since birth. Laboratory findings include:
hemoglobin, 6.1 g/dL; white blood cell count, 4,800/mm3; platelets,
144,000/mm3; mean corpuscular volume, 109 fL; and differential count, 29%
neutrophils, 68% lymphocytes, and 3% monocytes. There also is
hypersegmentation of the neutrophils.
Of the following, the MOST likely cause of the hematologic abnormalities is
A. acute lymphoblastic leukemia
B. Fanconi anemia
C. folate deficiency
D. iron deficiency
E. vitamin B12 deficiency

85. You are evaluating a 4-year-old boy for bowing of the legs and short stature.
Findings include: creatinine, 0.4 mg/dL; calcium, 9.2 mg/dL; phos-phorus, 2.5
mg/dL; alkaline phosphatase, 950 IU/L; intact parathyroid hormone, 40 pg/mL
(normal, 10 to 60 pg/mL); 25-hydroxyvitamin D, 30 pg/mL (normal, 12 to 40
pg/mL); 1,25-dihydroxyvitamin D, 30 pg/mL (normal, 25 to 45 pg/mL); and
urinary tubular reabsorption of phosphorus, 50% (normal, >85%).
The MOST likely cause of these findings is
A. familial hypophosphatemic rickets
B. hypophosphatasia
C. renal osteodystrophy
D. vitamin D-deficiency rickets
E. vitamin D-dependent rickets

86.The mother of a 1-week-old infant asks your advice concerning infant


formula. After receiving cow milk-based formula, her child developed hives,
irritability, and vomiting. Both the mother and older sibling received “special
formulas” as infants because of milk and soy intolerance.
Of the following, your best INITIAL recommendation is to give the infant a trial
of a(n)
A. different type of lactose-containing cow milk-based formula
B. elemental formula
C. formula with hydrolyzed cow-milk proteins
D. lactose-free, cow milk-based formula
E. soy protein-based formula

87. A 6-month-old boy developed a weepy, crusted dermatitis around the eyes,
nose, mouth, diaper area, hands, and feet about 4 weeks after being weaned
from human milk to formula. He is listless, recently developed diarrhea, and has
stopped gaining weight. In addition to the dermatitis, he has sparse hair that is
fine and lightly pigmented.
Of the following, this constellation of findings is MOST consistent with a
deficiency in
A. copper
B. thiamine
C. vitamin A
D. vitamin C
E. zinc

88. A 42-year-old primigravida is in her 16th week of pregnancy. She is


concerned about the possibility of her child having myelomeningocele.
Of the following, the MOST useful diagnostic evaluation is
A. amniocentesis
B. chorionic villus sampling
C. cordocentesis
D. fetal ultrasonography
E. maternal alpha-fetoprotein screening

89. A female infant is born with unilateral cleft lip and palate. Findings on the
remainder of the physical examination are normal, and you inform the parents
that this apparently is an isolated birth defect.
During counseling of the family about their risk for having another similarly
affected child, the statement that you are MOST likely to include is that
A. because the defect is isolated, their risk is no greater than that of any
other couple
B. cleft lip and palate is a multifactorial trait that has a 4% risk of recurrence
C. the recurrence risk can be estimated only after a chromosome analysis has
been obtained
D. the risk is increased for future females, but not males
E. the risk would be increased only if one of the parents had cleft lip and
palate

90. A newborn male has multiple congenital anomalies, including bilateral


clubfeet, amputation of the fourth and fifth digits on the right hand, and distal
syndactyly of the third and fourth digits on the left hand.
Of the following, the MOST likely etiology of these birth defects is
A. amniotic bands
B. fetal hydantoin syndrome
C. fetal varicella effects
D. maternal hyperthermia
E. trisomy 13

91. A stillborn male is delivered at 33 weeks' gestation after a pregnancy


complicated by oligohydramnios. He is noted to have dysmorphic features,
including a blunt nose, recessed chin, micrognathia, large low-set ears, and
wrinkled facial skin. In addition, he has flexion contractures of the knees and
talipes equinovares.
Of the following, the additional abnormality that is MOST likely to be identified
by postmortem examination is
A. complex congenital heart defect
B. diaphragmatic hernia
C. hepatosplenomegaly
D. lissencephaly
E. renal agenesis

92. At birth, a male infant exhibits bilateral cleft lip and palate. Further
examination in the nursery reveals microphthalmia, scalp defects in the
parietooccipital area, cryptorchidism, and a cardiac murmur.
Of the following, the MOST likely underlying condition is
A. holoprosencephaly sequence
B. retinoic acid embryopathy
C. trisomy 13
D. trisomy 18
E. valproate embryopathy
93. A newborn boy delivered at term is found to have bilateral colobomas,
choanal atresia, and cryptorchidism.
Of the following, the most appropriate INITIAL test to exclude other associated
conditions is a(n)
A. Barium swallow
B. Echocardiogaphy
C. Flexible fiberoptic bronchoscopy
D. Renal ultrasonography
E. Skeletal survey

94. A 27-year-old woman gave birth to a daughter who had trisomy 18 and died
during the newborn period.
Of the following, the MOST correct statement concerning her risk of having
another infant who has any chromosome abnormality is that it is
A. about 5% at any maternal age
B. dependent on the chromosome constitution of the mother
C. increased for all aneuploidies
D. independent of the age of the mother at the time of delivery
E. the same as that of any other women of the same age

95. Boys who have Klinefelter syndrome usually appear physically normal at
birth.
Among the following, the constellation of findings that is MOST consistent with
the diagnosis of Klinefelter syndrome in boys during childhood is
A. growth retardation, clinodactyly, and hypospadias
B. mental retardation, large ears, and large testicles
C. scoliosis, astigmatism, and inguinal hernia
D. short stature, cardiac murmur, and undescended testicles
E. tall stature, learning disability, and small testicles

96. A 2-year-old boy is placed on oral antibiotic therapy to treat otitis media.
Three weeks later he becomes febrile (temperature, 102°F [38.9°C]) and
develops abdominal distension and tenderness and bloody diarrhea.
Of the following, the MOST likely etiology of his bloody diarrhea is an infection
caused by:
A. Campylobacter jejuni
B. Clostridium difficile
C. enteropathogenic Escherichia coli
D. Salmonella sp
E. Shigella sp

97. A 5-year-old child who has acute lymphoblastic leukemia and is receiving
chemotherapy was exposed to varicella 2 days ago. He has no history of prior
varicella infection or vaccination.
Of the following, the MOST appropriate management is to prescribe
A. acyclovir intravenously
B. acyclovir orally
C. immunoglobulin G intravenously
D. varicella vaccine
E. zoster immune globulin intramuscularly

98. The MOST common cause of acute laryngotracheobronchitis in children that


also causes many cases of bronchiolitis and pneumonia among young infants is
A. adenovirus
B. influenza virus
C. parainfluenza virus
D. respiratory syncytial virus
E. rhinovirus

99. During the health supervision visit of a 12-year-old female, you find that her
thyroid gland is firm, nontender, symmetrically enlarged, mobile, and has a
granular texture. The thyroxine level is 5.2 mcg/dL (normal, 5.6 to 11.7 mcg/dL)
and the thyroid-stimulating hormone level is 17.5 mIU/mL (normal, 0.2 to 7.6
mIU/mL).
Of the following, the MOST likely diagnosis is:
A. acute suppurative thyroiditis
B. Graves disease
C. Hashimoto thyroiditis
D. simple colloid goiter
E. solitary thyroid nodule

100. A 5-year-old girl presents with hypotension following a mild case of


gastroenteritis. Upon further questioning, you elicit a history of weakness,
malaise, anorexia, and weight loss over the past few weeks. On physical
examination, you note diffuse hyperpigmentation of the skin. Measurement of
serum chemistries reveals hyponatremia, hyperkalemia, and hypoglycemia. You
suspect Addison disease.
Of the following, the MOST common etiology of Addison disease in children is
A. adrenal hemorrhage
B. amyloidosis
C. autoimmune destruction of the adrenal cortex
D. histoplasmosis
E. tuberculosis

Which of the following statements applies to infants with gastroenteritis:


1 ) In most instances require treatment with an antibiotic
2 ) May have blood in their stools
3 ) Should be admitted to hospital if they are unable to tolerate fluid orally
4 ) Always develop lactose intolerance

Which of the following statements regarding kernicterus is incorrect?


1 ) Usually occurs in the first month of life
2 ) The diagnosis of kernicterus requires the histological confirmation of yellow
staining of brain tissue
3 ) May cause a chronic syndrome of athetosis, gaze disturbance and
hearing loss
4 ) Is associated with Gastrointestinal haemorrhage

Which one of the following statements is true concerning subjects with


untreated celiac disease?
1 ) They are likely to have normal jejunal villi
2 ) Usually absorb glucose normally
3 ) Usually have anorexia
4 ) May safely resume a normal diet in adult life

Which of the following statments is characteristic of acute hepatitis B infection?


1 ) Most patients present with splenomegaly.
2 ) It confers immunity to hepatitis A.
3 ) It commonly presents with distal joint arthritis.
4 ) There is increased infectivity in the presence of the e antigen.

Which of the following is true of Gilbert's syndrome?


1 ) inheritance is autosomal recessive
2 ) serum conjugated bilirubin levels are elevated
3 ) serum bilirubin levels are decreased by fasting
4 ) serum bilirubin levels are decreased by liver enzyme
Inducers phenoparbeton

Which of the following statements correctly applies to a baby of 2 months who


is considered to be 10% clinically dehydrated with gastroenteritis:
1 ) He may have blood and mucus in his stools
2 ) He will have a full fontanelle
3 ) He will have a low packed cell volume
4 ) Urea level will be normal

Which of the following conditions may give a false/positive sweat test?


1 ) Non-classic Congenital adrenal hyperplasia
2 ) Hyperthyroidism
3 ) Hyperparathyroidism
4 ) Glucose-6-phosphatase deficiency

A 17 year old male student attends clinic complaining of right knee pain and left
ankle pain. These pains have deteriorated when associated with episodic
diarrhoea. What is the most likely diagnosis?
1 ) Amoebic dysentery
2 ) Coeliac disease
3 ) Chronic appendicitis
4 ) Ulcerative colitis

A 6 week old infant presents with vomiting and failure to thrive. A


diagnosis of pyloric stenosis is considered. Which one of the following
statements is correct regarding congenital hypertrophic pyloric stenosis?
1 ) A “double bubble” is likely to be seen on abdominal X-ray
2 ) A hyperchloraemic alkalosis would be expected
3 ) Refusal of feeds is a feature
5 ) Vomiting would have typically started between 2 to 4 weeks

An infant is admitted with diarrhoea and a diagnosis of Rotavirus is suspected.


Which one of the following is correct regarding Rotavirus
infection?
1 ) It typically affects infants older than 18 months
2 ) The Rotazyme test to detect virus particles is a direct enzyme-linked
immunosorbent assay
3 ) Blood and mucus is found in the stools of about 50%
4 ) It occurs most often in the summer months
A young infant presents with a suspected diagnosis of pyloric stenosis. Which of
the following is a feature of this diagnosis?

1 ) Projectile vomiting
2 ) Anorexia
3 ) Loose stools
4 ) More frequent in girls

Which one of the following is a typical feature of toddler’s diarrhoea.


1 ) Abdominal cramps
2 ) Failure to thrive
4 ) Poor growth [0]
5 ) Undigested food in the stools

Which one of the following is a reliable method of assessing dehydration in


infants: Useful indices of dehydration in infants include:
2 ) Skin turgor over the dorsum of the hand
3 ) Palpation of the anterior fontanelle
4 ) Blood pressure
5 ) Serum sodium concentration

Which one of the following suggest a diagnosis of Hirschsprung's disease?


1 ) A contrast-study showing dilatation of
the aganglionic bowel segment.
2 ) Early presentation with vomiting.
3 ) neonatal large bowel obstruction.
4 ) Presentation after 1 year of age.

An infant boy is brought to clinic with a short history of vomiting and


diarrhoea. Which of the following is most applicable concerning this infant?
1 ) Do not show signs of dehydration until more than 5% of the body
weight is depleted
2 ) Bloody diarrhoea would be expected with Staph Aureus food poisoning
3 ) If dehydrated, he is likely to be hyponatraemic
5 ) Stool culture will typically reveal pathogenic E coli
Which of the following statements is true concerning: Intussesception in
children:
1 ) Is more common between the age of 3 months to 8 years
2 ) Bile stain vomiting is an early feature
3 ) Bleeding per rectum is the most common presenting symptom
5 ) Hydrostatic reduction plays an important role in the management

Which one of the following statements is true regarding giardia lamblia.


1 ) May cause intestinal malabsorption
2 ) Often causes a bloody diarrhoea
3 ) May cause of haemolytic uraemic syndrome
4 ) May be excluded by stool microscopy

An infant is suspected of having gastrooesophageal reflux.


Which one of the following statements relating to this diagnosis is correct?
1 ) He is unlikely to present with vomiting.
2 ) Vomitus will be bile stained.
4 ) He may have apnoeic episodes
5 ) A barium swallow would be the investigation of choice to diagnose it

A male child weighing 4.2kg is admitted at age 4 weeks with a one week history
of forceful vomiting after feeds. He is alert and eager for food but exhibits a loss
of skin elasticity and has apparent distension in
the left hypochondrium.
Which of the following statements is correct for this patient?
1 ) The most likely diagnosis is hiatus hernia
2 ) He has hyperosmolar dehydration
3 ) Metabolic acidosis would be anticipated.
4 ) Initial fluid replacement should comprise normal saline 10-20 ml/kg

A 2 year old child is admitted with a 2 day history of diarrhoea.


Which of the following statements correctly applies to his condition?
1 ) Hyponatraemia is expected.
2 ) Adenovirus is the commonest pathogen isolated.
3 ) He should be treated with anti-diarrhoeal agents.
4 ) Dehydration is best assessed clinically rather than biochemical

The following laboratory results were returned in a 6 week old boy admitted
with 6 days of severe projectile vomiting:
Ph 7.51
PO2 (95 mmHg)
PCO2 (35 mmHg)
Blood Urea 11 mmol/l
Na+ 131 mmol/l
K+ 3 mmol/l
Chloride 83 mmol/l
Which of the following is true concerning this patient?
1 ) He has respiratory alkalosis
2 ) He is likely to have a bulging anterior fontanelle
3 ) He should be resuscitated immediately with normal saline
4 ) He should be commenced immediately on half strength soy protein, low
lactose formula

1-The Virus most commonly associated with the clinical picture of acute
bronchiolitis in infants and young children is ?
(A) adenovirus
(B) respiratory syncytial virus
(C) Rhinovirus
(D)coxsackievirus B
(E) herpes virus type 2

2- It recently has been recommended that young infants should sleep in the
supine rather than in the prone position. This is based on data suggesting that
the prone position is associated with an increased in cadence of :
(A) delayed eruption of the first deciduous teeth
(B) gastroesophageal reflux and aspiration
(C) macrognathia
(D) strabismus
(E) sudden infant death

3- Early in an attack of asthma, which of the following sets of arterial blood


values is most likely?
(A) Normal PO2 and normal PCO2
(B) Normal PO2 and low PCO2
(C) Normal PO2 and elevated PCO2
(D) Low PO2 and low PCO2
(E) Low PO2 and elevated PCO2

4- The most common roentgenographic abnormality in a child with asthma is


(A) bronchiectasis
(B) generalized hyperinflation
(C) lower lobe infiltrates
(D) pneumomediastinum
(E) right middle lobe atelectasis

5- the most commonly encountered bacterial pathogens in the lungs of patients


with cystic fibrosis are:
(A) E.coli and alpha Streptococcus
(B) E. coli and pseudomonas
(C) Staphylococcus and proteus
(D) Staphylococcus and pseudomonas
(E) Hemophilus influenzae and Pneumococcus

6- Serous otitis media is defined as the accumulation of fluid , usually sterile, in


the middle ear. The chief symptom is:
(A) fever
(B) hearing loss
(C) pain
(D) tinnitus
(E) Vertigo
7- A 12-year-old child presents with fever, purulent nasal discharge, and
swelling and tenderness over the erea below the left eye. The remainder of the
physical examination is within normal limits. The most likely diagnosis is:
(A) acute sinusitis
(B) periorbital cellulitis
(C) purulent cellulitis
(C) retropharyngeal abscess
(E) rhabdomyosarcoma

8- Chronic upper airway obstruction from enlarged tonsils and adenoids in a


child may cause:
(A) convulsions
(B) cor pulmonale
(C) a pneumothorax
(D) thymic hyperplasia
(E) asthma (reactive airway disease)

9- A 2-year-old infant has a chronic cough, clubbing, and rectal prolapse, the
most likely diagnosis to explain all of these findings would be :
(A) agmmaglobulinemia
(B) cirrhosis of the liver
(C) cystic fibrosis
(D) granulomatous colitis
(E) pinworms

10- A 2-year-old drinks some kerosene that had been left in a glass. After the
first swallow she cries and drops the glass . she is most likely to develop :
(A) aplastic anemia
(B) chemical pneumonitis
(C) coma and/or convulsions
(D) hepatitis
(E) peripheral neuritis
11- A 4-year-old child, previously well, presents with the rather sudden onset of
wheezing that does not respond to treatment with aerosolized albuterol. The
most likely diagnosis is :
(A) asthma
(B) foreign body
(C) infantile lobar emphysema
(D) pulmonary hypoplasia
(E) right middle lobe syndrome

12- The diagnosis of cystic fibrosis is usually confirmed by the finding of :


(A) elevated sweat chloride
(B) decreased sweat chloride
(C) elevated serum chloride
(D) decreased serum chloride
(E) elevated sweat and serum chloride

13- Which of the following statements regarding acute bacterial otitis media in
childhood is correct ?
(A) fever is common, but pain is rare
(B) it is a common cause of deafiness
(C) infection is almost invariably bilateral
(D) the most frequent organism is staphylococcus aureus
(E) it usually can be treated successfully with antibiotics without
tympanocentesis.

14- Which of the following would be expected in a 6-month-old child with a


large ventricular septal defect ?
(A) cyanosis
(B) an enlarged heart
(C) a continuous cardiac murmur
(D) decreased pulmonary vasculature on roentgeno-gram
(E) evidence of predominantly right ventricular hypertrophy on ECG

15- Rectal prolapse is a possible complication of all the following conditions,


Except:
(A) cystic fibrosis
(B) severe malnutrition
(C) whooping cough
(D) chronic dysentery
(E) Enterobiasis

16-All of the following statements concerning the incidence of the sudden infant
death syndrome (SIDS) are correct, except:
(A) the incidence of the sudden infant death syndrome is the highest at the age
of 5-6months
(B) it is more frequent in families with poor social conditions
(C) the incidence is higher among the subsequent siblings of SIDS victims
(D) the incidence is higher among girls.

17- Which of the following is a typical symptom of Henoch-Schonlein purpura?


(A) polyarthralgia
(B) jaundice
(C) a purpuric skin rush involving the face
(D) a decreased serum complement level
(E) a prolonged bleeding time
18- A possible cause of frequent pulmonary infections in a neonate is:
(A) cystic fibrosis
(B) hypogammaglobulinemia
(C) chronic granulomatous disease
(D) all of the above

19- Which childhood age is the onset of acute lymphoblastic leukemia most
frequent?
(A) 10-12 years
(B) The neonatal period
(C) Adolescence
(D) 3-5 years
(E) 6 months

20- Bleeding, due to thrombocytopenia occurs if the platelet count is less than:
(A) 150.000/ul
(B) 100.000/ul
(C) 80.000/ul
(D) 50.000/ul
(E) 20.000/ul

21- Meconium ileus is a possible early symptom of :


(A) Pyloric stenosis
(B) Hirschsprung's disease
(C) cystic fibrosis
(D) intestinal perforation

22- A 6- hour-old newborn exhibits tachypnea and dyspnea and vomits


repeatedly the body temperature of the newborn is normal . the physical
examination reveals a tympanic resonance over the left side of the chest no
respiratory sounds are audible over this side . the surface of the abdomen is
concave. What is the most likely cause of this condition?
(A) IRDS
(B) neonatal sepsis
(C) left-sided pneumonia
(D) a diaphragmatic hernia on the left side
(E) pneumothorax
(F) congenital valvular disease

23 Which of the following congenital valvular heart diseases is associated with


severe cyanosis manifested during the first few days of life ?
(A) a orto-pulmonary fenestration .
(B) the postductal form of coarctation of the aorta .
(C) common atrioventricular ostium .
(D) transposition of the great arteries .
(E) patent ductus arteriosus .

24- Which is the most common valvular abnormality developing as a


consequence of rheumatic fever?
(A) mitral insufficiency
(B) mitral stenosis
(C) aortic insufficiency
(D) aortic stenosis
(E) tricuspid insufficiency

25- Symptoms of a digitalis overdose include all of the following , EXXEPT:


(A) nausea, vomiting
(B) bradycardia , arrhythmia
(C) a prolongation of the PR interval , 2nd or 3rd degree AV-block observed on
the ECG
(D) atrial premature complexes

27- Possible causes of hypertension in childhood include all of the following ,


EXCEPT:
(A) hypothyroidism
(B) pheochromocytoma
(C) coarctation of the aorta
(D) acute glomerulonephritis
(E) encephalitis

28- Possible complications of tonsillitis include all of the following, EXCEPT:


(A) cervical lymphadenitis
(B) acute rheumatic fever
(C) sepsis
(D) chronic rheumatoid arthritis
(E) acute diffuse glomerulonephritis

30- Which of the following therapeutic interventions is prohibited in an


asthmatic crisiss?
(A) an increased intake of fluids
(B) beta receptor agonists
(C) beta receptor blockers
(D) theophylling
(E) corticosteroids

31- All of the following physical signs are characteristic of lobar pneumonia ,
EXCEPT:
(A) a duller percussion sound is heard over the affected lobe
(B) bronchophony is detected over the affected lobe
(C) bronchial respiratory sound are heard over the affected lobe
(D) a tympanic resonance is detected over the affected lobe
(E) crepitation is audible over the affected lobe

32- All of the following microorganisms are associated with lung abscess
formation,
except:
(A) streptococcus pneumoniae
(B) klebsiella pneumoniae
(C) Chlamydia trachomatis
(D) staphylococcus aureus
(E) Haemophilus influenzae

33- All of the following statements about pneumonia caused by pneumocystis


carinii are correct, except:
(A) the incubation period is 3-8 week's
(B) premature newborns and immunodeficient patients are a higher risk
(C) the clinical course is acute (several hours)
(D) the presenting symptoms are marked tachypnea, cyanosis, pallor
(E) a frosted glass-like shadow is observed on the x-ray
F) the therapeutic drug of choice is trimethoprim/sulfamethoxazole

34- Which of the following procedures is reliable in the diagnosis or exclusion of


a foreign body in the airways ?
(A) a physical examination
(B) a thorough history taking
(C) bronchoscopy
(D) chest transillumination (Holzknecht's sign)
(E) chest X-ray

35- All of the following diseases are associated with a decreased production of
red blood cells except:
(A) iron deficiency anemia
(B) leukemia
(C) hypothyroidism
(D) the early type anemia of a premature newborn
(E) renal failure
36- Which of the following is the most important therapeutic intervention in
familial spherocytosis of a 6-year- old child, who frequently becomes anemic?
(A) a transfusion of packed red blood cells
(B) a splenectomy
(C) the prolonged administration of steroids
(D) immunosuppressive therapy
(E) iron replacement therapy

37- Symptoms and signs of hemolytic uremic syndrome (HUS) include all of the
following:
(A) fragmentocytes in the blood smear
(B) thrombocytopenic hemorrhagic diathesis
(C) oliguria , edema
(D) hypotension
(E) hyperkalemia
(F) acidosis
38 -You are examining an 18-month-old boy. The parents have noted the
occurrence of large hematomas on the skin following mild traumas during the
last 6 months. The results of the laboratory tests are not yet ready , when
theparents inform you that the child's finger started to bleed again , two
hoursafter the puncture . what is the most likely diagnosis ?
(A) thrombocytopenia
(B) von willebrand's disease
(C) hemophilia
(D) vasculopathy
(E) thrombocytopathy

39- An 8-year-old , febrile child is admitted to the pediatric department large


lymph nodes are palpable on both sides of the neck. The examination reveals
pharyngitis and hepatosplenomegaly .No symptoms of anemia or bleeding are
observed .Atypical mononuclear cells are seen in the peripheral blood smear .
what is the most likely diagnosis?
(A) Leukemia
(B) Toxoplasmosis
(C) Lymphoma
(D) infectious mononucleosis
(E) cytomegalovirus infection
40- A 3.5kg full-term infant is born after an uncomplicated delivery with apgar
scores of 9 and 9at 1 and 5minutes. The infant cries vigorously after birth but
then goes into aquiet state. Within 10minutes, the infant experiences cyanosis
and respiratory arrest . during resuscitation the nurse is unable to pass a
nasogastic tube. The most likely diagnosis is :
(A) tracheoesophageal fistula
(B) pneumothorax
(C) persistent fetal circulation
(D) choanal atresia
(E) laryngotracheomalacia
41- A 1-year old presents with an acute onset of cough , choking, and
respiratory distress physical evaluation reveals respiratory rate of 45 and
wheezing there is no family history of asthma and no one at home is ill. The
older sister states that they were both playing house and that they both had
eaten sunflower seeds. The most likely diagnosis is:
(A) anaphylaxis
(B) bronchiolitis
(C) cystic fibrosis
(D) foreign body aspiration
(E) angioedema

43- Which of the following is the most likely diagnosis in an otherwise normal
adolescent with the sudden onset of respiratory distress, cyanosis , retractions,
and markedly decreased breath sounds over his left lung ?
(A) Empyema
(B) Chylothorax
(C) Pneumothorax
(D) staphylococcal pneumonia
(E) aspiration of a foreign body

45- The best way to identify the anatomy of a child with choanal atresia is :
(A) Ultrasonography
(B) Rhinoscopy
(C) Rhinogram
(D) MRI
(E) CT scan
46- A 3- year-old presents to your office with a 3-week history of unilateral
nasal discharge. The discharge is malodorous and lately has been blood tinged.
The most likely diagnosis is:
(A) Tertiary syphilis
(B) unilateral choanal atresia
(C) chronic rhinovirus infection
(D) foreign body
(E) wegemer granulomatosis

47- A 12-year-old male presents with recurrent left-sided unilateral epistaxis


that has been present for 6 months. possible causes include all of the following
EXCEPT:
(A) Juvenile nasopharyngeal angiofibroma
(B) von willebrand disease
(C) idiopathic thrombocytopenia purpura
(D) hereditary hemorrhagic telangiectasia
(E) Hench-Schonlein purpura

48- The most common cause of a nasal polyp in children is :


(A) aspirin allergy
(B) Juvenile nasopharyngeal angiofibroma
(C) peutz-Jeghers syndrome
(D) renal failure
(E) cystic fibrosis

49- A 1-month-old male has had noisy breathing since birth . he is worse when
supine . the physical examination reveals retractions, mild thoracic deformity,
and inspiratory stridor. The most likely diagnosis is:
(A) diaphragmatic hernia
(B) tracheoesophageal fistula
(C) laryngomalacia
(D) choanal atresia
(E) vocal cord paralysis

50- Indicators for serious respiratory tract disease in children with chronic
pulmonary symptoms include all of the following except;
(A) clubbing
(B) cough that disappears with sleep
(C) cyanosis
(D) failure to thrive
(E) chronic purulent sputum
51- The most common manifestation of preliminary involvement in children
with cystic fibrosis is :
(A) cyanosis
(B) clubbing
(C) cough
(D) wheezing
(E) nasal polyps

52- A 3-year old has had recurrent episodes of cough, pneumonia , and sinusitis
repeated sweat chloride levels are normal, and the F 508 mutation is not
present. In the last year, the child has had repeated episodes of otitis media .
the chest x-ray reveals dextrocardia. The next diagnostic test should be :
(A) quantitative immunoglobulins
(B) IgG subclass determination
(C) CBC
(D) nasal scraping for electron microscopy
(E) HIV serology

53- A 3-day-old presents with fussiness and poor feeding, on examination , the
heart rate in noted to be 250. the ECG reveal a rate of 250, a QRS of 0.07
seconds ,, and no visible P waves. the most likely diagnosis is :
(A) ventricular tachycardia
(B) supraventricular tachycardia with aberrant conduction
(C) supraventricular tachycardia
(D) heart block
(E) none of the above

54 The first approach to the therapy of the dysrhythmia described in Question


53 is :
(A) fluid challenge
(B) iced saline bag placed over the face
(C) carotid massage
(D) propranolol
(E) verapamil

55- A previously well 3½ -month- old presents with poor feeding, diaphoresis
during feeding, and poor growth. Vital signs reveal respirations of 70, pulse of
175, and blood pressure of 90/65mmHg in the upper and lower extremities . the
cardia examination reveals apalpable parasternal lift and a systolic thrill. A grade
4 holosystolic murmur and a mid diastolic rumble are noted. The chest
radiograph reveals cardiomegaly. The most likely diagnosis is :
(A) cardiomyopathy
(B) myocarditis
(C) VSD
(D) coarctation of the aorta
(E) transposition of the great arteries

56- A 6-month-old presents with tachycardia, tachypnea, and poor feeding for 3
months. Physical examination reveals a continuous machinery murmur and a
wide pulse pressure with a prominent apical impulse . the most likely diagnosis
is :
(A) pulmonic stenosis
(B) aortic stenosis
(C) ventricular septal defect
(D) patent ductus arteriosus
(E) anomalous coronary artery

59 A 4-day- old previously healthy term infant presents in heart failure. There is
a gallop, a grade 3/6 systole murmur, hepatomegaly, and cardiomegaly on chest
x-ray. The blood pressure is 95/70. all of the following will be helpful in
evaluating the patient EXCEPT:
(A) arterial blood gas
(B) blood pressure in the upper and lower extremities
(C) complete blood count
(D) determination of splenomegaly
(E) chest x-ray

60- A 2-month-old male presents with tachycardia, dyspnea, tachypnea, and a


gallop rhythm with no heart murmur He was perfectly well until 1 day prior to
the episode. The physical examination reveal a heart rate of 235, a temperature
of 37.8ºC, and a normal blood pressure with warm, well-perfused extremities.
The most likely diagnosis is:
(A) Sepsis
(B) supraventricular tachycardia
(C) ingestion
(D) ventricular tachycardia
(E) sinus rhythm

61- A 5-year- old white female has multiple bruises on her lower extremities
and oral mucosal bleeding of 3 day's duration. Two weeks before these signs,
she had a mild respiratory tract infection. Physical examination reveals multiple
ecchymoses and petechiae, no lymphadenopathy or hepatosplenomegaly is
noted. The next diagnostic step is:
(A) a complete blood count
(B) a prothrombin time
(C) a bleeding time
(D) a partial thromboplastin time
(E) an antinuclear antibody titer

62- A 10-month-old white male presents with a 1-day history of persistent


bleeding after cutting his lip slightly. The family history is unremarkable, and the
patient is receiving no medication. Laboratory date reveal a hemoglobin value
of 11g/dl, platelets of 350.000, a prothrombin time, of 11.8seconds, and a
partial thromboplastin time(PTT)of 100 seconds, which is corrected by mixing of
normal plasma. The most likely diagnosis is :
(A) von Willebrand disease
(B) hemophilia A
(C) Hageman factor deficiency
(D) scurvy
(E) anti-cardiolipin antibody syndrome

63- A 5-day-old full term male presents with intense cyanosis, tachypnea, and
tachycardia. Physical examination reveals cyanosis of the skin and mucous
membranes, the lungs, heart, pulse, and general examination show no
abnormalities. Arterial blood gas determination reveals a PaO2 of 95 while the
patient is breathing room air and an oxygen saturation of 40%. The most likely
diagnosis is:
(A) transposition of the arteries
(B) pulmonary atresia
(C) pulmonary hypoplasia
(D) methemogloninemia
(E) Heinz body anemia

66- Chronic thrombocytopenia in a male with eczema should suggest :


(A) infection
(B) malignancy
(C) Evans syndrome
(D) Wiskott-Aldrich syndrome
(E) JRA

Pediatrics MCQ’s by Anmar Razak


1. You are called to do a routine check up on a 7 day old Rh –ve term neonate
with a weight of 3.2 Kg who was kept in the hospital because of low apgar
score at birth. Which of the following signs requires further thorough
investigation:
A. Jaundice
B. Absence of Parachuting reflex
C. Absence of Grasp reflex
D. Presence of head lag
E. Positive babinski sign
F. 300 grams weight loss

2. At 5 minutes after birth, a male neonate was noted to have blue extremities
with a pulse of 90 and a shallow, irregular breathing pattern. He was also
noted to be moving all 4 limbs. He showed no response when his foot was
slapped. What would be the appropriate Apgar score for this baby:
A. 3
B. 4
C. 5
D. 6
E. 7
F. 8
3. The most important parameters in the Apgar scoring system is(are):
A. Color
B. Reflex irritability
C. Muscle tone
D. Heart rate and respiration
E. All are equally important
4. All of the following are contraindications to breat feeding EXCEPT:
A. HIV +ve mother
B. Mother with active untreated TB
C. Mother on Chemotherapy
D. Mother taking Tetracycline
E. Mother with Hepatitis B infection
F. Active Herpes in the breast region
5. Presence of which of the following is considered abnormal in a neonate:
A. Erythema Toxicum
B. Epstein’s pearls
C. Mongolian spots
D. Capillary hemangioma (stork Bites)
E. Subconjunctival hemorrhage
F. Nevus flammeus (port wine stain)
6. On a routine exam of a 1 day old newborn, a non tender swelling of the scalp
was noted. It does not cross the suture lines. What is the most likely cause:
A. Caput succedeneum
B. Subperiosteal bleeding
C. Subdural bleeding
D. Epidural bleeding
E. Subcutaneous bleeding

7. A baby is delivered following a prolonged and difficult labour due to shoulder


dysticia. On physical examination of the new born 24 hours later, it is noted
that the Moro reflex is absent on the right. What is the most likely diagnosis:
Erb Duchene paralysis
Fracture of clavicle
Klumpke’s paralysis
Transient brachial plexus paralysis
Cerebral palsy
8. What is the most appropriate treatment approach in this baby:
A. Send for X-ray
B. Immobilize in a figure of 8 sling
C. Observe for spontaneous resolution
D. CT scan of head
E. Surgical intervention
9. A 1 week old female infant is being examined as part of a routine general
checkup. She is a term baby and was delivered vaginally without
complications. On examination, which of the following findings requires
further diagnostic or therapeutic intervention:
A. Unilateral breast enlargement with milk discharge
B. White vaginal discharge with a small amount of blood
C. Talipes of the left foot which can be fully dorsiflexed
D. Absent femoral pulses
E. Umbilical hernia
10. On a routine physical examination, a 1 day old male infant is found to have
a palpable abdominal mass. What is the most likely cause of this mass:
A. Polycystic kidneys
B. Posterior urethral valve
C. Hepatomegally
D. Wilms tumour
E. Neuroblastoma

11. A 4 months old male infant was brought to the clinic by his mother. She
has noticed a swelling of his left scrotum. On examintion, a non tender
swelling of the left scrotum was noted. Translumination is positive. The
swelling does not resolve on compression. He was a term infant, had an
uncomplicated delivery and had a normal routine newborn examination
before discharge from the hospital. What is the most appropriate approach in
this baby boy:
A. Surgical correction now
B. Observe until 1 year of age, and repair surgically if no resolution
C. Order an U/S
D. Aspirate fluid for analysis
E. Order an abdominal CT scan

12. A male newborn was delivered vaginally at 38 weeks to a mother who


developed gestational diabetes that was poorly controlled during pregnancy.
All of the following are possible complications to occur in this baby EXCEPT:
A. Hypoglycemia
B. Hyperbillirubinemia
C. Polycythemia
D. Respiratory distress syndrome
E. Necrotizing enterocollitis

13. A 4 day old female newborn is brought to the ER with a a 1 day history of
vomiting, fever and diarrhea in adition to being irritable with a hight pitched
cry. Her mother is known to be an IV drug abuser. The baby was delivered at
term by C-section due to inadequate maternal contractions. If the baby’s
symptoms, which of the following drug effects is known to cause such
presentation:
A. Cocaine Intoxication
B. Heroin intoxication
C. Methadone withdrawal
D. Heroin withdrawal
E. Cocain withdrawal

A 12 hour old male neonate is noticed to have rapid shallow breathing with
nasal flaring. The baby was delivered at 41 weeks gestation by C- section due to
difficult labour which was later complicated by meconuim staining of amniotic
fluid. He was suctioned properly after birth an APGAR scores were reassuring.
He has no cyanosis, and Blood pressure is normal. Temp. is normal. Xray shows
fluid in the fissures with increased vascular markings. The most appropriate
approach in this patient is:
A. Intubate with artificial ventilation
B. Observe and supplemental Oxygen if needed
C. Administer surfactants to baby, with ventilatory support
D. Order an Echocardiogram
E. Insert chest tube with an underwater seal

Soon after birth, a female infant developes tachypnea with nasal flaring and
grunting. Physical Examination reveals hyper-resonance on the right, with
absent air entry on the left and a displaced apex beat and heart sounds to the
right. No murmurs are heard and peripheral pulses are normal. The most likely
diagnosis in this baby is:
A. Pneumothorax
B. Pneumonia
C. Meconuim aspiration
D. Diaphragmatic hernia
E. Situs invertus

All of the following maybe indicated in the above patient EXCEPT:


A. Nasogastric suctioning
B. Surgical intervention
C. Chest tube insertion
D. Bag and mask ventilation
E. Intubation
A 2 week old breast fed infant was brought to the clinic with Jaundice. He has
normal coloured stool and urine. The baby was delivered vaginally at term. The
most common cause of his jaundice is:
A. Breast milk jaundice
B. Rh incompatibility
C. Billiary atresia
D. ABO incompatibility
E. Infection

A 3 day old newoborn is noted to have poor feeding, irritability and vomiting.
His pulse was 170, RR is 70, temp is normal. Tense fontanelles were noted.
What is the most appropriate approach for this baby:
A. Ampicillin and Gentamicin therapy
B. Ceftriaxone therapy
C. Surgical evacuation of a hematoma
D. Steroid therapy
E. Reassurance and monitor feeding pattern.

the most common congenital transmitted infection is :


A. Toxoplasmosis
B. Rubella
C. CMV
D. Syphilis
E. Herpes

A 12 hour old male newborn is noted to have difficulty feeding. He develops


respiratory distress with choking and becomes cyanotic soon after he starts
feeding. He is normal when not feeding and during sleep. A nurse’s effort to
insert a nasogastric tube was unsuccessful. Which of the following statements is
correct:
A. His pregnancy was complicated by Polyhydramnois
B. His pregnancy was complicated by Oligohydramnois
C. This baby probably has choanal atresia
D. Xray of this baby will show bowel loops in the lung fields
A cleft palate will be found on oral examination
A 1 day old male infant of a diabetic mother is seen before discharge for a
complaint of vomiting. The baby was normal at birth and was delivered vaginally
at term. He has not yet passed any meconuim. The mother states that soon
after she finishes breast feeding the baby, he starts vomiting green coloured
fluid. What is the most accurate statement:
A. Examination will reveal and olive shaped mass in the epigastrium
B. Rectal examination will reveal an empty rectal vault with rectal biopsy
showing absence of ganglion cells
C. A nasogastric tube will be seen coiled in the esophagus on Xray
D. Abdominal Xray will show a double bubble
E. Diabetes is a major predisposing factor for this condition

A 10 day old female infant was brought to the emergency department after the
mother noticed that over the past 24 hours the baby started crying intensely
with poor feeding. She also noticed that the baby was lethargic and says that
she thinks the baby’s abdomen was becoming a little distended. She finally
brought her to the ER when she noticed some blood in her stool. The baby was
delivered at 32 weeks gestation vaginally when the mother went into preterm
labour. What is the most accurate statement regarding this condition:
A. Maternal diabetes will probably be revealed on further questioning of the
mother
B. Abdominal Xray may show pneumatosis interstinalis
C. The source of bleed is most probably ectopic gastric mucosa in meckels
diverticulum
D. Surgical intervention is urgently indicated

Hirschprung disease should be highly suspected.


A 2 day old premature male newborn was noted to have intermittent absence
of respiratory effort of 15 seconds duration occurring every 2 minutes. He is
acyanotic, with a pulse rate of 140 and a RR of 45. The baby was delivered by c-
section at 32 weeks gestation due to placental bleeding. What is the most
appropriate management approach in this baby:
A. Reassurance
B. Full septic workup
C. Intubate and ventilate for respiratory support
D. Measure blood electrolytes and glucose, possible EEG
E. Cranial Ultrasound

When assessing the need of nutritional supplements for a preterm infant, which
of the following is INCORRECT:
A. Vitamen K should be given at birth
B. Iron supplement should be started earlier at 6-8 weeks of age rather than
6 months
C. Fluoride supplement is recommended at 6 months of age if inadequate in
water supplies
D. Vitamen D is indicated if low birth weight
E. Folate supplement should not be given if goat’s milk is used for feeding.

In protein calorie malnutrition, all of the following is true EXCEPT:


A. Marasmus usually presents with less severe weight deficit than kwashiorkor
B. Edema is a key feature of kwashiorkor
C. Dermatitis, stomatitis, depigmented hair are seen in kwashiorkor but not in
marasmus
D. Marasmus is typically seen in polygamous societies in which children are not
weaned from breast feeding until 12 months of age

Kwashiorkor can be precipitated after an acute intercurrent infection


A father brings his 12 year old daughter to your clinic. He thinks she is too short.
Measurements show that she is below the 3rd percentile for her height and age.
Her father states that she has always looked short to him when compared with
her siblings. Both her parents are of normal height. Physical examination is
normal. The single most important parameter to use in evaluating this patient
is:
A. Growth chart
B. Karyotype
C. Lateral skull X-ray
D. Wrist X-ray
E. Growth Hormone measurment
F. Thyroid hormone measurment

A 2 months old male infant is brought to the ER. His mother states that over the
past 2 days he developed severe diarrhea and was feeding poorly. On
Examination, the child was irritable with sunken fontanelles, had dry mucosa
and was crying but no tears were seen. Blood pressure is low. Mother said that
his last urine output was very small and concentrated about 8 hour ago. He
weighs 5000 grams. The most appropriate next step in this patients
management is:
A. Draw blood for complete blood count, electrolytes and glucose
B. Infuse normal saline at a rate of 20 ml/kg
C. Start IV 0.33 Normal saline in 5% dextrose at a rate of 500 mls/day
D. Rehydrate using 0.45 normal saline in 5% dextrose IV over 48 hours
E. Rehydrate using 0.45 normal saline in 5% dextrose orally over 48 hours

Concerning developmental milestones, all of the following require further


investigations EXCEPT:
A. No social smile by 8 weeks of age
B. No good eye contact by 3 months of age
C. Not reaching for objects by 5 months of age
D. Not sitting unsupported by 9 months of age
E. Not walking unaided by 12 months

Note: the following is a list of important ABNORMAL milestone abnormalities:


No social smile by 8 weeks of age
No good eye contact by 3 months of age
Not reaching for objects by 5 months of age
Not sitting unsupported by 9 months of age
Not walking unaided by 18 months
Not saying single words with meaning at 18 months of age
No two or three word sentences by 30 months

A 6 year old boy was brought to your clinic by his family who have become
concerned over the child wetting his bed at night. The family states that the
child has been dry for about two years but has recently started wetting his bed,
especially at night, although daytime wetting is also noted. The family had just
relocated to this town 2 months ago. The child has no history of mental or
neurological disorders and has had a healthy childhood so far. The father says
that his son is about to join a summer camp and wants a relief for his child
before he goes away. Physical examination is normal. The most appropriate
next step in this child’s management is:
A. Reassurance
B. Urinalysis
C. Urine culture
D. Prescribe imipramine
E. Prescribe Oxybutynin

A 3 year old male child was brought to the ER 6 hours after he ingested 30
tablets of ASA pills at home. He was lethargic, tachypneic and tachycardic. He
had a seizure on his way to the ER. Arterial blood gasses on this child will most
probably reveal:
A. Metabolic acidosis with respiratory compensation
B. Respiratory Alkalosis
C. Respiratory Acidosis with renal compensation
D. Metabolic Alkalosis with respiratory compensation
E. Normal PH

Therapeutic measures for the above patient may include each of the following
EXCEPT:
A. Activated charcoal
B. Ipecac
C. Gastric lavage
D. Alkaline Diuresis
E. Vitamen K, glucose, and Fluid support
F. Dialysis
Lead poisoning can present with all of the following EXCEPT:
A. Microcytic Anemia
B. Haemolytic anemia
C. Wrist drop
D. Encephalopathy
E. Decreased erythrocyte protoporphyrins

A mother brings her 6 month old infant to your clinic for his routine
immunization update. He is to have his third DPTP and Hib vaccine. The mother
says that after his second immunization dose, the child developed redness and
swelling at the injection site with a fever of 38C for 2 days. Acetaminophen
given to her at the time helped relieve the complications. 4 days ago he was
diagnosed with acute otitis media and is currently on a 10 day course of
Amoxicillin. What is the best approach in this child regarding his immunization:
A. Defer immunization until after the child finishes his antibiotic course
B. Adminster both vaccines only after complete resolution of Otitis media
C. Give both vaccines now and continue antibiotic therapy
D. Substitute DT for DPTP in view of the child’s previous reaction of fever,
swelling and redness and administer with Hib now
E. Give both vaccines now but stop antibiotic therapy

Which of the following vaccines is contraindicated in an AIDS patient:


A. DPTP
B. BCG
C. HiB
D. Influenza
E. Hepatitis B vaccine
Which of the following vaccines should not be given to a 4 month old infant:
A. Influenza
B. Varicella
C. Oral polio
D. BCG
E. Cholera
A 3 year old male infant is brought to your clinic following a 2 week history of
persistant violent coughing paroxysms that are sometimes followed by
vomitting. The mother states that 3 weeks ago the child developed sysmptoms
of a common cold with rhinorrhea, conjunctival injection and mild cough which
has resolved except for the cough. The child on examination was noted to have
facial petichiae and conjunctival hemorrhages. Lung examination is clear. A
peripheral blood film shows absolute lymphocytosis. His mother states that he
did not receive any immunizations after the age of 1 year. The most probable
diagnosis is:
A. HiB pneumonia
B. Measles
C. Pertussis
D. Diptheria
E. TB
management of the above patient is best done by:
A. Admission to hospital and supportive care
B. Outpatient therapy with Erythromycin for patient and his family
C. Outpatient therapy with Erythromycin for patient, and booster vaccine for
entire family
D. Outpatient supportive care only
E. Outpatient therapy with Ceftriaxone for patient and Rifampin for family

A 5 year old child comes to your clinic with his mother who states that the child
started developing a rash 5 days ago. She states that the child first had
symptoms of upper respiratory tract infection with cough and conjunctivits
about 10 days ago. On examination, the child was febrile, with macular rash on
the head and trunk. She was also noted to have whitish leasions on the buccal
mucosa. There is no lymphadenopathy. The most common complication of the
above condition is:
A. Otitis Media
B. Encephalitis
C. Subacute sclerosing panencephalitis
D. Pneumonia
E. Myocarditis
F. Arthritis

A mother brings her 1 year old infant to your clinic complaining of a pink
coloured slightly raised rash that developed over the past 7 days covering the
face, neck and extremities. She says that the child first developed a high fever of
40C 10 days ago. The fever subsided three days later only to be followed by the
current rash. Examination shows, in adition to the rash, Occipital
lymphadenopathy. The child is afebrile. The most probable diagnosis is:
A. Measles
B. Rubella
C. Roseola
D. Scarlet fever
E. Erythema infectiosum

A 9 year old boy has microscopic heamturia. A maternal uncle receives chronic
dialysis to for treatment of glomerulonephritis. Which of the following findings
would be most supportive of a diagnosis of a hereditary renal disease:
A. Cherry red spot of macula
B. Chronic hypertension
C. Duplicated renal collecting system
D. Multiple café au lait spots
E. Sensorineural deafness

A 3 months old female infant is suspected of having Turner’s syndrome, which


of the following findings on physical Examination is most suggestive of this
condition:
A. Head circumference of 31 cm, hepatomsplenomegally and absent uterus
B. Head circumference of 36 cm with short arms and legs
C. Hypotonia, high pitched cry and cyanosis
D. Low set ears, short neck and clubbed feet
E. Webbed neck, pedal edema and poor femoral pulses.

An 8 year old had a tonsillectomy and adenoidectomy 10 days ago, she has
complained of pain in the right ear for the past week. Examination of the ear is
normal. Which of the following is the most likely diagnosis:
A. Infected tooth
B. Nasopharyngitis as a complication of the surgery
C. Referred pain from pharynx
D. Subclinical otitis media due to edema of the Eustachian tube orifice
E. Temporomandibular joint pain referred to the ear.

A child born at 32 weeks gestation is brought to your office at 2 months of age


for a routine check up. The child has an upper respiratory tract infection but is
afebrile. Family history of seizure disorder is also present. What decision should
you make regarding the child’s first vaccination:
A. Wait until the child is two months older to administer the first vaccination
B. Wait until the child is clinically well to administer the first vaccination
C. Administer the first vaccination now
D. Consult a peadiatrician regarding the effect of the family’s seizure disorder
E. None of the above

A 23 month old boy has had two episodes of otitis media during the past year
and five episodes of bronchitis with wheezing during the past 6 months. The
appropriate first step in evaluating this child would be to:
A. Ask the parents about smoking in the houshold
B. Determine immunoglobulin levels
C. Place the child on prophylactic antibiotic therapy
D. Refer the child to an allergist
E. Perform an HIV Elisa test

a 4 year old previously healthy girl has had abdominal pain for 12 hours. There
has been no diarrhea or vomiting. She appears ill and has grunting respirations.
Vital signs are RR 50, HR 140, TEMP 39.5. She also has some neck stiffness.
Examination of the abdomen reveals tenderness with mild guarding in the right
upper quadrant. Which of the following is the next appropriate investiation:
A. A barium Enema
B. A cholecystogram
C. A chest Xray
D. An upper gastrointestinal Series
E. CT scan of the abdomen

A 7 year old has a persistant night time cough for 1 year. The most likely cause
of the patients cough is:
A. Foreign body aspiration
B. Immotile cilia syndrome
C. Psychogenic cough
D. Reactive airway disease
E. TB
A 5 year old boy has a seizure that began 45 minutes ago. Initial treatment
includes O2 and anticonvulsant. History and physical examination reveal no
precipitant cause. Which of the following tests would be most urgent to obtain:
A. Toxicology screen
B. EEG
C. CT scan of head
D. Serum Glucose
E. Serum Monia and lactic acid
A 4 year old girl has complained of joint pain for 6 weeks. Physical examination
reveals swelling and tenderness of the knees and right ankle. All other findings
are normal. Lab studies reveal a normal complete blood count, normal ESR, - ve
RF and + ve ANA. Patient with such findings are most likely to develop which of
the following:
A. Pericarditis
B. Iridocyclitis
C. Scaroiliitis
D. Severe destructive joint disease
E. Enthesitis
A 2 year old boy has had fever to 39C, irritability and drooling for 1 day. Physical
Examination reveals the uvula to be in the midline but erythematous and
swollen. The epiglottis appears normal on lateral radiograph of the neck. The
most likely causative organism in this patient is:
A. Coxackie virus
B. Heamophilus influenza type B
C. Non typeable H. flue
D. Pseudomonas aerogenosa
E. Staph. Aureus

A 9 year old boy had had fecal soiling for several years. He denies associated
illness, abdominal pain, or constipation. Physical examination reveals a left
sided mass. Rectal examination reveals stool present with a normal anal tone.
Which of the following is the most likely explanation for this boy’s soiling
problem:
A. Chronic constipation with stool impaction
B. Hirschprung disease
C. Hypothyroidism
D. Left sided colon tumour
E. Psychiatric dysfunction

A case of Kawasaki disease is suspected in a 4 year old male child. Physical


examination will likely reveal each of the following EXCEPT:
A. Cervical lymphadenopathy
B. Edema of hands and feet
C. Fissured lips
D. Polymorphous rash
E. Strawberry tongue
F. Purulent conjunctivits

the most important complication of Kawasaki disease is:


A. Coronary artery aneurism
B. Arthritis
C. Secondary infections
D. Strokes
E. Seizures

A 9 year old girl is seen in the ER because of another episode of recurring


abdominal pain that she has had for the last 4 months. Which of the following
clinical features is LEAST suggestive of an organic cause for this recurrent pain:
A. Pain that is localized to the left flank
B. She has missed three weeks of school in the past four months
C. Recurrent fever of 38 C
D. Recurrent Nausea and vomiting
E. Intermittent diarrhea

A 2 year old girl who is in day care program has had persistent diarrhea for the
past 6 weeks. The stools are described as watery, green, occurring 4-5 times per
say with neither blood nor mucus. She has not gained nor lost any weight. Her
height and weight are both between the 25th and 50th percentile. Physical
examintion is normal. Which of the following is the most likely diagnosis:
A. Celiac disease
B. Giardia Lamblia infection
C. Crohn’s disease
D. Milk allergy
E. Pancreatic insufficiency

In addition to E coli and proteus species. The organism most likely to cause
urinary tract infections in children is:
A. Candida Albicans
B. Enterococcus species
C. Klebsiella species
D. Psudomonas
E. Staph. Saprophyticus
A 16 year old boy presents with a 6 month history of detrioration in school
performance, fatigue and now has become jaundiced. The mother states that
his handwriting has become illegible. His liver is enlarged and hard and the
spleen is also palpable. What is the most likely diagnosis:
A. Autoimmune hepatitis
B. Hepatitis A
C. Hepatitis C
D. Wilson Disease
E. Primary sclerosing cholangitis

On examination, a 2 month old infant is found to have a liver 4 cm below the


costal margin with a respiratory rate of 70/ minutes and a pansystolic murmur
in the 4th intercostal space near the left sternal border with a rumbling mid-
diastolic murmur at the apex. Which of the following congenital abnormalities is
most likely to be present in this child:
A. Transposition of the great vessels
B. Coarctation of the aorta
C. VSD
D. Teralogy of fallot
E. Isolated ASD
A child has fever, sore throat and bilateral non purulent conjunctivitis. Which of
the following organisms is the most likely cause:
A. Adenovirus
B. CMV
C. Beta hemolytic group A strep.
D. Parainfluenza virus
E. Respiratory syncetial virus
A 2 day old breast fed newborn who was delivered at home presents to the ER
with oronasal bleeding. The mother has been on phenytoin during her
pregnancy. The most appropriate management in this patient is to:
A. Administer factor VIII
B. Administer 1-2 mg of vitamin K
C. Administer 20 ml/kg whole blood
D. Adminster platelets
E. Adminsiter 10ml/Kg packed cells

Which of the following is most important in the pathogenesis of Otitis Media:


A. Viral infection of middle ear
B. Bacterial infection of pharynx
C. Obstruction of Eustachian tube
D. Impaired cellular immunity
E. Feeding formula rather than breast milk

the most important goal for rubella immunization is to prevent:


A. Complications of rubella in children
B. Rubella arthritis in adults
C. Subacute sclerosing panencephalitis
D. Congenital rubella syndrome
E. Post rubella encephalitis

A 15 month old infant presents with the sudden onset of rectal bleeding with
large amounts of blood mixed with loose stools. Each of the following should be
considered in the differential diagnosis EXCEPT:
A. Anal fissures
B. Meckel’s diverticulum
C. Intussusception
D. Intestinal polyps
E. Salmonella enteritis
Complications of bacterial meningitis in children include each of the following
EXCEPT:
A. SIADH
B. Deafness
C. Cerebral palsy
D. Seizures
E. Intraventricular haemorrhage

A six year old girl comes to your office for a routine annual check up. On
physical examination, you find that she has a heart murmur. Each of the
following findings would support a diagnosis of a pathological murmur EXCEPT:
A. Murmur occurs in diastole
B. Murmur is associated with a thrill
C. Child has mild finger clubbing
D. Murmur is louder in supine and softer when child sits upright
E. There is a loud P2 component of the heart sounds

A child who has ingested a large quantity of iron tablets will demonstrate each
of the following EXCEPT:
A. Hemorrhagic vomiting
B. Hemorrhagic diarrhea
C. Status epilepticus
D. Metabolic acidosis
E. Radiopaque particles in GI tract

Infants have increased risk of hyperblirubinemmia with each of the following


conditions EXCEPT:
A. Infants with sepsis
B. Premature infants
C. Infants with congenital hypothyroidism
D. Infants with meconium aspiration syndrome
E. RH positive infants born to RH negative mothers

Prostaglandin E2 infusion can be used for the intial management of each of the
following neonatal heart leasions EXCEPT:
A. Pulmonary atresia
B. Coarctation of aorta
C. Tetralogy of fallot
D. Patent Ductus arteriosus
E. Hypoplastic left heart syndrome

A 6 week old infant presents with recurrent vomiting, each of the following is
consistent with a diagnosis of pyloric stenosis EXCEPT:
A. Non bilious vomiting
B. Hypochloremic metabolic alkalosis
C. Palpable epigastric mass
D. Persistent unconjugated hyperbilirubinemia
E. Abdominal distention

The average 4 year old has 8 to 10 respiratory tract infections per year. Factors
contributing to this rate include each of the following EXCEPT:
A. The large number of different respiratory viruses
B. Lack of long lasting immunity after many respiratory viruses
C. Frequent Exposure to other children
D. Chronic excretion of respiratory viruses for months after infection
E. High degree of contagiousness of most respiratory viruses.
A 2 year old boy is brought to the ER with high fever, cough and respiratory
distress. Examination confirms right lung consolidation. There is no
lymphadenopathy and no splenomegally. He is also noted to have small tonsils.
his weight is below the 3rd percentile for his age. Review of records reveals
history of repeated pneumonias, sinusitis and otitis media since the age of 9
months. His immunization record is not available. Investigation later shows sub
normal levels of T lymphocytes and low levels of circulating B lymphocytes
associated with extremely low levels of IgG, IgM, IgE and IgA. What is the most
likely diagnosis in this patient:
AIDS
A. Child neglect (child abuse)
B. Incomplete immunization
C. Inherited Immune deficiency
D. Transient hypogammaglobulinemia of infancy

A 4 year old male child is seen by the doctor because of history of recurrent
respiratory tract and urinary tract infections associated with chronic diarrhea.
His investigation reveals low levels of IgA with normal levels of IgG, B and T cells.
Which of the following statements is correct:
Immunization with live attenuated vaccines should not be given to this child
A. Blood and Blood products should be carefully screened before
administering if this patient needs transfusion for any reason
B. This condition could have been prevented by careful screening of mother
during pregnancy
C. Adenosine deaminase deficiency is the defect found in most of these cases
D. NADPH Oxidase deficiency will be found on further testing

a 90 days old female child was brought to your clinic after her mom noticed that
the cord stump is still not separated. The child was born at term with no
complications. Examination is normal. Which of the following actions is
appropriate in this patient:
A. Reassure the mother that this is normal and it will separate on its own in 1
year
B. Surgically remove the stump
C. Investigate for immunological defects
D. Perform an HIV ELISA test
E. Send the child to the hospital for surgical closure of this umbilical hernia
A 3 week old baby boy was brought to the ER after experiencing a seizure. After
stabilization, the baby was noted to have wide spread eyes, low set ears, and a
narrow mouth opening with a small mandible. He had normal levels of Na, K,
and Glucose. Serum calcium was below normal. What is the most likely cause of
his seizure:
A. Fetal alcohol syndrome
B. DiGeorge syndrome
C. Wiscott Aldrich syndrome
D. McCune Albright syndrome
E. Congenital hypoparathyroidism

A 4 year old boy is brought to the ER 24 hours after he developed respiratory


distress. His parents state that the child has become increasingly agitated with
cough, shorness of breath and fever. Examination shows a child in moderate
respiratory distress with wheezing and stridor. There is decreased breath
sounds on the right with increased percussion note. Examination of the sputum
shows small amount of blood. Xray shows hyperlucency of the right lung field
with signs of hyperinflation especially on expiratory film. The child has no
previous health problems and his immunization record is complete. There are
no pets nor smoking around the house. What is the best next step in the
management of this patient:
A. Give O2 with nebulised salbutamol and start treatment with IV
hydrocortisone
B. Send sputum for culture and start Empiric antibiotic treatment
C. Perform rigid bronchoscope
D. Intubate patient and insert chest tube on the right
E. Perform chest CT scan

A 2 year old female is brought to the ER with a 24 hour history of respiratory


distress consisting of cough with no sputum and fever. The parents indicate that
the child had some sneezing and rhinorrhea about 7 days ago but seems to have
progressed to the current condition. On examination, the child was lying in bed
and appears in moderate distress with interminttent stridor and a brassy
barking cough. There is no wheeze and breath sounds are equal bilaterally. A
chest Xray will most probably show which of the following characterstic signs:
A. Hyperinflation of the lung fields
B. Lateral thumb sign
C. Steeple sign
D. Supraglottic stenosis
E. Diffuse oppacification of the lung fields

the best course of action for the above patient is:


A. Observe in the ER with O2 support
B. Send home and instruct parents to ameliorate symptoms with with
vapourr steam and keeping the child calm
C. keep in ER and give racemic epinephrine and corticosteroid IM plus O2
support
D. Intubate patient and start Ceftriaxone therapy
E. Admit patient to hospital for ventilatory support

A 15 months old baby boy is brought to the ER with a 24 hisotry of respiratory


distress with cough, fever and wheezing that followed a 4 day period of
sneezing and rhinorrhea. There is no history of similar episodes in the past. On
examination, the child is lying in bed with severe respiratory distress evident by
tachypnea with nasal flaring, intercostal recession and wheeze. There is no
stridor. Pulse Oxymetry shows an O2 saturation of 90% Temp is 38 C. Chest Xray
shows hyperinflation. What is the most likely diagnosis:
A. Reactive airway disease
B. Respiratory Syncytial virus infection
C. Parainfluenza virus infection
D. Foreigh body aspiration
E. Acute bronchitis

Management of the above diagnosis may, in general, include all of the following
EXCEPT:
A. Fluid adminstration
B. Bronchodilators
C. Ribavirin
D. RSV immune globulins
E. Steroids

a 4 year old girl presents to the ER with a 24 hour history of fever and
respiratory distress. She has newly relocated to this town with her parents. Her
immunization record is not available. History from her parents reveals that the
child was born at term with no complication during delivery but failed to pass
meconuim in the first 48 hours of life. She also later developed rectal prolapse
and required hospitalization. Examination reveals nasal polyps and suggests left
lung field consolidation with rales. What is the most likely underlying
mechanism causing her current presentation:
A. Thick mucus secretions
B. Abnormally cilliary movements
C. Incomplete immunization
D. Immune deficiency state
E. Increased chloride absorption
Which of the following congenital heart disease does not cause cyanosis:
A. Tetralogy of fallot
B. Transposition of great vessels
C. Truncus arteriosis
D. Ventricular septal defect
E. Coarctation of the aorta

A 12 hour old infant born at 32 weeks gestation was found to have a continuous
murmur over the left sternal border. There are no other findings. What is the
best action to take in this child:
A. Give endomethacin and restrict fluid intake
B. Transfer to surgery for closure of defect
C. Transfer to surgery for ligation of PDA
D. Give PGE1
E. Observe for spontaneous resolution

A 12 hour baby boy born at 38 weeks gestation was found to be cyanotic 12


hours after birth. He was tachycardic and tachypneic with no fever. Xray showed
a narrow upper mediastinum with a heart shadow having an appearance of an
egg lying on its side. What is the best course of action in this patint:
A. Immediate surgical repair of a VSD
B. surgical ligation of a PDA
C. infuse PGE1 to keep ductus arteriosus open
D. Give endomethacin to close PDA
E. Give Oxygen and observe for improvement

features of tetralogy of fallot may include each of the following EXCEPT:


A. VSD
B. Overriding aorta
C. Right sided aortic arch
D. Right ventricle outlet obstruction
E. Hypertrophy of left ventricle
A 5 year old child presents to the ER with a 24 hour history of progressive
shortness of breath, fever and fatigue, and tachypnea. Parents indicate that he
developed symptoms of common cold around six days ago. Examination reveals
a child in moderate distress with sweating and cool extremities. The child has
tachycardia, tachypnea, and hepatomegally. A pansystolic murmur is found. The
child has been previously normal with no history of respiratory or cardiac
disease. What is the most likely diagnosis:
A. Myocarditis
B. Rheumatic fever
C. Infective endocarditis
D. Coxackie A virus infection
E. Undiagnosed coarctation of the aorta

the most common cause of acute diahrrea in children is:


A. infection with E Coli
B. Infection with Rota virus
C. Infection with salmonella
D. Manifestation of systemic infections
E. Side effects of antibiotics

A 5 month old is brought to your clinic with a complaint of the child developing
cough and wheeze and sometimes apnea after feeds. The mother states that
the child has also been spitting up and vomitting since he was 1 month old. The
mother describes posturing of the child consistent with sandifer syndrome. The
child is at the 5th percentile for weight. What is the most likely cause of the
patients symptoms:
A. Tracheoesophageal fistula and atresia
B. Pyloric stenosis
C. Gastroesophageal reflux
D. Diaphragmatic hernia
E. Tracheomalatia

the most common cause of lower GI bleed in a child less than 1 year is:
A. Intussusception
B. Colonic polyps
C. Meckel diverticulum
D. Anal Fissure
E. Volvulus
an 10 months old infant is brought to the ER 12 hours after he started having
colicky abdominal pain. The mother states that the child was diagnosed with
viral gastroenteritis 7 days ago. He had since recovered well until today when he
started having severe pain with vomiting and a slight fever. There is no diarrhea
and the last bowel motion 8 hours ago was normal, although it seemed to have
temporarily relived the pain. On examination, a listless child is seen with a
tender abdomen. A sausage shaped mass is felt in the upper abdomen
associated with an empty left lower quadrant. Digital rectal exam shows normal
stool which is heme negative. What is the best next step in the management of
this patient:
A. Abdominal U/S
B. Baruim enema
C. Emergency laporotomy
D. Sigmoidoscopy
E. Observation with fluid support

4 year old girl presents to her clinician with dysurea. A diagnosis of UTI is made.
This was her first presentation with a UTI. What is the best course of action to
take in this patient:
A. Treat her infection as outpatient, and treat recurrence as it occurs, no
need to investigate
B. Treat her infection as outpatient, reculture urine at end of treatment and
perform renal US
C. Treat her infection, fully investigate only if recurrence occurs
D. Admit to hospital for treatment and immediate VCUG
E. Treat her infection and place patient on long term prophylaxis, no need for
investigation

After an abnormal U/S following the first episode of a UTI in a 3 year old, a
VCUG showed grade 3 vesicoureteric reflux. No renal scarring is seen on IVP.
What is the best action to take:
A. Place child on nitrofurantoin therapy and wait for spontaneous resolution
B. Treat only if infections occurs
C. Refer for surgical correction immediatly
D. No need for intervention at this time
E. Re evaluate in 1 month, and surgically correct if no resolution
All of the following are poor prognostic factors associated with Acute
Lymphoblastic Leukemia EXCEPT:
A. Male
B. Black
C. WBC > 100, 000/mm3
D. CNS involvement
E. Age between 2 and 10
Most common solid tumor in children is:
A. Wills Tumor
B. Brain Tumor
C. Neuroblastoma
D. Leukemia
E. Lymphoma

a 2 year old child was brought to the ER after having a seizure. The parents who
accompanied the child stated that the child had a fever of 38 the night before
associated with earache. The parents were planning to bring the child to his
physician today but because of the seizure, they decided to take him to the ER.
The seizure lasted 5 minutes, was a generalized tonic clonic and the child
recovered rapidly after the seizure ended. There is no family history of seizure
disorder. Examination reveals bulging, immotile and hyperemic right tympanic
membrane with a temp. of 39C. What is the most appropriate next step in
management:
A. Reassurance, antipyretic and antibiotic
B. Antibiotic, Anticonvulsant and EEG
C. Lumbar puncture, Anticonvulsant and Antipyretic
D. Reassurance, anticonvulsant and EEG
E. Lumbar puncture, Antibiotics and Anticonvulsant
F. EEG, Lumbar puncture and anticonvulsants

A 6 months old infant is brought to the clinic by his parents. They state that over
the last 2 months, they noticed that the child was having episodes of symmetric
rhythmic contractions of the trunk and extremities that were very brief but very
frequent. Suspecting a particular cause, the doctor orders an EEG, which
showed hypsarrhythmia. What is the best treatment modality for this condition:
A. Treatment with Phenytoin
B. No treatment required as spontaneous resolution is expected
C. Treatment with thyroid hormone
D. Diagnoses of the underlying condition is required before treatment can be
decided
E. Treatment with adrenocorticotropic hormone
In cerebral palsy, which statement is CORRECT:
A. Progressive motor deterioration is a common feature
B. Birth asphexia is a very common cause
C. Prematurituy does not increase risk
D. Has a higher incidence of scoliosis
E. Spastic cerebral palsy occurs in less than 30% of cases

Paediatrics EOP 2010/2011 Group A - Females


4 years old patient with fever and vomiting, shortness of breath on exertion, on
examination he was feverish with tachycardia, gallop rhythm & muffled heart
sounds. The most likely diagnosis is:
A. ASD
B. Pericarditis
C. Costochondritis
D. Mitral heart prolapse
E. Non of the above

Regarding napkin rash:


A. Mainly it resolve by exposure to air
B. Associated with infantile eczema
C. More common in boys
D. Due to ammonia
E. Decreased in infants taking soy milk formula

Female child came with +ve nitrate & +ve leukocyte estrase. Regarding UTI
which of the following is most appropriate :
A. All children <5 years with recurrent UTI should be investigated
radiologically for anomalies.

The prevalence of congenital heart disease among general population


B. 0.8 %
C. 3 %
D. 5 %
E. 10 %
F. 20 %
Which of the following is associated with turner syndrome:
A. Lymphedema of the hands and feet
B. Low FSH
C. Low LH
D. Ovarian cyst
E. Malignancy

The most diagnostic feature of tuberous sclerosis is:


A. Ash leaf spots
B. Café-au-lea spots
C. Lipoma

Patient looking ill, high grade fever, drooling of saliva with inspiratory stridor:
A. Epiglottitis
B. Croup

Patient irritable, crying, pulling ear. Ear drum was erythematous and bulging on
examination. The most likely organism is:
A. Streptococcus Pneumoniae
B. Haemophilus influenzae
C. Moraxella catarrhalis
D. CMV

Patient, 4 years old, with exertional dyspnea, has ejection systolic murmur on
upper sternum, fixed splitted S2, normal S1, grade 2/6 murmur:
A. ASD
B. VSD

Regarding nephrotic syndrome:


Steroids are given every other day to maintain remission.

A case of a patient with DKA, high blood glucose, ketonuria, glucosuria,


metabolic acidosis & dehydration. The most important step in management is:
A. Intravenous normal saline
B. IV insulin

A case of short stature: a child shorter than his class peers but otherwise
healthy, his upper and lower segments found to be appropriate:
A. Growth Hormone Deficiency : Give growth hormone.
Infant with galactosemia most likely he will present with:
A. Vomiting, jaundice, hepatomegaly
B. Glucosuria

Patient with cystic fibrosis: sweat chloride test found to be > 40 mmol/L :
A. Pancreatic enzymes replacement.

Celiac disease associated with:


B. Tall stature
C. IDDM ( DM type 1 )
D. DM type 2

7 months old child he can’t do:


A. Pincer grasp.

A case of a child inhaled gasoline gas and started to vomit and then stopped
vomiting when arrived to the ER, the 1st thing to do in ER is:
A. Pulse oximetry, and ABGs.
B. Administer ipecac

6 months old. What he can do:


A. Reach out for toys.

Most common presentation of neonatal meningitis is:


B. Seizures
C. Neck rigidity
D. Kerning’s sign

Cause of polyhydrominous:
A. Oesophageal atresia
B. Cleft palate
C. Renal agenesis
A case of a newborn after his first feed developed abdominal distension,
drooling of milk, cough, choking:
A. Diaphragmatic hernia
B. Duodenal atresia
C. Oesophageal atresia with fistula
A case of a boy with URTI received antibiotic and he developed pallor and
jaundice:
A. G6PD deficiency

Regarding childhood bronchial asthma:


B. History of allergic diseases is uncommonly present
C. Normal PCO2 in acute attack of asthma is reassuring for the treating
physician
D. Absence of wheeze is a good sign of improvement and indicate no need
for treatment
E. URTI is the most common precipitating factor
F. Nebulised salbutamol should not be given for less than Q4 h.

Umbilical cord usually fall of at:


A. Immediately
B. 2 days
C. 2 weeks
D. 2 months

Regarding breast milk (or cow’s milk ) . 100 cc = :


A. 67 kCal
B. 76 kCal

Regarding Hb electrophoresis. Elevation of Hb A2 is found in:


A. SCA
B. Thalassemia trait

Most serious complication of systemic onset juvenile idiopathic arthritis is:


A. Uveitis ( not 100% sure !! )
B. Pericardial effusion

Regarding breast milk jaundice:


A. Mainly unconjugated hyperbilirubinemia

Posterior fontanelle open:


A. Do TSH. Most common cause is congenital hypothyrodism.

Q1) Among the following. Life threatening anaphylaxis in children occurs most
often as a result of exposure to :
A. Exercise
B. Foods
C. Latex
D. Preoperative drugs
E. Vaccine

Q2) which of the following clinical features is most likely to be associated with a
benign condition?
A. Bleeding 7 days after a tonsillectomy
B. Bruises over the bony prominences of the extremities, both proximal and
distal
C. Epistaxis (worse in winter)
D. Hemarthrosis
E. Menstrual bleeding that last 8 days.
Q3) which of the following findings on the newborn examination may be
normal?
A. Fixed S2
B. High-pitched murmur
C. Obscured S1
D. Precordial thrill
E. Systolic ejection murmur

Q4) you identify a 3-cm nonfluctuant abscess on the shoulder of a previously


well 3 years old boy who has an allergy to both penicillin and ???.Because the
incidence of MRSA is less than 5% in your community, the cephalosporins with
warm compresses is likely to provide effective treatment. The best choice is :
Cefaclor
A. Cefadroxil
B. Cefixime
C. Ceftriaxone
D. Cefuroxime axetil

Q5) which of the following symptoms of GERD is more common in older children
and adolescents than in infants and young children?
A. Apparent life-threatening event
B. Asthma
C. Failure to thrive
D. Hematemesis
E. Recurrent pneumonia
Q6) A previously healthy 5- year old boy presented with hematochezia. Physical
examination ?????????. the mother reports that he has had ????????? that did
not affect his activity. The most likely :
A. Henoch-schonlein purpura
B. Infective colitis
C. Juvenile polyp
D. Meckel diverticulum
E. Superior mesenteric aneurysm
F. Q7) postnatal immuno-prophylaxis is more effective against hepatitis B
During delivery
G. In utero
H. Through breast feeding
I. Through salivary transmission
J. Via feco-oral

Q8) HBs AB with negative HBc AB :


A. Active infection
B. High viral reproduction rate
C. Immunity after immunization
D. Immunity after recovery from infection
E. Low viral reproduction rate.
Q9) most common blood type:
A. B-
B. AB-
C. AB+
D. O-
E. O+
Q10) the percentage of blood loss during surgery that typically is used to trigger
transfusion in the absence of ???? measures is :
A. 5%
B. 10%
C. 15%
D. 20%
E. 25%

Q11) A 4 week old breastfeeding boy is jaundiced and has a total billirubin
concentration of 1.3mg/dl . the laboratory test that maximizes diagnostic
efficiency is:
A. Complete blood count
B. A reticulocyte count
C. Billirubin fractionation
D. Gamma glutamyl transferase
E. Hepatic Aminotransferase
Q16) what is the most common underlying cause for mild to moderate
neutropenia?
A. Exposure to medication such as antibiotics
B. Immune neutropenia
C. Shwachman-diamond syndrome
D. Sequestration
E. Transient marrow suppression due to viral infection

Q17) which of the following statements regarding pneumonia in children is


true?
A specific microbial pathogen usually can be identified.
A. All children who have pneumonia should be hospitalized for observation
and treatment.
B. Pneumonia is a rare cause of child mortality worldwide
C. Radiographs of the chest always should be obtained to determine the
cause
D. Viral agents are the most common cause of pneumonia in older infants
and young children.

Q18) A healthy 2 month old infant was born at 32 weeks gestation. She has
grown well since birth. On physical examination of this infant, the MOST likely
finding is:
A. Ability to fixate on a face and follow ???
B. Ability to reach and grasp a ???
C. Ability to watch an object and follow it to midline
D. Absence of the motor reflex
E. Dabbling and cooing vocalization

Q19) The decreased incidence of enteric infections in breastfed infants


compared with formula-fed infants is MOST likely due to the :
A. More alkaline stool pH in breastfed infants
B. Nutritional benefits of human milk on the infant’s immune system
C. Predominance of bacteroides and ?? in the gut of breastfed infant
D. Presence of ?? antibodies against enteric infection in human milk
E. Sterility of human milk
Q20) of the following, a TRUE statement regarding use of oral rehydration
solution in the management of acute gastroenteritis is that
A. Feedings are initiated following 24 hours of bowel rest and intravenous
fluid therapy
B. Fermented carbohydrates are resorbed in the colon
C. Fluid and electrolytes are replaced via sodium-glucose cotransport in the
small intestine
D. High sodium concentrations allows for replacement of total body sodium
deficiency
E. Hyperosmolar solutions containing complex carbohydrates are required.

Q21) A TRUE statement about glumerular filtration rate, corrected for surface
area, at 2 years of age is that it is :
A. Double that observed at birth ‫وهللا اعلم‬
B. Equal to that observed at birth
C. Double that observed at 6 months
D. Triple that observed at 6 months
E. Equal to that observed in adults

Q22) iron deficiency in a child who has unrepaired cyanotic congenital heart
disease is MOST likely to increase the risk for :
A. Congestive heart failure
B. Digoxin toxicity
C. Osteoarthropathy
D. Pulmonary hemorrhage
E. Stroke

Q23) A 1 day old term baby develops bilious vomiting and poor feeding. You
recall that the deferential diagnosis of vomiting is age related. Of the following,
the condition that is MOST likely to cause bilious vomiting in this infant is :
A. Gastric stress ulcer
B. GERD
C. Gastrointestinal food allergy
D. Intussusception
E. Midgut volvulus

Q24) in a patent with rheumatic fever, of the following manifestations of acute


rheumatic fever which is not relieved by salicylate or steroid therapy?
A. Carditis
B. Abdominal pain
C. Arthritis
D. Chorea
E. Fever
Q25) A new born is diagnosed with congenital heart disease. You counsel the
family that the incidence of heart disease in future children is:
A. 1 %
B. 2 to 6 %
C. 8 to 10 %
D. 15 to 20 %
E. 25 to 30 %
F. Nutrition

1-Good signs of dehydration


A) palpation anterior fontanel
B) tachycardia
C) delayed capillary refill
D) hypotension
E) lethargy, coma

2- Diet Management in Nephrotic syndrome include


A) protein 2 g/kg
B) Fat more than 30 % of total cal.
C) cholesterol more than 200 mg per day
D) low in polyunsaturated fatty Acid
E) Do not administer Iron unless there is clear evidence of Iron deficiency

3- All of the following true for Diet Management in Diabetes Mellitus EXCEPT?
A) Regular food pattern should be emphasized
B) Caloric intake 40% CHO
C) Encouraged low salt.
D) Encouraged low saturated fats and high fiber diet.
E) Distribute carbohydrate load evenly during the day preferably 3 meals & 2
snacks

4- All of the following Indications of Total PARENTERAL NUTRITION EXCEPT?


A) Esophageal atresia
B) Severe FTT
C) Severe burn
D)Inflammatory Bowel Disease
E) BMI<25

5- All of the following Comlication of Total PARENTERAL NUTRITION EXCEPT?


A) Hypoglycemia
B) Hyperglycemia
C) Hypocalcaemia
D)Metabolic alkalosis
E) Metabolic acidosis

6- All of the following associated with food allergy EXCEPT?


A) Angioedema
B) Vomiting
C) Fever
D) Laryngeal edema
E) Abdominal cramps

7- All of the following associated with Familial Hypercholesterolemia EXCEPT?


A) Autosomal co-dominant disorder
B) Elevated LDL cholesterol
C) Tendon xanthomas
D) Elevated HDL cholesterol
E) Incidence 1/5000
F)D+E
8- All of the following associated with Familial Hypertriglyceridemia EXCEPT?
A) Autosomal co-dominant disorder
B) It is characterized by elevation of plasma triglycerides
C) May be effected coronary vascular system
D) Incidence ≈1/500 individuals
E) Decreased levels of chylomicrons

9- All of the following SECONDARY CAUSES OF Hypercholesterolemia EXCEPT?


A) Hyperthyrodism
B)Nephrotic syndrome
C)Cholestasis
D)Anorexia nervosa
E)Tegretol
10- All of the following risk factor for coronary heart disease EXCEPT?
A) Elevated HDL cholesterol
B) Smoking
C) Obesity
D)Malnutrition
E)B- Blockers

11- All of the following associated with bacterial rather than viral GE EXCEPT?
A) Visible blood mixed with faeces
B) Febrile seizure
C) Diarrhoeal frequency more than 5 stool per day
D)Full blood count showing Hb 5.2 g/l,WBC14.5 PLT 102
E)Pulse rate 80/min. in a child of 2 years

12- All of the following causes the onset of persistent vomiting in 3 week old
child?
A)Disaccharidase intolerance
B)Duodential atresia
C) Pyloric stenosis
D)Hiatus hernia
E)Choledochal cyst
F)C+D

13-A reduced red cell folate may be found


A)In celiac disease (gluten enteropathy)
B)After resection of the terminal ileum
C) After long term treatment with valporal
D) After 2 weeks of a low folate diet
E) Where there is deficiency of B 12
F)A+C

14- All of the following associated with infantile GE EXCEPT?


A)Is commoner in bottle-fed babies than breast-fed babies
B) In frequently due to rotavirus
C)May be complicated by subsequent Disaccharit
D)Antibiotic not indicated
E)Is associated with pathogenic E.Coli in the majority of cases
15- All of the following associated with Congenital Pyloric stenosis EXCEPT?
A)Familial tendency
B) Crumb in abdomen
C) Alkaline urine
D)Increase incidence in boys
E) Dehydration

16- All of the following are reccgnised causes of nonorganic failure to thrive ?
A) Maternal eating disorders
B) Maternal depression
C) Inadequate housing
D) Inadequate social support
E)Lack of extended family
F)A+B

17- All of the following are though to be lung term out comes of early nutritional
deficiency?
A)Short stature
B) DM
C) Ischaemic heart disease
D) Obesity
E)Obstructive lung disease

18- All of the following are known disadvantages of milk formulae EXCEPT?
A) Approximately 25%of infants with cows milk protein intolerance with
developsoy intolerance
B) Soy milk formulae tastes worse than elemental milks
C) Infants respond poorer to vaccination than elemental milks
D) Infants have lower complement levels than elemental milks
E) Infants have lower aluminum levels than elemental milks

19- All of the following causes of noninfective diarrhea EXCEPT?


A) Lead poisoning
B) Hypothyrodism
C)Hirschprungs disease
D)Gliadin sensitivity
E)Feeding difficulty

20- All of the following are true regarding the diagnosis of cows milk intolerance
EXCEPT?
A)Skin test usually confirm the diagnosis
B) RAST test of >grade 2 makes the diagnosis highly likely
C) Bloodly diarrhea excludes the diagnosis
D)It is common in breast-feed infants
E)Rechallenge should be extremely caution us in the comptred with bottle-fed

21-Breast feeding is redatively protective against


A)Late haemorrhagic disease of the nowborn
B) Maternal breast cancer
C) Late onset diabetes
D)Prolonged jaundice
E)Under feeding

22-Malabsorption syndrome with primary mucosal abnormally inclue


A) Pancreatic insufficiency
B) Cron s disease
C) Abetalipoproteinemia
D)Blind-loop syndrome
E)Coeliac disease
F)B+C+E

23- Breast milk is more rich than Cow’s milk in:


A) Lactalbumin
B) Sodium.
C) Calcium.
D)Iron
E)Folic acid

24- All of the following are true Breast feeding EXCEPT?


A) Recommended food for infants both term and preterm
B) 50%of energy from proteins
C) Contains immunological benefits (Ig A ,active lymphocyte)
D)Promotes growth of lactobacillu in GI
E)Decreases incidence of allergy disease

25-Signs and symptom of 5% dehydration?


A) Oliguria , tear with crying ,loss active than usual .normal skin turgor , moist
oral mucosa
B) Oliguria ,no tears with crying ,loss active than usual,sticky oral mucosa,
normal or slightly diminished skin turgor
C) Oliguria ,no tears ,sunken eyes, slightly diminished skin turgor
D) Oliguria , sunken eyes, tenting ,tachycardia , hypotension

26-Common allergy disease in school age ?


A) Atrophic dermatitis
B) Food allergy
C) Asthma
D)Allergic rhinitis
E) Drug allergy

27- Cause of Food allergy?


A)Recurrent dizziness after eating Chinese foods
B) Recurrent tingling sensation in the mouth after eating apiece of apple
C) Recurrent palpitations after drinking a cup of coffee
D) Recurrent diarrhea after drinking a glass of milk

28- Calculate fluid .,Na, K for baby has 40 kg EXCEPT?


A) Fluid 1900 ml
B) Na 120 meq
C) K 80 meq
D) A,B,C
E) Na 80 meq

29- All of the following are wrong for dehydration EXCEPT?


A) Capillary refill less than 2 second mild dehydration
B) Absent tear mild dehydration
C) Drowsy moderate- mild dehydration
D) Sunken eyes mild dehydration
E) Increased urine output sever dehydration

30- All of the following are true for hypernatramia EXCEPT?


A) The patient look not dehydration
B) Doughy skin
C) Serum Na 155 meq \L or more
D) The fluid given 1\2 of deficit with 8 h
E) Convulsion hypocalcaemia can occur

31- All of the following are nutrition delivery enteral feeding EXCEPT?
A) Oral feeding
B) NOT
C)Subcutaneous infusion
D) Gastrostomy feeding
E) Jejenostomy feeding

32. Breast milk is more rich than Cow’s milk in:


A) Carbohydrates.
B) Proteins.
C) Sodium.
D) Calcium.
E) Phosphorus.

33. All are early findings in Rickets EXCEPT:


A) Normal serum calcium.
B) High serum phosphorus.
C) Radiological changes.
D) Craniotabes.
E) Rachitic rosary.

34-Skeletal manifestations of advanced rickets include:


A-Head bossing.
B-Delayed dentition.
C-Marfan’s sign.
D-Harrison’s sulcus at thorax.
E-All of the above.

35-Causes of death in kwashiorkor include:


A-Intercurrent infection.
B-Water and electrolyte disturbances.
C-Hypoglycemia.
D-Hypothermia.
E-All of the above.

36.Recognized clinical pictures of marasmus include:


A-Growth failure.
B-Hunger.
C-Loss of subcutaneous fat.
D-Muscle wasting.
E-All of the above.

37-Adequate diet should supply:


A-Adequate amount of water.
B-Adequate calories.
C-Adequate carbohydrates, fat & proteins.
D-Adequate amount of fiber.
E-All of above

38-Contraindications of breast feeding include all of the following except:


A-Active Tuberculosis.
B-Severely undernourished mother.
C-HIV infection.
D-Premature infant.
E-Abnormalities of the mouth of the newborn.

39-Constant findings of clinical pictures of kwashiorkor include:


A-Growth failure.
B-Mental changes.
C-Edema.
D-Disturbed muscle-fat ratio.
E-All of the above.

40- Weaning means:


A-Stop breast feeding completely and start normal food.
B-Stop breast feeding completely and start formula feeding like Nan or Materna
milk.
C-Introduction of foods other than milk in the infant’s diet.
D-Usually started at the age of 10-12 months.
E-None of the above.

41-The following advantage of breast milk except:


A. Is chosen in preference to a preterm formula in< 30 wk.
B. Should be avoided during episodes of GE.
C. Contains Ig A.
D. Has lower phosphate content than cow's milk.
E. Helps prevent GE.
42- A 4-month-old boy weighed 3500 g at birth. He now weighs 4.5 kg. He has
been formula fed since1 week of age.The MOST likely reason for this child's
failure to thrive is:
A) absence of solids in his diet
B) exaggerated parental concern about overfeeding
C) improper feeding technique
D) omission of supplemental vitamins
E) withholding of nighttime feedings

43-You are asked to help prepare an oral rehydration solution.You recall that, in
addition to water, the MOST important components of such a solution are:
A) bicarbonate and potassium
B) chloride and potassium
C) lactate and potassium
D) sodium and fructose
E) sodium and glucose
44-Which of the following vitamins is in higher concentration in cow's milk than
in human milk?
A. A
B. C
C. E
D. K
E. B6

45- in vitamin deficiencies all are correct EXCEPT?


A. xerosis conjunctivae in vitamin A deficiency.
B. tender nerves in vitamin B1 deficiency.
C. photosensivity in niacin deficiency.
D. seizures in pyredoxin deficiency.
E. cerebellar ataxia in riboflavin deficiency.

46- A TRUE statement regarding anthropometric measurements in the


assessment of nutritional status is:
A) Acute changes in weight reflect changes in muscle mass
B) Arm circumference is the best screening tool for malnutrition
C) Single measurements are the most sensitive indicators of nutritional
problems
D) Standard growth curves are equally applicable to all ethnic groups
E) Standard growth curves overestimate the early gains to be made by breastfed
infants

47- Of the antibodies found in human colostrum and milk, the immunoglobulin
(Ig) that is MOST likely to prevent organisms from adhering to the infant's
intestinal mucosa is:
A) IgA
B) IgD
C) IgE
D) IgG
E) IgM

48- Advantages of breast milk include all of the following except:


A-Economic and cheaper than artificial milk.
B-Sterile & free from contamination.
C-Contains antibodies.
D-Contains protein higher than artificial milk.
E-Has psychological advantages.
1 A 11 C 21 B 31 C 41
2 E 12 F 22 F 32 A 42
3 B 13 F 23 A 33 E 43 E
4 E 14 E 24 B 34 44
5 D 15 C 25 A 35 45
6 C 16 F 26 D 36 46
7 F 17 D 27 B 37 E 47 A
8 E 18 B 28 E 38 48 D
9 A 19 B 29 A 39
10 A 20 E 30 D 40

A 9-month-old infant has atopic dermatitis characterized by erythematous,


chapped-appearing patches on the cheeks and extremities. Of the following, the
MOST appropriate initial treatment is
A. administration of an oral antibiotic
B. administration of an oral antihistamine
C. administration of an oral corticosteroid
D. application of a low-potency corticosteroid
E. avoidance of cow milk protein
A mother calls to report that her child was exposed to chickenpox in preschool.
The girl is healthy and has no history of varicella. The parents have refused
varicella vaccine. Of the following, your BEST advice is that the child should not
attend preschool
A. for the period 10 to 28 days following the exposure
B. for the period 10 to 21 days following the exposure
C. for 2 weeks following the exposure
D. if she develops chickenpox lesions
E. until chickenpox is no longer circulating in the classroom
A reporter for the local newspaper is interviewing you for an article on
preschoolers. He asks you to list some normal milestones for 4-year-olds. Of the
following, the MOST typical milestone for a 4-year-old is to
A. copy a square and triangle
B. prefer solitary or parallel play
C. print his or her first name
D. speak clearly in sentences
E. tie his or her shoelaces

A 10-year-old boy develops severe headache, photophobia, and emesis. He is


afebrile, and his blood pressure is 180/100 mm Hg. Results of cranial nerve and
motor examination are normal. He has nuchal rigidity and extensor plantar
reflexes bilaterally (positive Babinski sign). Of the following, the MOST
appropriate study to obtain at this time is
A. cerebrospinal fluid analysis
B. computed tomography
C. electroencephalography
D. magnetic resonance imaging
E. radiography of the skull

5. After intubation, arterial blood gas measurements for a 12-hour-old term


infant include a PO2 of 18 torr and a PCO2 of 25 torr while receiving ventilation
with 100% FIO2. Chest radiography shows normal cardiac size and diminished
pulmonary vascularity. The mechanism by which intravenous prostaglandin E1
will benefit this infant is MOST likely due to
A. decreased pulmonary vascular resistance
B. decreased systemic vascular resistance
C. increased mixing of systemic and pulmonary circulations
D. increased pulmonary blood flow
E. increased systemic blood flow
6. A female infant born to a 24-year-old woman has been diagnosed clinically
as having Down syndrome. The mother is concerned about her risk of having
another child who has a chromosomal abnormality. The statement that you are
MOST likely to include in your discussion is that her risk
A. can be estimated by determination of maternal serum alpha-fetoprotein in
all future pregnancies
B. cannot be estimated until her infant's chromosome complement has been
determined
C. is increased for Down syndrome, but not for any other chromosomal
abnormality
D. is no greater than that of any other woman her age
E. is not increased until she reaches the age of 35

7. A 3-year-old boy has a 1-week history of abdominal distension, flatulence,


and frequent bowel movements (six to eight movements every 24 hours). You
suspect a parasitic infection and obtain stool samples to test for ova and
parasites. Results are positive for Entamoeba histolytica. Of the following, the
MOST appropriate initial medication to administer is
A. diloxanide
B. furazolidone
C. iodoquinol
D. metronidazole
E. paramyocin

8. A 6-month-old boy has a pattern of sleep and feeding that is very


unpredictable from day to day. He is easily startled by new sounds. He cries
loudly when upset and takes several minutes to console. He sleeps with his
parents, and his growth and development have been normal.Among the
following, the best INITIAL management of this infant would be to
A. counsel the parents about differences in infant temperament
B. hospitalize the infant for observation
C. obtain an upper gastrointestinal series
D. prescribe chloral hydrate for sedation
E. switch to a protein hydrolysate formula

9. A 10-year-old boy is brought to the emergency department after being struck


by a car while riding his bicycle. Physical examination immediately upon arrival
reveals that he is responsive to painful stimuli only by moaning. His pupils are
equal, small, and reactive.
Of the following, the sign MOST LIKELY to be associated with increasing
intracranial pressure in this patient is
A. hyperventilation
B. hypotension
C. hypothermia
D. nystagmus
E. Tachycardia

10. An infant who was born with myelomeningocele has hydrocephalus, and a
ventriculoperitoneal shunt is placed. His parents are concerned about
recognizing the signs and symptoms of shunt blockage. Of the following, the
BEST information you can give the parents is that
A. behavioral changes such as decreased spontaneity and mild lethargy may
indicate shunt malfunction
B. fewer than 30% of all shunts malfunction
C. most shunt failures occur more than 5 years after the initial surgery
D. seizures are the most common manifestation of shunt malfunction
E. shunt malfunction is usually due to infection

11. A 6-year-old girl is brought to your office for evaluation of swelling of her
eyes and legs. Three weeks ago she had a fever, bloody diarrhea, and abdominal
pain that resolved spontaneously. Laboratory studies reveal anemia, mild
thrombocytopenia, normal electrolyte levels, elevated blood urea nitrogen and
creatinine concentrations, and proteinuria. Of the following, the MOST likely
cause of this child's edema is
A. acute onset of nephrotic syndrome
B. Berger disease
C. hemolytic-uremic syndrome
D. Henoch-Schönlein purpura
E. poststreptococcal glomerulonephritis

12). A male infant is born at an estimated gestational age of 34 weeks. His


measurements at birth are: weight, 1,200 g (<10th percentile); crown-heel
length, 40 cm (10th percentile); and head circumference, 31.5 cm (50th
percentile).
Of the following, the MOST likely explanation for the growth pattern of this
infant is
A. chromosomal abnormality
B. congenital viral infection
C. gestational diabetes
D. hereditary constitution
E. pregnancy-induced hypertension

13. A 16-month-old girl is brought to the emergency department for


evaluation of a fever, irritability, and possible seizure. She has been receiving
amoxicillin to treat otitis media for the past 3 days. The child appears ill, is
irritable, and has a temperature of 39°C. Studies obtained on the cerebrospinal
fluid reveal: glucose, 1.67 mmol/L (30 mg/dL); protein, 0.8/L (80 mg/dL); and
white blood cell count, 500 x 106/L (500/mm3) with 85% polymorphonuclear
leukocytes. Gram stain is negative, and cultures are pending.Of the following,
the MOST likely diagnosis is
A. herpes simplex virus encephalitis
B. Lyme disease
C. partially treated bacterial meningitis
D. tuberculous meningitis
E. viral meningitis

14) You have been asked to evaluate a 7-year-old boy who has moderate
persistent asthma. He has not seen any primary care physician for more than 6
months. You decide to formulate an appropriate treatment plan for him.
Of the following, the BEST maintenance therapy is
A. inhaled beta-agonist
B. inhaled corticosteroid daily
C. leukotriene receptor antagonist daily
D. nebulized cromolyn sodium
E. oral theophylline

15) A7-year-old boy presents with left-sided proptosis and a 1-week history of
purulent nasal discharge and fever. On physical examination, you observe
downward and outward left-sided proptosis with good visual acuity but with
limitation of left lateral gaze.
Of the following, the MOST appropriate diagnostic test is
A. a lumbar puncture
B. computed tomography with contrast of the brain
C. computed tomography with contrast of the paranasal sinuses
D. magnetic resonance imaging of sinuses
E. sinus radiography

16) A 16-month-old boy has had severe emesis and diarrhea for 3 days. On
physical examination, he appears dehydrated. You administer fluids rapidly to
correct his volume loss and serum electrolyte abnormalities. On the next day,
he appears confused and exhibits quadriparesis and dysarthria. Magnetic
resonance imaging reveals demyelination of the central basis pontis.
Of the following, the rapid correction of which condition is MOST likely
responsible for these findings?
A. hyperkalemia
B. hypermagnesemia
C. hypocalcemia
D. hyponatremia
E. hypophosphatemia
17) A 6-month-old previously healthy infant presents with a 2-day history of
cough and difficulty breathing. On physical examination, she appears lethargic,
pale, and poorly perfused. She exhibits marked tachypnea, and auscultation of
the chest reveals decreased breath sounds bilaterally and poor aeration. She
has supraclavicular and intercostal retractions. Of the following, the MOST
appropriate therapy for this infant is
A. continuous positive airway pressure via face mask
B. endotracheal administration of surfactant
C. endotracheal intubation and positive pressure ventilation
D. negative pressure ventilation
E. nitric oxide

18) The decreased incidence of enteric infections noted in breastfed infants


compared with formula-fed infants is MOST likely due to the
A. more alkaline stool pH in breastfed infants
B. nutritional benefits of human milk on the infant's immune system
C. predominance of Bacteroides and Clostridium in the gut of breastfed
infants
D. presence of protective antibodies against enteric infection in human milk
E. sterility of human milk

19) A newborn who weighs 600 g and whose estimated gestational age is 24
weeks at birth is admitted to the neonatal intensive care unit after successful
resuscitation in the delivery room. Arterial blood gas measurements on room air
are: pH, 7.35; PCO2, 42 mm Hg; PO2, 68 mm Hg; base deficit, 2 mEq/L.Of the
following, the MOST appropriate initial management is to
A. begin intravenous vancomycin
B. begin phototherapy
C. initiate enteral feeding
D. provide bicarbonate infusion
E. provide glucose infusion

20) You have been asked to evaluate a 15-month-old toddler who was admitted
to your community hospital for observation 6 hours ago after a near-drowning
episode. The nurse caring for the toddler reports that the child has become
more tachypneic and distressed. Her oxygen requirement also has increased.
Of the following, the MOST likely diagnosis is
A. adult respiratory distress syndrome
B. upper airway oedema
C. bacterial pneumonia
D. pneumothorax
E. reactive airway disease

21) Two days ago a 5-year-old girl undergoing chemotherapy for acute
lymphoblastic leukemia spent the morning with a playmate that now has
developed varicella. She has not received varicella vaccine and has no history of
having varicella.
Of the following, the MOST appropriate action is to
A. administer varicella vaccine
B. administer varicella-zoster immune globulin
C. begin oral acyclovir
D. discontinue chemotherapy for 3 weeks
E. obtain varicella titers to determine the need for therapy

22) The 23-valent pneumococcal polysaccharide vaccine is RECOMMENDED for


the child who
A. attends a large child care center
B. has frequent ear infections
C. is older than 2 years of age and has nephrotic syndrome
D. is younger than 2 years of age and has sickle cell disease
E. lives in the same household as a patient who has asplenia
23) A 9-month-old girl who attends chid care presents with fever, irritability,
and pulling at her left ear. Physical examination reveals a bulging, erythematous
left tympanic membrane and no movement with pneumatic otoscopy. Findings
on the remainder of the examination are unremarkable, and the girl has no
history of ear infections.
Of the following, the MOST appropriate STEP to manage this child
A. obtain blood and CSF culture
B. amoxicillin 80 mg/kg per day for 10 days
C. referred to ENT specialist for tympanoplasty
D. covered with penicilline G 100 mg/kg
E. azithromycin 10 mg/kg on day 1 and 5 mg/kg for days 2 through 5

24) An 18-month-old infant is brought to your office for evaluation of diarrhea


that occurs every 2 or 3 days, but does not interfere with her appetite, activity
level, or sleep. Attempts at controlling her loose stools with dietary modification
have been unsuccessful. Results of a physical examination, including growth and
development, are normal.
Of the following, the MOST likely cause of this child's diarrhea is
A. chronic nonspecific diarrhea
B. cow milk allergy
C. giardiasis
D. lactose intolerance
E. postviral gastroenteritis syndrome

24) You are examining a term newborn in the nursery. His weight is 3.27 kg
(50th percentile), and his length is 50.5 cm (50th percentile). The pregnancy,
labor, and delivery were unremarkable. There are no significant findings on
physical examination.
The MOST likely head circumference in this child, if it is consistent with his other
growth parameters, is
A. 31 cm
B. 33 cm
C. 35 cm
D. 37 cm
E. 39 cm

25. A healthy 8-month-old boy has had intermittent episodes of intense


cyanosis of the hands, lower arms, and feet since birth. The infant appears alert
and playful during the episodes. At other times, his arms and legs are pink but
have a "mottled" appearance. Physical examination reveals no remarkable
findings other than mottling of the extremities.
Of the following, the BEST diagnostic test for this child is
A. ambulatory electrocardiography to detect arrhythmia
B. co-oximetry to detect methemoglobinemia
C. echocardiography to detect congenital heart disease
D. electroencephalography to detect a seizure disorder
E. no test because these symptoms are benign

26. You are examining a 6-month-old boy who has infantile spasms. There is a
family history of tuberous sclerosis.
Of the following, the cutaneous finding MOST likely to be present in this infant
is
A. adenoma sebaceum
B. ash-leaf macule
C. forehead plaque
D. periungual fibroma
E. shagreen patch

27. A 6-year-old girl presents for evaluation of her shot stature. Chromosome
analysis reveals a 45,X karyotype that is consistent with Turner syndrome.
Of the following, the MOST appropriate evaluation to obtain is
A. echocardiography
B. hysterosalpingography
C. magnetic resonance imaging of the brain
D. pulmonary function tests
E. slitlamp ophthalmologic examination

28. A 2-year-old girl is referred for evaluation of global developmental delay


and the loss of developmental milestones. Physical examination reveals
hepatosplenomegaly and coarse facial features. Findings on computed
tomography of the head are unremarkable.
Of the following, the most appropriate evaluation to obtain NEXT is
A. chromosome analysis
B. serum thyroxine
C. TORCH titers
D. urine mucopolysaccharide levels
E. urine organic acids
29. In pediatric follow-up clinic, you are discussing various aspects of cerebral
palsy with parents.
Of the following, the MOST accurate statement is that
A. cerebral palsy is a progressive disorder of neuromotor function
B. complications of labor and delivery are the leading causes of cerebral palsy
C. most cases of cerebral palsy are diagnosed by 6 months of age
D. most children who have cerebral palsy have an associated seizure disorder
E. the prevalence of cerebral palsy has risen in the past two decades
30. An otherwise healthy 10-year-old girl is brought to your office for
evaluation of unilateral breast tenderness without galactorrhea. Physical
examination reveals bilateral breast buds that measure approximately 2 cm in
diameter. She has no axillary or pubic hair.
The Sexual Maturity Rating (Tanner) stage of this girl's tissue is stage
A. 1
B. 2
C. 3
D. 4
E. 5

31. A 4-year-old boy has a 1-month history of fevers to 39°C twice a day
accompanied by a rash. He feels well during his afebrile periods. Physical
examination reveals generalized lymphadenopathy.
Of the following, the MOST common other manifestation of this boy's illness
would be
A. iridocyclitis
B. headech
C. marked leukocytosis
D. presence of antinuclear antibodies
E. presence of rheumatoid factor

32. A 2-year-old girl presents with a 2-day history of cough, congestion, and
bilateral red eyes with an associated watery discharge. She denies any
photophobia or pain, and there is no edema of the eyelid.
The MOST likely diagnosis is
A. blepharitis
B. conjunctivitis
C. corneal abrasion
D. sclerouveitis
E. uveitis
33. A 3-year-old boy presents with a 3-day history of a sore throat,
temperature of 38.3°C, nasal congestion with cloudy rhinorrhea, and
conjunctivitis.
Of the following, a TRUE statement about acute pharyngitis is that

A. a negative streptococcal screening test reliably excludes streptococcal


infection and obviates the need for further assessment
B. all children who have signs and symptoms of pharyngitis should be treated
with antibiotics to prevent rheumatic fever
C. oral penicillin, 250 mg tid for 10 days, is appropriate treatment in most
cases
D. streptococcal pharyngitis can be differentiated from viral pharyngitis by
the presence of tonsillar exudate and cervical adenopathy
E. viruses are the cause in at least 85% of cases

34. As part of the examination of a 3-year-old girl during a health supervision


visit, you review her medical history.
Of the following, the condition that is the STRONGEST indication for routine
annual influenza immunization is
A. all healthy children
B. asthma
C. asymptomatic heart disease
D. attendance at child care
E. frequent otitis media

35. You are evaluating an ill-appearing 9-month-old child . Findings on physical


examination include a brassy cough, conjunctivitis, and coryza. The child has a
temperature of 39°C and a red, maculopapular rash on the head, trunk, and
proximal extremities.
Of the following, the complication that is MOST responsible for mortality in
young children who have this condition is
A. encephalitis
B. hemorrhagic shock
C. hepatitis
D. myocarditis
E. pneumonia
36. A 3-day-old term infant has cyanosis, no cardiac murmur, tachypnea, and
no retractions. Pulse oximetry reveals an oxygen saturation of 60% in the right
hand and 75% in the right foot.
Of the following, the MOST likely cause of the cyanosis is
A. group B streptococcal sepsis
B. persistent fetal circulation
C. pulmonary valve atresia
D. tetralogy of Fallot
E. transposition of the great vessels

37. ron deficiency in a child who has unrepaired cyanotic congenital heart
disease is MOST likely to increase the risk for
A. congestive heart failure
B. digoxin toxicity
C. osteoarthropathy
D. pulmonary hemorrhage
E. stroke

38. A 12-year-old girl develops low-grade fever and a nonproductive cough


following several days of sore throat and malaise. She appears tired, and
crackles are audible over the lower right lung field on chest auscultation. Chest
radiography reveals patchy alveolar infiltrates in the right lower and middle
lobes.
Of the following, the diagnostic test MOST likely to identify the etiology of this
illness is
A. blood culture
B. serology for Mycoplasma
C. sputum culture
D. urine antigen detection studies
E. viral culture of respiratory secretions

39. A 12-year-old girl experiences acute onset of weakness in the lower


extremities. On physical examination, the deep tendon reflexes are absent, and
results of the sensory examination are normal.
Of the following, the MOST likely findings on examination of cerebrospinal fluid
are
Glucose Protein Pressure
A. decreased normal normal
B. increased increased normal
C. normal decreased normal
D. normal increased normal
E .normal normal increased

40. An 8-year-old girl is evaluated for fever and vomiting of 2 days' duration.
Physical examination reveals a temperature of 39.9°C and blood pressure of
105/68 mm Hg; the remainder of the findings are normal. Results of laboratory
evaluation include a positive urine nitrite test, more than 100 white blood cells
per high-power field, and serum white blood cell count of 25 x 109/L
(25,000/mm3), with 4% bands and 80% neutrophils. A midstream clean catch
urine culture grows 10,000 to 100,000 CFU/mL of Escherichia coli.
Of the following, the MOST likely diagnosis in this child is
A. acute appendicitis
B. acute cystitis
C. acute pyelonephritis
D. renal abscess
E. viral gastroenteritis

41. The parents of a 5-year-old boy who has severe factor VIII deficiency
hemophilia bring him in for evaluation after he fell off his bicycle. He was not
wearing a helmet. He had no loss of consciousness and appears uninjured
except for a small hematoma over the right side of his forehead.
Of the following, the MOST appropriate next step is to
A. admit for observation without therapy
B. discharge home
C. infuse with factor VIII
D. obtain computed tomography
E. treat with desmopressin

42. You are examining a girl at her 1-year health supervision visit. Her weight,
length, and head circumference all were at the 10th percentile at birth. There
were no pregnancy, labor, delivery, or nursery complications. Physical
examination reveals her weight, length, and head circumference are at the 5th
percentile.
Of the following, this child's growth parameters MOST likely represent
a chromosomal abnormality
a malabsorptive disorder
an endocrine disorder
inadequate caloric intake
normal growth

43. A 1-day-old term infant develops bilious vomiting and poor feeding. You
recall that the differential diagnosis of vomiting is age-related.
Of the following, the condition that is MOST likely to cause bilious vomiting in
this infant is
gastric stress ulcer
gastroesophageal reflux
gastrointestinal food allergy
intussusception
midgut volvulus

44. An 8-year-old girl presents to the emergency department with a history of


vomiting, weight loss, and rapid breathing. Findings on physical examination
include lethargy, tachycardia, tachypnea, cool extremities, and pallor. Initial
laboratory evaluation reveals a venous pH of 7.10 and a blood glucose
concentration of 36 mmol/L (650 mg/dL). Urinalysis is positive for glucose and
ketones.
Of the following, the MOST immediate life-threatening complication of this
child's illness is
A. cerebral edema
B. hyperkalemia
C.hypocalcemia
D.hypoglycemia
E.hypovolemia

45. On the second day of hospitalization, a 5-year-old child who has


pneumococcal meningitis develops a serum sodium concentration of 120
mmol/L (120 mEq/L). Physical examination reveals an awake and responsive
child whose weight is 20 kg (an increase of 1 kg from admission), temperature is
37.8°C (100°F), blood pressure is 100/60 mm Hg, and pulse is 100 beats/min.
The MOST appropriate management is
A.administration of demeclocycline
B.infusion of a solution containing 5% glucose and 0.3% saline at a maintenance
rate
C.infusion of 80 mL 3% saline over 10 minutes
D.infusion of 0.9% saline at a maintenance rate
E.restriction of fluids
46. You are examining a 12-month-old girl whose is pale looking . Her mother is
concerned about lead. Of the following, the MOST appropriate test(s) to screen
this infant for lead poisoning is(are)
A.free erythrocyte protoporphyrin (FEP)
B.FEP and plasma lead level
C.FEP and wrist radiographs
D.plasma lead level
E.plasma lead level and wrist radiography

47. A 5-year-old girl develops fever, swelling of the parotid gland, and
headache.
Of the following, the BEST diagnostic test for this child is
A. bacterial culture of parotid duct secretions
B. Epstein-Barr virus serology
C. mumps serology
D. serum amylase
E. viral culture of respiratory secretions

48. The mother of a 3-year-old reports that her son has begun to cough and
have difficulty breathing. She was keeping him home from preschool today
because he had a cough and upper respiratory tract infection without fever.
The MOST likely cause of this child's symptoms is
A.aspiration pneumonia
B. bacterial pneumonia
C. foreign body aspiration
D.reactive airway disease
E.vascular ring

49. Compared with human milk, cow milk formula is MORE likely to contain a(n)
A. greater concentration of essential fatty acids
B. higher protein concentration
C. increased lactose content
D. lower calcium-phosphorus ratio
E. lower iron concentration.

50. A newborn is recognized clinically to have Down syndrome. The parents are
very concerned about the disorder and its manifestations.
Of the following, the MOST important hematologic/oncologic complication is
A. aplastic anemia
B. leukemia
C. macrocytic anemia
D. platelet dysfunction
E. thrombocytosis

51. A 10-year-old boy presents with headache. His father has renal failure. On
physical examination, the boy's blood pressure is 145/100 mm Hg and pulse is
90 beats/min. Urinalysis reveals 10 to 20 red blood cells per high-power field,
and serum creatinine level is 61.9 mcmol/L). Renal ultrasonography shows
enlarged kidneys, with three renal cysts in each. Of the following, the MOST
likely diagnosis in this child is
A. autosomal dominant polycystic kidney disease
B. autosomal recessive polycystic kidney disease
C. juvenile nephrophthisis
D. medullary sponge kidney
E. multicystic dysplastic kidney disease

RESPIRATION
1) All of the following concerning the differentiation of viral and bacterial
pneumonias are correct EXCEPT :-
a. the white blood cell count in a viral pneumonia is lower than in bacterial
pneumonia
b. viral pneumonia of the infants is always of a mild severity
c. the progression of a viral pneumonia is slower
d. a lobar infiltration is usually indicative of a bacterial origin

2) Symptoms of brochiectasis include all of the following EXCEPT :-


a. the recurrence of pneumonia affecting the same area
b. voluminous secretion of the airways
c. a loss of appetite , growth retardation & clubbing
d. cyanosis due to the circulatory abnormality

3) Recognized causes of pleural effusion include all EXCEPT :-


a. heart failure
b. pneumonia
c. nephrotic syndrome
d. none of the above

4)Recognized causes of chronic cough include :-


a. acute bronchitis
b. pneumonia
c. immunodefeciency
d. none of the above

5)All of the following is involved in the pathogenesis of asthma EXCEPT :-


a. hyperinflation
b. bronchospasm
c. excess mucous secretion
d. edema of the submucosa

6) Concerning bronchiolitis :-
a. most cases are in infants below 15 months
b. 25% of cases are due to respiratory syncytial virus
c. steroids are usually helpful
d. antiviral drugs are indicated in some cases

CARDIOLOGY
1) Which of the following would differentiate most clearly between a venous
hum & the murmur caused by a patent ductus :-
a. tha site of auscultation
b. auscultation during systole and diastole
c. a venous murmur is always soft
d. altering the paient's head position diminishes or accentuates the intensity of
the murmur

2) All of the following conditions are associated with a systolic murmur EXCEPT:-
a. anemia
b. aortic reguge
c. mitral incompetence
d. ventricular septal defect

3) Which of the following congenital valvular dis is associated with sever


cyanosis manifested during the first few days of life :-
a. PDA
b. the post ductal form of coarctation of the aorta
c. transposition of great arteries
d. tetralogy of Fallot
4) all of the following ststements concerning isolated ventricular septal defect
are correct EXCEPT :-
a. a left to right shunt become more sever wih time
b. decompansation might occur
c. signs of pulmonary congestion might occur
d. prophylaxis against endocarditis is not needed if asymptomatic and small
lesion

5) Which is the most common valvular abnormality with rheumatic fever :-


a. mitral insufficiency
b. mitral stenosis
c. aortic insufficiency
d. tricuspid insufficiency

6)All of the following ststements concerning rheumatic poly artheritis are


correct EXCEPT :-
a. it usually affect the great joints
b. joint deformaties develop if the appropriate ttt is not introduced in time
c. it is associated with a high fever
d. the affected joints are swollen, warm , and movements are painful

7) Symptoms of chorea minor include all of the following EXCEPT :-


a. impaired coordination
b. muscular hypotonia
c. increased reflexes
d . emotional lability

8) All of the following statements concerning the therapy of rheumatic fever is


correct EXCEPT :-
a. penicillin therapy is indicated
b. in case of carditis , prednisone is indicated for at least 6 weeks
c. ibuprofen is sufficient if the patient only exhibits polyartheritis symptoms
d. bed rest is indicated until complete remission

9) symptoms of subacute bacterial endocarditis include all of the following


EXCEPT :-
a. the initial symptoms include fatigue , pallor and loo of appetite
b. initially a moderate then a high grade fever is detcted
c. microembolization of the skin and kidneys
d. painful erythematous subcutaneous nodules about the tips of the digits

10) The effects of digitalis administration in a patient with cardiac


decompansation include all of the following EXCEPT :-
a. a declining elevated central venous pressure
b. hepatomegaly become less pronounced
c. the heart rate decreases
d. a short PR segment is observed on the ECG

NEUROLOGY
1)Which of the following is the most common cause of meningitis in infants and
children :-
a. meningococcus
b. enteroviruses
c. tuberculosis
d. haemophilus influenzae

2) All of the following statements concerning the Guillian Barre' syndrome are
correct EXCEPT :-
a. a symmetrical , flaccid pralysis and abnormality of sensation are characteristic
for the disease
b. the disease improves spontaneously despite its long clinical course
c. corticosteroids are the mainstay of ttt of acute cases
d. differentiation of the disease from poliomyelitis is difficult in some cases

3) Which of the following statements concerning Duchenne muscular dystrophy


is correct :-
a. it develops during the 3rd – 5th years and involves the lower extremities and
the pelvis
b. it affects boys and girls
c. the prognosis for survival is good
d. the condition is associated with decreased activities of creatine kinase and
aldolase in serum

4) All of the following statements about the epilepsy characterized by grand mal
(tonic- clonic ) seizures are correct EXCEPT :-
a. the seizures are characterized by flexion or extension positions of the lower
extremities
b. an interictal EEG is not sufficient for the diagnosis
c. phenytoin , Phenobarbital and valproate are the most important therapeutic
drugs
d. despite appropriate therapy , dementia develops in the majority of cases
5) Muscular hypotonia, without muscular weakness is characteristic for a.
Guillain-Barre' syndrome
b. Rickets
c. Down's syndrome
d. Werdnig-Hoffmann syndrome

6) Which of the following statements about absence epilepsy in childhood are


false :-
a. it is accompanied by a loss of consciousness for a few seconds
b. spike discharges with 3\s frequency in the EGG are typical
c. it responds well to drug therapy
d. it does not usually affect school performance

7) Which of the following condition is correctly paired with expected CSF


findings :-
a. tuberculous meningitis and high glucose with raised protein
b. Guillain-Barre' syndrome and normal cell count with reduced protein
c. viral meningitis and CSF lymphocytosis with normal glucose
d. bacterial meningitis and low glucose and protein

8) Spasticity may be associated with all of the following EXCEPT:-


a. clonus
b. clasp knife rigidity
c. lower motor neuron lesion
d. quadriplegia

9) Which of the following is true about cerebral palsy :-


a. it is a progressive motor deficit
b. the cause is usually a perinatal insult
c. mental retardation is a constant feature
d. should be considered if neonatal reflexes are absent beyond 6 months of age

10) What is the most common cause of obstructive hydrocephalus in an infant :-


a. Dandy-Walker syndrome
b. meningitis
c. Arnold-Chiari malformation
d. Aqueductal stenosis.

NUTRITION & NUTRITIONAL DISORDERS


1) Which of the following tests is not suitable to confirm the diagnosis of rickets
:-
a. the serum alkaline phosphatase activity
b. an x-ray picture of the wrist
c. the serum calcium level
d. the serum 25-OH-D3 level

2) All of the following abnormalities of body constitution and function can be


detected in nutritional marasmus EXCEPT :-
a. the subcutaneous adipose tissue of the face is lost first
b. a tendency for hypothermia is observed
c. hunger is usually observed
d. a more frequent occurance of pneumonia

3) Which of the following is an absolute contraindication of breast feeding :-


a. smoking
b. insulin therapy
c. mastitis
d. phenylketonuria

4) Which of the following is correct about weaning :-


a. it is usually started as early as 8 months of age
b. cow's milk is better avoided during 1st year of life
c. egg white can be given at about 7 months of age
d. weaning practice mean decreasing the amount of milk given

5) Symptoms of hypervitaminosis D include all of the following EXCEPT a.


polydypsia and polyuria
b. anorexia and constipation
c. hypocalcemia
d. calcification of tissues( kidney, vessel)

6)Possible causes of the development of rickets in a child regularly


supplemented with oral vit. D include all of the following EXCEPT :-
a. an abnormality in the metabolism of vitamin D in the skin
b. an abnormality in the metabolism of vitamin D in the kidney
c. lipid malabsorption
d. hyperphosphaturia

7) All of the following steps in the therapy of marasmic infants are correct
EXCEPT :-
a. feeding during night is important
b. large volumes of parenteral infusions should be avoided to prevent overload
on the atrophic myocardium
c. vitamins, K+ and folic acid should be supplemented
d. the marasmic infant should be given large amounts of food, primarily milk

8) What is the protein requirement of a 2 month old infant :-


a. 1 g/kg/day
b. 2.5 g/kg/day
c. 10 g/kg/day
d. 4.5 g/kg/day

GENETICS
1)Characteristic features of autosomal recessive inheritance include
a)50%risk of recurrence
b)both parents can be normal
c)more males are affected
d)consanguineous marriage is not a feature

2)One of the following anomaly is not associated with Down syndrome


a)mental retardation
b)congenital valvular heart disease
c)simian crease
d)webbing of neck

3)In case of autosomal dominant inheritance the inherited feature would be


manifested in one of the parents and in
a)50%of daughters and 75% of sons
b)25% of sonsand 75% of daughters
c)50% of sons and 50% of daughters
d)the daughters only

4)All of the following statements concerning sex linked recessive inheritance


pattern are correct except
a)the disease only affects boys
b)the father of the diseased sons might be affected
c)the diseased boys might have diseased grandfathers
d)not all of the daughters of a carrier mother will be carriers

5)The parents of a child who is suffering from a metabolic disease characterized


by an autosomal recessive inheritance pattern ask for your advice they intend to
have a second baby which of the following consideration is incorrect
a)the phenotype of 75% of the potential offspring will be normal
b)the genotype of 25% of the potential offspring will be normal
c)the genotype of 50% of the potential offspring will be normal
d)50% of the offspring will be heterozygous

Infection & Growth


1-Which of the following vaccines would be contraindicated in a 4 year old boy
receving immunosuppressive therapy for autoimmune hepatitis
a)Hepatitis A vaccine
b)Hepatitis B vaccine
c)Acellular pertussis vaccine
d)Varicella vaccine
2-Possible causes of fever of unknown origin include all of the following except
a)Pylonephritis
b)Subacute bacterial endocarditis
c)Hypothyroidism
d)Salmonella enteritis

3-How much is the head circumferance of a mature well-developed 6 month


infant
a)43 cm
b)38cm
c)46 cm
d)50 cm

4-Characteristic milestones in a 9 month old infant include


a)Can wave "bye-bye
b)Walks holding furniture
c)Sits unsupported
d)Can say "mama'dada
5-A 4-month-old baby boy born at term with 3,200 g birth weight,currently
weighs 6,000g What is your opinion
a)The infant is overweight
b)The body weight of the infant is normal
c)The infant's nourishment is deficient
d)The infant has a chronic illness

6-Possible complications of scarlet fever include all of the following EXCEPT


a)Submandibular lymphadenitis
b)Otitis media
c)Subacute sclerosing panencephalitis
d)Acute glomerulonephritis

7-Which of the following statements concerning Koplik's spots detectable in


measles is correct
a)They develop synchronously with the eruption on the skin
b)They are usually seen in the late phase of the catarrhal incubation period
c)They are most common during the convalescent phase
d)The cause is bacterial superinfection

8-Which of the following statements concerning varicella is correct


a)Desquamated crusts containing the virus can transmit the disease
b)School age children are routinely vaccinated
c)The patient should be bathed regularly , neutral powder is applied to relieve
itching
d)Oral acyclovir (Zovirax) therapy is indicated

9-Possible manifestations of mumps virus infection include all of the following


EXCEPT
a)Pancreatitis
b)Submandibular lymphadenitis
c)Orchitis
d)Meningoencephalitis

10-Possible causes of fever of non -infectious origin include all of the following
EXCEPT
a)Hyperthyroidism
b)Epilepsy
c)Systemic lupus erythematosus
d)A regulatory imbalance of the autonomic nervous system

Neonatology (M C Q)
1-WHICH OF THE FOLLOWING FACTORS LEADS TO NEONATAL
HYPERBILIRUBINEMIA:-
A-shortened neonatal red cell life span.
B-impaired excretion of unconjugated billirubin
C-limited conjugation of billirubin in liver
D-increased enterohepatic circulation
E-all of the above

2-WHICH OF THE FOLLOWING IS TRUE ABOUT RESPIRATORY DISTRESS


SYNDROME?
A-ground glass appearance on chest x-ray is common
b-the disease usually worsens 2-3 days after birth
C-steroids reduce severity if given early to premature
D-surfactants therapy is rarely useful

3-CHARACTERISTICS OF CEPHALOHEMATOMA INCLUDE ALL EXCEPT:-


A-subperiosteal
B-limited by margin of bones
C-resolves by 48 hours
D-soft tissue swelling

4-PERSISTENT MORO REFLEX AFTER 16 WEEKS INDICATES:-


A-normal infant
B-brain damage
C-hungry infant
d-none of the above

5-A NEONATAL CAUSE OF SEIZURES WHICH HAS THE BEST PROGNOSIS:-


A-birth asphyxia
B-bacterial meningitis
c- Hypoglycemia
D-hypocalcaemia

6-PHYSIOLOGICAL JAUNDICE USUALLY APPEARS ON …..DAY AND REGREE ON


…….DAY:-
A-first day and third day
B-fifth day and ninth day
C-third day and seventh day
D-seventh day and eleventh day

7-ALL OF THE FOLLOWING ARE EXPECTED IN INFANT OF DIABETIC MOTHERS


EXCEPT:-
a- large baby
B-hyperglycemia
C-hypocalcaemia
D-respiratory distress syndrome

8-AT BIRTH NORMAL HEART RATE:-


a-60-80/m
b-70-120/m
c-80-110/m
d-110-150/m

9-ALL OF THE FOLLOWING ARE SEEN DURING HYPOTHERMIA EXCEPT:-


A-bradycardia
B-excessive shivering
C-pallor
D-metabolic acidosis

10-WHICH OF THE FOLLOWING ARE CORRECT ABOUT NEONATAL INFECTIONS:-


A-it needs a high index of suspicion for diagnosis
B-umbilical infection is usually a minor problem
C-positive cultures are essential for diagnosis
D-respiratory distress is not an expected feature

HAEMATOLOGY
1) Which of the following is a typical symptom of Henoch-Scholein purpura :-
a. polyarthralgia
b. jaundice
c. a purpuric skin rush invoving the face
d. a prolonged bleeding time

2) During which childhood age is the onset of acute lymphoblastic leukemia


most frequent :-
a. 10-12 years
b. 6 months
c. aldoscence
d. 3-5 years

3) Bleeding due to thrombocytopenia occurs if the platelet count is less than :-


a. 150,000 / ul
b. 100,000 / ul
c. 80,000 / ul
d. 50,000 / ul

4) All of the following results are indicative for iron deficiency , EXCEPT a.
anemia
b. large, hypochromic RBCs observed in the peripheral blood smear
c. a low serum ferritin
d. an elevated total iron binding capacity (TIBC

5) All of the following diseases are associated with a decreased production of


RBCs ,EXCEPT :-
a. hepatitis
b. leukemia
c. hypothyroidism
d. ankylostoma

6) Which of the following interventions is not suitable for the therapy of ITP :-
a. glucocorticoid therapy
b. intravenus immunogloblin therapy
c. anabolic hormone therapy
d. plasmapheresis

7) All of the following laboratory findings are characteristic for ITP , EXCEPT :-
a. the platelet count is low
b. the prothrombin time (PT) , the partial thromboplastin time ( PTT) and the
thrombin time (TT) are all normal
c. the number of megakayocytes in the bone marrow is low
d. the WBC count is normal
8) Which of the following disorders may be associated with thrombocytopenic
purpura :-
a. meningitis
b. hypersplenism
c. Henoch-Scholein purpura
d. thrombasthenia

9) Which of the following statements is FALSE about anemias :-


a. a sever anemia causes dyspnea and tachycardia
b. cyanosis develops in sever anemia
c. a cardiac murmur may become audible in anemia
d. reticulocytosis ensures a hemolytic origin of anemia

10) Which of the following statements about Schonlein-Henoch purpura is


FALSE :-
a. swelling of the joints lasts for few days or weeks
b. the occurance of a microscopic hematuria suggests renal involvement
c. it may be associated with abdominal pain due to edema or hemorrhage of the
intestinal
d. the disease lasts for years even in uncomplicated cases

GIT
1- the minimal fluid requirement of a 6-months infant with mild diarrhea is :
a. 50 ml/kg
b. 75 ml/kg
c- 150 ml/kg
d. 300 ml/kg
2-what is the optimal initial therapy in case of moderately severe dehydration :
a. Transfusion of whole blood
b. infusion of half-isotonic sodium chloride –dextrose solution
c. infusion of 10 % dextrose solution
d. infusion of 0.9% sodium chloride solution

3- Disease in which a specific diet is indicated include all the following except :
a. post-enteric malabsorbtion
b. congenital sucrase-isomalatase deficiency
c. galactosemia
d. congenital hepatic fibrosis

4- Manifestations indicative of intussusception include all the following except :


a. A suddenly developing and recurring abdominal pain
b. the passage of non-feculent ,bloody mucus
c. during painful episodes , the infant looks flaccid & limp
d. the detection of fluid levels in abdominal X-ray

5- possible causes of hepatic cirrhosis includes all the following except a.


hepatitis A virus infection
b. chronic hepatitis
c. biliary atresia
d. hepatitis B virus infection

6- The therapy of hepatic cirrhosis include all the following except :


a. an energy-rich, fat free diet & vitamin substitution are necessary
b. the protein intake must be restricted & the intestinal ammonia production
should be inhibited
c. in case of esophageal bleeding an immediate surgical shunt operation is
needed
d. ascites formation may be decreased by a low salt intake & administration of
spironolactone
7- Which of the following statements about recurrent abdominal pain in
childhood is false :
a. the peak of incidence is at the age of 9-10 Years
b. the most common localization is the peri-umbilical area
c. the pain is usually vague & lasts few minutes
d. it's usually associated with rigidity or abdominal tenderness

8- Possible causes of diffuse enlargement of the abdomen include all the


following except:
a. Enteral infections
b. malabsorption syndrome
c. pyloric stenosis
d. hypokalemia

9- Whicj of the following is correct about stomatitis


a. herpangina is caused by herpes simplex infection
b. oral thrush causes small sized erythrmatous ulcers
c. herpetic stomatitis is usually afebrile
d. is a common problem in malnourished infants

10- features of hepatic cirrhosis include all the following except


a. cirrhotic nodules are usually palpable
b. bleeding from esophageal varices is not uncommon
c. the serum albumin concentration is decreased
d. ascites develops in the advanced stages

Nephrology
1- All the following concerning acute post-streptococcal glomerulonephritis are
correct,Except:
a. it can be prevented by the early antibiotic therapy of streptococcal infections
b. steroids treatment Is usually helpful
c. the prognosis of the disease in childhood is usually good
d. a hypertensive encephalopathy may occur
2- Possible complications of acute post-streptococcal glomerulonephritis
include all the following, Ecxcept:
a. hypermagnesemia
b. hyperkalemia
c. anuria
d. pulmonary edema

3- typical laboratory alterations in acute post-streptococcal glomerulonephritis


include all the following, Ecxcept:
a. the antistreptolysin titer in the serum is usually elevated
b. elevated serum creatinine & blood urea nitrogen levels
c. hematuria, proteinuria & granular casts in the urine
d. A normal increased C3 complement level in the serum

4- All the following statements concerning idiopathic nephritic syndrome are


correct, Except:
a. the onset is usually at 1-6 years of age
b. the history reveals a " minimal change" process
c. the early development of renal failure is characteristic
d. patients usually do not exhibit hypertension

5- findings usually detectable in acute renal failure include all the following ,
Except :
a. the serum creatinine in concentration is elevated
b. hyperkalemia
c. hyponatremia
d. hypophosphatemia
6- The average insensible water loss per day is :
a. 500 ml/kg
b. 500 ml/m2
c. 300 ml/kg
d. 300 ml/m2

ENDOCRINOLOGY
1)Which of the following laboratory data is possible to differentiate between
hypothalamic-pituitary secondary hypothyroidism & primary hypothyroidism?
A) A low serum T4 level & normal TSH levels
B) An elevated serum T4 level & low TSH levels
C) A low serum T4 level & elevated TSH levels
D) An elevated serum T4 level & elevated TSH levels

2)All of the following diseases cause short stature EXCEPT


A)Turner's syndrome
B)Hypothyroidism
C)Achondroplasia
D)Klinefelter's syndrome

3)All of the following statements concerning delayed puberty are correct


EXCEPT
A)Diagnosis is made if signs of puberty are not detectable after the age of 13 in
girls or 14 in boys
B)Majority of cases is constitutionally delayed puberty
C)An elevated testosterone level is usually found
D)A pituitary or hypothalamic tumor is a possible cause

4)All of the following statements concerning congenital hypothyroidism are


correct EXCEPT
A)None or very few physical symptoms are observed at birth
B)The serum TSH level is low
C)The appetite is decreased but the infant doesnot seem to be thin
D)The early introduction of therapeutic measures gives good results

5)Which of the following is the first step in the therapy of a diabetic ketoacidotic
coma?
A)Correction of the acidosis with NaHCO3
B)The subcutaneous administration of insulin
C)Fluid replacement with the infusion of a solution containing no glucose & the
administration of a rapidly acting insulin preparation intravenously
D)Fluid replacement with the infusion of a glucose –containing solution

6)Manifestations of diabetic ketoacidosis include all of the following EXCEPT


A)Sw
B)Vomiting
C)Respiratory distress
D)Abdominal pain

EMERGENCIES
1)Symptoms of hypovolemic shock include all of the following EXCEPT
A)Pallor & cold exteremities
B)Tachycardia
C)A low central venous pressure
D)Cardiac enlargement
E)Acidosis

2)Which of the following conditions associated with airway obstruction ,is NOT
life-thr?
A)Acute rhinopharyngitis
B)Acute epiglottitis
C)Laryngeal foreign body
D)Laryngeal oedema
E)None of the above

3)In which of the following intoxication is gastric lavage CONTRAINDICATED?


A)Narcotics
B)Hydrocarbons (gasoline , petroleum)
C)Barbiturates
D)Acetaminophen
E)Salicylates

4)All of the following are guidelines for the therapy of circulatory failure in a
child EXCEPT
A)The administration of oxygen
B)The administration of digitalis in all cases
C)Discontinuation of oral feeding , infusion of volume expander
D)Mechanical ventilation for respiratory insufficiency
E)The elimination of the negative inotropic factors (acidosis, hypoglycemia,etc.)

5)Possible causes of the development of a coma include all of the following ,


EXCEPT
A)Diabetic ketoacidosis
B)Cerebral edema
C)Phenobarbital intoxication
D)Encephalitis
E)Chorea minor

MCQ PAPER 1
1-Concerning bronchiolitis:
A. Epidemics often occur during winter.
B. A chest radiograph usually shows a poorly inflated chest.
C. Poor feeding is an indication for treatment with ribavirin.
D. The disease is more benign in infants born prematurely.
E. Respiratory syncytial virus is the most common causative virus.

2-The following are single gene disorders:


A. Cystic fibrosis.
B. Type 1 diabetes mellitus.
C. Duchenne muscular dystrophy.
D. Marfan syndrome.
E. Turner syndrome.

3-The risk of neonatal jaundice is increase by:


A. Prematurity.
B. Trisomy 21.
C. Elective caesarean section.
D. Congenital hypothyroidism.
E. Cephalahaematoma.

4-In the emergency treatment / rescusitation of infants:


A. Tension pneumothorax is diagnosed by chest x-ray.
B. Intraosseous needles must not be used under 1 year old.
C. Oxygen should be administered in anaphylaxis.
D. Cardiac arrest is usually secondary to hypoxia.
E. Cardiac compressions are contraindicated when the heart rate is 4
5-Recognised complications of measles include:
A. Encephalitis.
B. Febrile convulsions.
C. Diarrhoea.
D. Infertility in males.
E. Corneal ulceration.

6-Concerning asthma:
A. Inhaled steroids are useful in the treatment.
B. Wheeze on auscultation is pathognomonic.
C. It is seldom a cause of hospital admission.
D. Exacerbation of symptoms can occur with exercise.
E. Sufferers should be excluded from sport at school.

7-The following can be the cause of a non-blanching rash:


A. Idiopathic thrombocytopenic purpura.
B. Systemic lupus erythematosis.
C. Henoch-Schönlein purpura.
D. Haemolytic uraemic syndrome.
E. Meningococcal sepsis.

8-Regarding nocturnal enuresis:


A. Is more common in girls.
B. Can be a presenting feature of diabetes melitus.
C. Can be a presenting feature of diabetes insipidus.
D. Emotional disturbance can be a cause.
E. Desmopressin is used in the treatment.

9-Concerning childhood immunisations (in the UK):


A. BCG vaccine should be given to children with HIV.
B. A history of febrile convulsions is a contraindication.
C. Meningococcal C vaccine is routinely offered.
D. Polio vaccine is usually administered orally.
E. A fever Hib vaccine means futher Hib vaccinations are contraindicated.

10-The following congenital heart lesions are "cyanotic":


A. Ventricular septal defect.
B. Atrial septal defect.
C. Patent ductus arteriosus.
D. Tetralogy of Fallot.
E. Transposition of the great arteries.

MCQ PAPER 2
1-Concerning gastroenteritis (in the UK):
A. It can be fatal.
B. The most common cause is Salmonella spp. infection.
C. Broad-spectrum antibiotics are usually indicated.
D. A degree of lactose intolerance may follow gastroenteritis.
E. Blood pressure is a sensitive indicator of dehydration.

2-Skin peeling occurs in the following conditions:


A. Infectious mononucleosis.
B. Kawasaki disease.
C. Scarlet fever.
D. Toxic shock syndrome.
E. Erythema infectiosum (slapped cheek disease).
3-In neonatal RDS (respiratory distress syndrome):
A. Surfactant is useful in the treatment.
B. It is rare in infants below 28 weeks gestation.
C. Antenatal steroids are beneficial.
D. Maternal opiate abuse increases the risk.
E. Maternal diabetes increases the risk.

4-In children with cerebral palsy:


A. All have a degree of learning impairment.
B. Birth asphyxia is the most common cause.
C. Hand preference before 12 months can be a useful sign.
D. The spastic form is the most common clinical type.
E. Management should adopt a multidisciplinary approach.

5-The following conditions are haemaglobinopathies:


A. G6PD deficiency.
B. von Willebrand disease.
C. Spherocytosis.
D. Thalassaemia.
E. Sickle cell disease.
6-Theses neuromuscular disorders are "genetic diseases":
A. Duchenne muscular dystrophy.
B. Myotonic dystrophy.
C. Spinal muscular atrophy.
D. Guillain-Barre syndrome.
E. (Typical) Myasthenia gravis.

7-Concerning oncological disease in children:


A. Acute myeloid leukaemia is the most common leukaemia.
B. Most brain tumours are metastases from other primaries.
C. Wilm's tumour can present with haematuria.
D. Neuroblastoma can present in the first month of life.
E. Squint can be the presenting feature of retinoblastoma.

8-In pyloric stenosis:


A. Is more common in boys.
B. Surgery is the usual treatment.
C. Infants often have a metabolic acidosis.
D. Constipation can be a presenting feature.
E. A barium meal is usually performed to confirm the diagnosis.

9-The following can cause failure to thrive:


A. Coeliac disease.
B. Cystic fibrosis.
C. Emotional neglect.
D. Gastro-oesophageal reflux.
E. Patau syndrome.

10-The following are causes of polyhydramnios:


A. Maternal diabetes mellitus.
B. Potter syndrome.
C. Anencephaly.
D. Oesophageal atresia.
E. Polycystic kidneys.

MCQ PAPER 3
1-The following congenital abnormalities are deformations:
A. Coarctation of the aorta.
B. Haemangioma.
C. Talipes.
D. Cleft lip.
E. Imperforate anus.

2-Regarding anorexia nervosa:


A. Is never fatal.
B. Occurs exclusively in females.
C. Amenorrhoea can be a feature.
D. Growth hormone is typically elevated.
E. Distorted body image is common.
3-Systemic lupus erythematosus (SLE):
A. Is more common in females.
B. Psychosis is a recognized manifestation.
C. There is usually an associated thrombocytosis.
D. Is more common in Caucasians than Africans.
E. Autoantibodies against double-stranded DNA are frequently found.

4-In children with eczema:


A. Topical steroids are the first-line treatment.
B. IgE is raised in most cases.
C. Parental atopy is a positive risk factor.
D. In infants the extensor surfaces are mostly affected.
E. Herpes simplex can cause serious infection.

5-Concerning congenital pseudohypoparathyroidism:


A. Inheritance is autosomal dominant.
B. Short 4th metacarpals are characteristic.
C. Parathyroid hormone levels are usually increased.
D. Calcium levels are decreased.
E. Urine analysis is helpful in confirming the diagnosis.

6-The following are inherited in an X-linked manner:


A. Duchenne muscular dystrophy.
B. Haemophilia A.
C. Cystic fibrosis.
D. Sickele cell disease.
E. Hunter syndrome.

7-Concerning chickenpox:
A. The incubation period is 7 days.
B. Aspirin is a useful anti-pyretic.
C. Transmission is usually by contact or airborne.
D. Encephalitis is a recognized complication.
E. There is no effective vaccine.

8-Regarding Kawasaki's disease:


A. Conjunctivitis is a recognized feature.
B. It is caused by a spirochaete organism.
C. It is a cause of coronary artery aneurysms.
D. Fever is not a typical feature.
E. Is associated with profound thrombocytopaenia

9-The following cause a microcytic anaemia:


A. Ulcerative colitis.
B. Pernicious anaemia.
C. Methotrexate therapy.
D. Folate deficiency.
E. ß-Thalassaemia.

10-A normal two year old child would be expected to:


A. Drink from a cup.
B. Build a tower of 5 blocks.
C. Copy a circle.
D. Kick a ball.
E. Hop on one foot.

MCQ PAPER 4
1-Concerning brain tumours in children:
A. A sixth cranial nerve palsy can be the presenting feature.
B. The majority are metastatic from other tumors.
C. They are the most common solid organ tumor.
D. Brain stem glioma usually present with personality change.
E. Medulloblastomas can metastasize to the spine.

2-In depression in childhood:


A. A family history is common.
B. Suicide attempts are likely to be successful.
C. Sleep is typically impaired.
D. Is more common in pre-pubertal boys than pre-pubertal girls.
E. Antidepressant medication is ineffective.
3-Risk of developmental dysplasia of the hips is increased by:
A. Male sex.
B. Asian ethnic origin.
C. Positive family history.
D. Oligohydramnios.
E. Breech presentation.
4-The following exhibit autosomal dominant inheritance:
A. Gilbert syndrome.
B. Becker muscular dystrophy.
C. Turner syndrome.
D. Tuberous sclerosis.
E. Cystic fibrosis.
5-Compared with cow's milk, human breastmilk contains:
A. Less sodium.
B. Less calcium.
C. Less protein.
D. Less fat.
E. Less carbohydrate.
6-Regarding cystic fibrosis:
A. It is most common in sub-Saharan Africa.
B. The most common mutation is called delta-F508.
C. The responsible gene is located on chromosome 14.
D. Can present in neonates with intestinal obstruction.
E. Can be investigated by measuring sweat electrolytes.
7-An 8 month old child would be expected to:
A. Roll from back to front.
B. Sit unsupported.
C. Transfer objects between hands.
D. Build a tower of 3 blocks.
E. Have full head control.
8-The following are recognised causes of clubbing:
A. Cystic fibrosis.
B. Familial.
C. Bacterial endocarditis.
D. Fallot's tetralogy.
E. Ulcerative colitis.
9-The following are causes of short stature:
A. Klinefelter syndrome.
B. Marfan syndrome.
C. Emotional deprivation.
D. Hypothyroidism.
E. Achondroplasia.
10-Concerning systemic lupus erythematosus (SLE):
A. It causes a characteristic "butterfly rash".
B. Is more common in males.
C. Arthralgia is a rare feature.
D. Maternal SLE can cause heart block in the neonate.
E. Anti-nuclear antibodies are usually present.

MCQ PAPER 5
1-Concerning epilepsy in children:
A. There is an association with cerbral palsy.
B. Most forms of childhood epilepsy require lifelong treatment.
C. Infantile spasms may respond to corticosteroid treatment.
D. Consciousness is usually impaired in absence seizures.
E. A post-ictal hemi-paresis may occur.
2-The following are common findings in Trisomy 21:
A. Learning difficulties.
B. Protruding tongue.
C. Congenital heart defects.
D. Single transverse palmar crease.
E. Hypertonia.
3-Regarding rheumatic fever:
A. It occurs following a staphylococcal infection.
B. May present with chorea (a movement disorder).
C. Erythema marginatum is a "major" diagnostic criterion.
D. It requires treatment with immunoglobulin.
E. Renal impairment is the most serious long-term risk.

4-UK infants are routinely immunised against:


A. Meningococcus type B.
B. Diphtheria.
C. Cholera.
D. Polio.
E. Pertussis.
5-These neuromuscular disorders are inherited conditions:
A. Duchenne muscular dystrophy.
B. Becker muscular dystrophy.
C. Spinal muscular atrophy Type 1 (Werdnig-Hoffmann disease).
D. Guillain-Barre Syndrome.
E. Dystrophia myotonica.
6-Concerning ADHD (attention deficit hyperactivity disorder):
A. Impulsive behaviour is a feature.
B. Is associated with a higher rate of expulsion from school.
C. Is associated with a higher rate of teenage pregnancy.
D. Is associated with a higher rate of tic disorders.
E. May respond to treatment with stimulants such as amphetamines.
7-The following are causes of jaundice:
A. Prune belly syndrome.
B. Wilson's disease.
C. Pityriasis rosea.
D. Alagille Syndrome.
E. Alpha-1-antitrypsin deficiency.
8-The following drugs used in asthma are beta-agonists:
A. Salmeterol.
B. Salbutamol.
C. Ipratropium bromide.
D. Fluticasone.
E. Terbutaline.
9-Neonates in the UK are routinely screened for the following:
A. Hirschprung's disease.
B. Phenylketonuria.
C. Fragile X syndrome.
D. Hypothyroidism.
E. Congenital rubella infection.
10-Regarding the condition of patent ductus arteriosus (PDA):
A. Chest CT is the optimum radiological investigation.
B. May cause a "collapsing pulse".
C. May be treated by open surgical ligation.
D. May be treated medically using indomethacin.
E. May be treated by catheter trans-venous umbrella occlusion.

1) Defibrillation ( non-sync. shock) is used to treat:


a- pulseless ventricular tachycardia
b- hemodynamically unstable patient with Atrial flutter
c- hemodynamically unstable patient with Atrial fibrillation
d- hemodynamically unstable patient with ventricular tachycardia
e- none of the above

2) When you see the following ECG in cardiac arrest , the most appropriate
management is :
Defibrillation
Cardioversion (synchronized)
Give Atropine
Give Adenosine
Give lidocaine

3)The following ECG shows:


Ventricular fibrillation
Atrial fibrillation
Pulseless ventricular tachycardia
Extracystole
First degree heart block

4) Adrenaline 1: 10000 means


A) 1mg adrenaline in 1 ml H2O
B) 1mg adrenaline in 100ml H2O
C) 1g adrenaline in 100 ml H2O
D) 1g adrenaline in 1000ml H2O
E) 1g adrenaline in 10000ml H2O‫اعتقد‬

5) The following drugs can be given via Endotracheal tube EXCEPT


a- Adrenalin
b- Atropine
c- Lidocaine
d- Sodium bicarbonate
e- Diazepam
6) Which of the following maneuver is contraindicated by removing foreign
body from an infant ?
a. abdominal thrusts
b. finger sweep
c. chest thrusts
d. back blows
e. All of the above

7) Which statements about cardiac arrest is FALSE


Cardiac arrest is a clinical diagnosis
Any patient who is unconscious and who does not have a major pulse ( carotid
or femoral ) is in cardiac arrest
Some respiratory efforts such as gasping may persists for several minutes
Asystole is the most frequent initial rhythm documented in witnessed cardiac
arrest
The aetiology of cardiac arrest in infants and children is different from that in
adults.

8) Which cardiac rhythm is shockable ( can be treated with defibrillation )


A) Ventricular fibrillation
B) Pulseless Ventricular tachycardia
C) Asystole
D) PEA
E) A+ B

9) The most appropriate crystalloid used for resuscitation of hypovolemic


trauma patient is :
A- Ringer lactate
B- DW5%
C-0,45% Normal saline
D-Gelatines
E-DW 10%

10) A 4-year-old child was struck by a car traveling approximately 30 MPH and
was thrown approximately 15 feet. She is complaining of abdominal pain.
Physical exam reveals a blood pressure of 68/40 mmHg, a heart rate of 200
beats per minute, and a firm, tender abdomen. She weighs approximately 20 kg.
As long as the systolic blood pressure remains below 90 mmHg, what is the
most appropriate fluid management?
(A) 800 cc of normal saline followed by 400 cc of blood.
(B) 400 cc of normal saline followed by 200 cc of blood.
(C) 400 cc of normal saline repeated 3 times, then 100 cc of blood.
(D) 800 cc of normal saline repeated 3 times, then 400 cc of blood.
(E) 400 cc of normal saline repeated 3 times, then 200 cc of blood

11) A 2-year-old male is rushed in by his parents after being found unconscious
near open bottles of his grandfather's medications. The airway is secure, and
successful bag- valve ventilations are started. He is pulseless and unresponsive.
Intravenous access is being obtained. The cardiac monitor shows a wide
complex tachycardia at a rate of 260. What is the next step in management?
(A) Shock the patient with 200 J.
(B) Shock the patient with 0.5 J/kg.
(C) Shock the patient with 2 J/kg.
(D) Administer epinephrine 0.01 mg/kg IV.
(E) Administer atropine 0.01 mg/kg IV

12) Synchronized cardioversion shock is used for all of the following EXCEPT:
A- VF
B- VT
C- SVT
D- AT
E- AF

13) Which of the following actions helps deliver maximum current during
defibrillation?
A) Apply conductive paste to the paddles
B) Reduce the pressure used to push down on the defibrillator paddles
C) Decrease shock energy after the 2nd shock
D) Place alcohol pads between the paddles and skin
E) Use small paddles

14) The most appropriate method to open the airway in comatose patient with
suspected cervical injury is :
A) Jaw thrust
B) Head tilt
C) Head tilt ,chin lift
D) Extension of the neck
E) Flexion of the neck

15- Endotracheal intubation in advanced CPR :


A- airway management of choice .
B- best airway control .
C- once achieved ,allow continuous chest compression .
D- not essential as first step .
E- all of above are true .

16) Appropriate Endotracheal tube size for 8 y child is :


A) Size 3
B) Size 4
C) Size 5
D) Size 6
E) Size 7

17) Daily fluid maintenance for 30 kg child is approximately


1000 ml
1300ml
1700ml
2000ml
2300 ml

18)- Advantages of the laryngeal airway mask compared with ETT include all of
the following EXCEPT
A- Less tramatic
B- Incidence of aspiration is less than the endotracheal tube
C- No laryngoscope needed
D- Quick
E- May be used in emergency situation

19) The cricoid cartilage in an infant is located at the level of which cervical
vertebra?
A. C1.
B. C2.
C. C3.
D. C4.
E. C5.
20)Which drug can be used to treat this ECG ?
Lidocaine
Atropine
Adenosine
Adrenaline
All of the above

21) Normal blood volume for 3Kg Neonate is approximately :


A ) 180cc
B) 260 cc
C) 360cc
D)480 cc
E) 540 cc

22) Which statement about iv cannulation is FALSE


A) External jugular IV is indicated for critically ill patients < 12y old in whom an
extremity vein is not obtainable.
B) Intraosseous IV is indicated for critically ill patients < 6y old in whom an
extremity vein is not obtainable
C) Umbilical veins is indicated for new born <1week
D) Upper extremity IV sites are preferable to lower extremity sites .
E) The acellular fluids have the highest flow rates

23) Lactated Ringer contains all of the following EXCEPT


A) Potassium
B) Calcium
C) Sodium
D) Magnesium
E) Lactate

24)Which of the following causes of PEA is most likely to respond to immediate


treatment?
a. massive pulmonary embolism
b. hypovolemia
c. massive acute myocardial infarction
d. myocardial rupture and stroke
e. all of the above have the same response
1. Dental caries (cavities) in children are due to:
A. Metabolism of Streptococcus mutans
B. Enamel defects in teeth
C. Hereditary factors
D. Failure to brush teeth
E. Allof the above

2. The most common cause of pancreatic insufficiency in childhood is


A. Chronic pancreatitis
B. Congenital lipase deficiency
C. Cystic fibrosis
D. Shwachman-Diamond syndrome
E. Pearson syndrome

3. A 5-mo-old boy was clinically diagnosed with gastroesophageal reflux at 3 mo


of age. Treatment with cimetidine resolved his constant crying and even
seemed to improve his spitting. His weight continues in the 50th percentile. On
a follow-up visit, however, his parents report that for the past 2 wk he once
again has begun frequent regurgitation, hiccups, and irritability. He is breast-fed
and is given occasional supplements of cow’s milk-based formula. His
examination is normal. Which of the following is the most appropriate
intervention ?
A. Reassurance
B. Fundoplication
C. Upper endoscopy with biopsies
D. Adjustment of the dose of eimetidine
E. Daily use of antacids

4. A 6-yr-old girl with documented ?l-antitrypsin deficiency presents to the


emergency department for assessment of large-volume hematemesis. Physical
examination is remarkable for clear lung fields, pallor, and splenomegaly.
Hepatomegaly and petechiae are absent. Which of the following is the most
likely source of the hematemesis ?
A. Peptic ulcer disease
B. Thromboeytopenia secondary to hypersplenism
C. Swallowed blood from pulmonary hemorrhage
D. Esophageal variees
E. Erosive reflux esophagitis.
5. A 6-wk-old infant has a thoracotomy for transposition of the great vessels.
Following surgery, the infant develops loose stools. Stool cultures are negative
and the stool is Hemoccult-negative. Fecal pH is 6.0. The infant fails to gain
weight. Chest x-ray shows a pleural effusion. Which of the following would be
the most appropriate dietary intervention ?
A. Medium chain tryglycerides-predominant formula
B. Soy formula
C. Lactose-free formula
D. Carbohydrate-free formula
E. Long chain fat-predominant formula.

6. A 10-yr-old girl is brought for evaluation because her mother was found to
have familial adenomatous polyposis coli (APC). Genetic screening identified
identical APC gene mutations in mother, aunt, and grandfather, but no
mutations were identified in the girl’s APC gene. The girl requires :
A. Colonoseopy
B. Proetoeolectomy
C. Annual physical examination
D. CT scan
E. Repeat genetic testing in 1 yr.

7. A 4-yr-old boy presents with diarrhea and headaches. He appears to have a


limp, which favors the left leg. On examination, he has a poorly defined mass in
the left flank and a blood pressure of 170/100 mm Hg. Abdominal CT scan
demonstrates a large paraspinous mass on the left. The next step in the care of
this boy is to :
A. Start therapy with a long-acting somatostatin analogue
B. Refer for immediate surgical exploration
C. Start a-adrenergic blockade and obtain MRI scan
D. Start -adrenergic blockade and obtain spine MRI scan
E. Refer to physical therapy.

8. A chronic or recurrcnt course of abdominal pain, weight loss, and anemia in


an adolescent girl suggests
A. Henoeh-Sch nlein purpura
B. Inflammatory bowel disease
C. Anorexia nervos
D. Pelvic inflammatory disease
E. Irritable bowel disease.
9. All of the following statements regarding cyclic vomiting are true Except
A. Onset occurs between ages 3 and 5 yr
B. Episodes last 2-3 days
C. There are four or more emesis episodes per hour
D. It is a migraine equivalent
E. It is associated with nystagmus.

10. Wilson disease is associated with all of the following Except


A. Cardiomyopathy
B. Hepatomegaly
C. Aseites and portal hypertension
D. Dystonia and tremor
E. Hemolysis
F. Fanconi syndrome.

11. A 2.9-kg girl is born at term after having carried a prenatal diagnosis of left
congenital diaphragmatic hernia since 19 wk of gestation. Delivery and the early
postnatal period are uneventful, with mild tachypnea and retractions
developing at 24 hr of age. Chest film reveals a normal abdominal gas pattern
and multiple lucent areas in the left lower thorax. The most likely diagnosis is:
A. Congenital diaphragmatic hernia (Bochdalek
B. Congenital diaphragmatic hernia (Morgagni
C. Congential cystic adenomatoid malformation
D. Congenital diaphragmatic hernia (hiatal
E. Cystic fibrosis.

12 A 5-yr-old child is referred with a 6-wk history of morning headaches, often


associated with vomiting. His parents have noted that during this period he has
become irritable and moody. Which of the following would be most likely to be
identified during the physical examination ?
A. Marked elevation of blood pressure
B. Tenderness on percussion of frontal sinuses
C. Papilledema
D. Loud orbital bruit
E. Significant refractive error

13. A 6-yr-old child with neurofibromatosis (NFl) is found to have an optic


glioma on a routine MRI study, confined to the right optic nerve. Findings on the
neurologic, physical, and retinal examinations are normal. The visual acuity is
20/20 bilaterally, uncorrected. Which of the following is the correct
management ?
A. Surgical removal of the tumor
B. MRI of the optic nerve every 3 mo
C. Chemotherapy
D. Annual examination by a pediatric ophthalmologist.

14. A 2-yr-old boy with the spastic diplegia form of cerebral palsy is being
evaluated. MRI of his brain is most likely to show:
A.Multicystic encephalomalacia
B.Periventricular leukomalacia
C.Normal anatomy
D.Basal ganglia abnormalities
E.Agenesis of the corpus callosum

15. Which of the following therapies has been found to be effective in long-term
follow-up for some patients with the X-linked neurodegenerative disease
adrenoleukodystrophy ?
A.Vegetable oil as a dietary supplement
B.Hyperbaric oxygen therapy
C.Bone marrow transplantation
D.Total volume exchange blood transfusion
E.Plasmapheresis

16. A 5-yr-old child has a 6-mo history of increasing difficulty in walking


associated with urinary incontinence. Physical examination shows an alert child
with increased deep tendon reflexes, clonus and bilateral Babinski reflexes in
the lower extremities, and absent deep tendon reflexes in the upper extremities
associated with grade 3/5 weakness symmetrically in all extremities. The most
likely diagnosis is:
A.Myasthenia gravis
B.Spinal cord tumor
C.Guillain-Barré syndrome
D.Metachromatic leukodystrophy
E.Peripheral neuritis
17. A 15-yr-old boy has lost his ability to walk. On physical examination, his
ankle and knee deep tendon reflexes are noted to be diminished. The weakness
is greatest in peripheral muscles. Cranial nerves all are normal. One week before
these symptoms arose, he returned from a camping trip. The most likely
diagnosis is:
A.Myasthenia gravis
B.Organophosphate poisoning
C.Spinal muscular atrophy
D.Botulism
E.Tick paralysis
18. All of the following statements regarding congenital neuromuscular
disorders are true except:
A.Most are hereditary
B.Most are nonprogressive conditions
C.The definitive diagnosis is best made by electromyography
D.Diagnosis for some disorders may be confirmed by genetic
analyses of lymphocytes
E.Hypotonia is a common feature.

19. All of the following statements regarding Duchenne muscular dystrophy are
true except:
A.It is the most common hereditary neuromuscular disease
B.It is inherited as an autosomal dominant trait
C.Symptoms are rarely present at birth or in early infancy
D.The serum creatine kinase is consistently greatly elevated
E.It is more common in males

Conjugated hyperbilirubinemia is seen in: َََQ1 -


A. Gilbert’s syndrome
B. Griggler Najjar syndrome
C. Breast milk jaundice
D. Dubin Johnson syndrome

Q 2 - A 15-year-old female presented to the emergency department with history


of recurrent epistaxis, hematuria and hematochezia. There was a history of
profuse bleeding from the umbilicus stump at birth. Previous investigations
revealed normal prothrombin time, activated partial thromboplastin time,
thrombin time and fibrinogen levels. Her platelet counts as well as platelet
function tests were normal but urea c1ot Jt. positive. Which one of the
following clotting factor is most likely to be deficient?
A. Factor X
B. Factor XI
C. Factor XII
D. Factor XIII
Q 3 - Which one of the following is the characteristic feature of juvenile
myoclonic epilepsy?
A. Myoclonic seizures frequently occur in morning
B. Complete remission is common
C. Response to anticonvulsants is poor
D. Associated absence seizures are present in majority of patients

Q 4 - Plethoric lung fields are seen in all of the following conditions, except:
A. Atrial septal defect (ASD)
B. TAPVC (Total Anomalous Pulmonary venous connection)
C. Ebstein’s anomaly
D. Ventricular septal defect

Q 5 - Which of the following is an example of disorders of sex chromosomes?


A. Marfan’s syndrome
B. Testicular feminization syndrome
C. Klinefelter’ s syndrome
D. Down’s syndrome

Q 6 - Which of the following haemoglobin (Hb) estimation will be diagnostically


helpful in a case of beta thalassemia trait?
A. Hb-F B. Hb1C
C. Hb-A2 D. Hb-H
Ans. (C) Hb-A2

Q 7 - Which of the following circulating antibodies has the best sensitivity and
specificity for the diagnosis of celiac disease?
A. Anti-endomysial antibody
B. Anti-tissue transglutaminase antibody
C. Anti-gliadin antibody
D. Anti-reticulin antibody

Q 8 - A couple has two children affected with tuberous sclerosis. On detailed


clinical and laboratory evaluation (including molecular studies) both parents are
normal. Which one of the following explains the two affected children in this
family?
A. Non penetrance
B. Uniparental diasomy
C. Genomic imprinting
D. Germline mosaicism

Q 9 - Cardiomyopathy may be seen in all of the following except :


A. Duchenne muscular dystrophy
B. Friedreich’s ataxia
C. Type II glycogen storage disease
D. Alkaptonuria

Q 10 - Enzyme replacement therapy is available for which of the following


disorders?
A. Gaucher disease
B. Niemann Pick disease
C. Mucolipidosis
D. Metachromatic leukodystrophy

Q 11 - In a child with acute liver failure, the most important prognostic factor for
death is:
A. Increasing transaminases
B. Increasing bilirubin
C. Increasing prothrombin time
D. Gram negative sepsis

Q 12 - Which of the following does not establish a diagnosis of congenital CMV


infection in a neonate?
A. Urine culture of CMV
B. IgG CMV antibodies in blood
C. Intra-nuclear inclusion bodies in hepatocytes
D. CMV viral DNA in blood by polymerase chain reaction
Q 13 - All of the following are true of β thalassemia major, except:
A. Splenomegaly
B. Target cells on peripheral smear
C. Microcytic hypochromic anemia
D. Increased osmotic fragility
Q 14 - (Transient synovitis (toxic synovitis) of the hip is characterized by all of
the following, except:
A. May follow upper respiratory infection
B. ESR and white blood cell counts are usually normal
C. Ultrasound of the joint reveals widening of the joint space
D. The hip is typically held in adduction and internal rotation

Q 15 - Which of the following statements is true of primary grade IV-V


vesicoureteric reflux in young children?
A. Renal scarring usually begins in the midpolar regions
B. Postnatal scarring may occur even in the absence of urinary tract infections
C. Long-term outcome is comparable in patients treated with either antibiotic
prophylaxis or surgery
D. Oral amoxicillin is the choice antibiotic for prophylaxis

Q 16 - 15-year-old boy presented with one day history of bleeding gums,


subconjunctival bleed and purpuric rash. Investigations revealed the following
results:
Hb-6.4 gm/dL; TLC-26,500/mm3 Platelet-35,000/mm3; prathrombin time–20
sec with a control of 13 sec; partial thromboplastin time-50 sec; and Fibrinogen
10 mg/dL. Peripheral smear was suggestive of acute myeloblastic leukernice.
Which of the following is the most likely?
A. Myeloblastic leukemia without maturation
B. Myeloblastic leukemia with maturation
C. Promyelocytic leukemia
D. Myelomonscytic leukemia

The defective migration of neural crest cells results in: ََQ 17 -


A. Congenital megacolon
B. Albinism
C. Adrenogenital hypoplasia
D. Dentinogenesis imperfecta

Q 18 - A premature infant is born with a patent ductus arteriosus. Its closure can
be stimulated by administration of:
A. Prostaglandin analogue
B. Estrogen
C. Anti-estrogen compounds
D. Prostaglandin inhibitors

Q 19 - The loading dose of Aminophylline is:


A. 50-75 ug/kg
B. 0.5-1.0 mg/kg
C. 2.0-3.5 mg/kg
D. 5-6 mg/kg

Q 20 - Cushing’s Triad includes all except:


A. Hypertension
B. Bradycardia
C. Hypothermia
D. Irregular respiration

Q 21 - All of the following drugs are used for managing status epilepticus except:
A. Phenytoin
B. Diazepam
C. Thiopentone sodium
D. Carbamazepine

Q 22 - Administration of glucose solution is prescribed for all of the following


situations except:
A. Neonates
B. Child of a diabetic mother
C. History of unconsciousness
D. History of hypoglycemia

Q 23 - Which organ is the primary site of hematopoiesis in the fetus before


midpregnancy?
A. Bone
B. Liver
C. Spleen
D. Lung

Q 24 - All of the following are the complications in the new born of a diabetic
mother except:
A. Hyper bilirubinemia
B. Hyperglycemia
C. Hypocalcemia
D. Hypomagnesemia

Q 25 - A 3-year-old boy presents with fever, dysuria and gross hematuria.


Physical examination shows a prominent suprapubic area which is dull to
percussion. Urinalysis reveals red blood cells but no proteinuria. Which of the
following is the most likely diagnosis?
A. Acute glomerulonephritis
B. Urinary tract infection
C. Posterior urethral valves
D. Teratoma

Neonatology
Respiratory distress in newborn babies is recognized complication of all of the
following Except:
a- Group B streptococcal infection.
b- Congenital heart disease.
c- Sickle cell anaemia.
d- Cerebral disorders.
e- Diaphragmatic hernia.
An infant born at 32 weeks gestation is at increased risk from all of the following
Except:
a- Meconium aspiration.
b- Respiratory distress syndrome.
c- Hypoglycemia.
d- Hypocalcemia.
e- Intraventricular heamorrhage.

Infants of diabetic mothers are at increased risk from all of the following except:
A. Hypocalcemia.
B. Anaemia.
C. Jaundice.
D. Shoulder dystocia.
E. Congenital abnormalities.

Infants of diabetic mothers are at increased risk from all of the following except:
a- Hypoglycemia.
b-Hypocalcemia.
c-Intrauterine growth retardation.
d-Hypomagnesemia.
e-Hyperbilirubinemia.

The following features are not typical of physiological jaundice except:


a- Recognizable jaundice in the first 48 hours.
b- Peak plasma bilirubin at four to five days.
c- Persistence beyond first week.
d- Irritability.
e- Pale stools.

Jaundice is most likely to be physiologic in a term infant, in which of the


following situations.
A. Jaundice at 12 hours of age.
B. Serum bilirubin level increasing less than 5mg\dl\day in the first 2-4 days
C. Direct serum bilirubin greater than 1 mg\dl.
D. Jaundice at 12 days of age.
E. Usually associated with anaemia.

7- Jaundice appearing on day 1 of life suggested all of the following except


A. Torch infection.
B. Erythroblastosis fetalis.
C. ABO incompatability.
D. Sepsis.
E. Intrahepatic biliary obstruction.

8- Hyperbilirubinemia at 2 weeks of age suggests all of the following except:


A. Physiologic jaundice.
B. Hypothyrodism.
C. Pyloric stenosis.
D. Biliary atresia.
E. Breast milk jaundice.

9- Which of the following is not a cause of apnea in infants..


A. Prematurely.
B. Hyperglycemia.
C. Severe hypoxemia.
D. Intraventricular hemorrhage.
E. Sepsis.

10- All of the following are included in evaluating ABGAR score except:
A. Heart rate.
B. Respiratory effort.
C. Muscle tone.
D. Maturity of the newborn.
E. Response to catheter in nostrils.
11- Which on of the following is not utilized in determining an APGAR score.
A. Colour.
B. Heart rate.
C. Deep tendon reflexes.
D. Grimace.
E. Respiration.

12-Factors that leads to higher incidence of hyaline membrane disease includes


of the following except:
A. Multiple pregnancy.
B. Cesarean section.
C. Intrauterine hypoxia.
D. Postmaturity.
E. Maternal diabetes.

13-Which of the non pulmonary causes of respiratory distress in the newborn.


A. Choanal atresia.
B. Neonatal pneumonia.
C. Congestive heart failure.
D. Lung collapse.
E. Hyaline membrane disease.

14- Regarding meconium,all of the following are true except:


A. More than90% of newborns pass meconium in the first 24h.
B. Meconium plug may cause intestinal obstruction.
C. It is composed of epithelial cells, bile and mucus.
D. Meconium stained amniotic fluid is a sign of intrauterine distress.
E. Meconium aspiration can lead to hyaline membrane disease.

in hyaline membrane disease all of the following is correct except.


A. is more common in babies of diabetic mothers.
B. Is due to surfactant deficiency.
C. It is more common in premature neonates .
D. Occurs most commonly at first few hours post delivery.
E. Always requires ventilation

CVs & RH. Fever


all of the following could be manifestation of large VSD Except:
A. feeding difficulties.
B. Poor growth.
C. Cardiomegally.
D. Recurrent chest infection.
E. Absent femoral pulse.

Teratology fallot consists of all of the following Except:


a- pulmonary stensie.
b- Ventricular sephal defect.
c- Aortic coarctation
d- Rt. Ventricular hypertrophy.
e- over-riding aorta.

Which of the following is not characteristic of cyanotic heart disease:


A. Elevated blood pressure in arms.
B. Polycythemia.
C. Digital clubbing.
D. Dyspnea on exertion.
E. Limited exercise tolerance.

Clinical manifestations of rheumatic fever include all of the following Except:


A. fever, usually present in early stage.
B. Arthritis characteristically involves small joints and painless.
C. Carditis occurs in about 40% of patients.
D. Erythemia morginatum in the associated skin rash.
E. Chorea.

The white strawberry tongue is associated with:


A. Scarlet fever.
B. Varicella.
C. Rubella.
D. Rubeolla
E. Diphteria.

Each of the following may be a common manifestations of mumps Except:


A. orchilis.
B. Pancreatitis.
C. Parotitis.
D. Bilateral arthritis.
E. Asception meningitis.

Teratology of fallot is characterized by all Except:


A. obstruction of RH. Out flow.
B. VSD.
C. Dextroposition of Aorta.
D. Rt. Ventricular hypertrophy.
E. Osticm secundum defect.

All of the following are regarded as major manifestations of rheumatic fever


Except of:
A. Carditis.
B. Prolonged P-R interval.
C. Erythema marginatum.
D. Subcutaneous nodules.
E. Chorea.

Which are of the following finding is commonly seen in patient with Rt. to left
shunt:
A. cyanosis.
B. Clubbing of the fingers and toes.
C. Seundery polylythemia.
D. All of the above.
E. Non of the above.
Which of the following is not a clinical feature or recognized complication of
Tetra-logy of fallot:
A. Anoxic Spells.
B. Brain abscens.
C. Congestive heart failure.
D. Cyanosis.
E. Poor growth.
A child who skips, name four color and dresses and undresses is how old:
A. 15 months.
B. 24 months.
C. 30 months.
D. 18 months.
E. 60 months.(3y 3 color-4y 8color)

The ability to manipulate a small objects with the pincer grasp is La capacité de
manoeuvrer de petits objets avec la prise de pincer est habituellement notée à
quel âge ? usually noted at what age?
A. 0-2 months.
B. 3-5 months.
C. 6-7 months.‫األقرب إل الصحيح‬
D. 8-9 months.
E. 10-12 months.

A developmentally normal child who is just able to sit without support, transfer
objects from hand to hand, and speak in monosyllabic babble, is probably what
age:
A. 3 months.
B. 4 months.
C. 9 months.
D. 6 months.‫األقرب إل الصحيح‬
E. 11 months.

Neurological, intellectual, and physical development in infants and children


occur in an orderly and sequential manner. Al the following are integers of
developmental milestones Except:
A. gross motor.
B. Fine motor.
C. Bone age.
D. Language.
E. Social.

Child at 12 months of age can do all of the following Except:


A. Say baba and mama.
B. Can walk with or without support.
C. Copy circle.
D. Move bye-bye.
E. Play simple ball game.

A 4 month old infant can:


A. hold her head reasonably steady in sitting position.
B. Site a lone without support.
C. Say mama and dada but in discriminately.
D. Sam mama and dada appropriately.
E. Point to object that she wants.

A6 month old infant differs from a 3 months old in regard to his or her ability to:
A. control bowel and bladder.
B. Crawl
C. Sit or almost sit, without support.
D. Simile socially.
E. Walk holding an to furniture.

Most infant loss weight immediately after birth, normal term infants generally
regain their birth, weight by:
A. 24 hours.
B. 48 hours.
C. 72 hours.
D. 3-5 days. (3-4 days loss weight )
E. 7-10 days. (1 week – 10 days regain their birth, weight )

At birth, the skull "Cranium":


A. Is larger than the face.
B. Is smaller than the face.
C. Cranium and face are equal in size.
D. May be larger or smaller than the face.
E. Is usually flattened anteriorly.

A weight gain of about 1.2kg during the first 3 months of life is:
A. About average.
B. Above average but within normal limits.
C. Below average but within normal limits.
D. Below average and probably abnormal.
E. Above average and probably abnormal.

The anterior fontanel usually feels closed on physical examination "palpation":


A. by 3 months.
B. Between 3 and 9 months.
C. Between 9 and 18 months.
D. Between 18 and 24 months.
E. Between 24 and 36 months.

During the first years of life on infant who weighted (3.4kg) at birth ordinarily
would gain about:
A. 2.3kg.
B. 4.5kg.
C. 6.8kg.
D. 9kg.
E. 11.4kg.
During the second year of life, the average weight gain is about:
A. 1.5kg.
B. 3kg. (2kg)
C. 5kg.
D. 8kg.
E. 12kg.

During the first year of life, the average gain in body length is about:
A. 12.7cm.
B. 25cm
C. 38cm
D. 51cm
E. 64cm.

The normal average of hemoglobin concentration of I year of age is about:


A. 17mg/dl.
B. 15mg/dl
C. 12mg/dl (12.5mg/dl)
D. 10mg/dl
8mg/dl

The average head circumference of a term infant at birth is about:


A. 25cm
B. 30cm
C. 35cm
D. 40cm
E. 50cm

During the first month of life, head circumference grows about:


A. ½ cm
B. 1.2cm
C. 2.5cm
D. 5cm
E. 7.5cm
The average growth in head circumference during the first year of life is about:
A. 4cm
B. 12cm
C. 25cm
D. 37cm
E. 50cm

During the second year of life the average gain in body length is about:
A. 12 to 15cm.
B. 20 to 25cm.
C. 30 to 40cm.
D. 40 to 50cm.
E. Over 50cm.

One year old child would be able to do all of the following Except:
A. build a tower of three cubes.
B. Grasp a pellet
C. Reach for an object
D. Stand a lone
E. Transferee an object from hand to hand.

By age of two years the child can speak:


A. 3 word sentences.
B. Four word sentences.
C. Five word sentences.
D. Pronoun I and you.
Six word sentences.

A child can hop-on-and foot by age of:


A. one year.
B. Two year.
C. One year and half.
D. Two years and half.
E. Four years.
A child triple their birth weight by age of:
A. one year.
B. Two year.
C. Sex months.
D. Three months.
E. Nine months.

Rickets
Craniotabes could be seen in all of the following condition Except:
A. Rickets.
B. Vitamin E deficiency.
C. Hydrocephalus.
D. Syphilis.
E. Premature baby.

Physical features of vitamin D-deficient rickets included all of the following


Except:
A. Bitot Spots.
B. Craniotabes.
C. Enlagement of the costochondral junction.
D. Thickening of the ankles and wrists.
E. Large anterin fontranell.
F. Bowed legs.

All of the following are true about the clinical manifestations of kwashiorkor
Except:
A. the presence of edema.
B. Rash in sun exposed area.
C. Hair changes.
D. Weak muscles.
E. An increased susceptibility to infection.

The recommended daily dietary allowance of vitamin D per young infant is:
A. 100 I.U
B. 400 I.U
C. 800 I.U
D. 1600 I.U.
E. 4000 I.U.
Which of the following Vitamin are Fat soluble:
A, D and C.
D, K and B1
A, D, K and E.
C, D, K and E
B1, K and E.

Supplementation with which of the following Vitamins is most important for an


exclusively breast feed infant:
A. Vitamin A
B. Vitamin E
C. Vitamin C
D. Vitamin B1
E. Vitamin D

All of the following are clinical signs of rickets Except:


A. Craniotabes.
B. Enlargement of the costochordral junction.
C. Thickening of wrists and ankles.
D. Poor growth.
E. Conjunctivitis.

Which of the following statement concerning Vitamin D metabolism is false:


A. it is Activated to cholecalciferol in the skin.
B. It requires bile for absorption.
C. The kidney is active in its metabolism.
D. It circulates in plasma as 25OH cholecalciferol.
E. It is stored in the liver but not metabolized there.

In the absence of Vitamin D, Serum calcium may be maintained by:


A. Parathyroid hormone secretion.
B. Decreased renal excretion of phosphate.
C. Small dietary increases.
D. Decreased renal excretion of alkali.
E. Increased amounts of Vitamin A in diet.

Clinical disorders associated with increased incidence of Vitamin D deficiency


include all of the following Except:
A. cystic fibrosis.
B. Hepatic disease.
C. Celiac disease.
D. Chronic anticonvulsant therapy.
E. Obesity.

Rickets may be treated by all of the following Except:


A. 1500 to 5000 I.U of Vitamin D daily for 2 to 4 weeks.
B. Sun light plus 1500-5000 I.U of Vitamin D daily until heading demonstrated
on x-ray.
C. Sun lights.
D. 6000 I.U of Vitamin D as single dose.
E. ↑Ca+2 in diet and ↓phosphate.

In infant rickets all of the following is correct Except:


A. craniotabes is an early finding.
B. Serum calcium is usually normal.
C. Muscle are a atrophied.
D. Low serum phosphoric.
E. Rosary.

All of the following are regarding Vitamin D Except:


A. the required prophylactic dose is 400I.U one day.
B. Vitamin D is metabolized in liver and kidney.
C. Causes muscle weakness when deficient.
D. Causes limb pain when deficient.
E. Excessive dose is safe.

a 4 month old with Vitamin D-deficient rickets. Would be expected to show all
of the following Except:
A. Craniotabes.
B. Bow Legs.
C. Rosary.
D. Low Serum phosphate level.
E. High Alkaline phosphatase level

Nephrology
All of the following are consistent with the diagnosis of idiopathic nephrotic
syndrome of child hood Except:
A. Onset is usually between 2 and 6 years of age.
B. Pathologic renal changes are minimal by light microscopy.
C. C3 level is depressed.
D. Hypertension is unusual.
E. Hyperlipidemea.

In nephrotic syndrome all of the following is correct Except:


A. Edema is massive.
B. Ascites may be present.
C. Total serum globulin is diminished.
D. massive proteinuria.
E. Steroid are the drug of choice for treated.

In acute post streptococcal nephritis all of the following is correct Except:


A. Smoky urine.
B. Usually present with massive edema.
C. Specific gravity of urine is high.
D. Hypertension.
E. Penicillin may be of value in patient management.

Concerning urinary tract infection in children all of the following is correct


Except:
A. Usually diagnosed if you find baeilliuria of 10000/mm2 or more.
B. Initial symptoms may be systemic (fever, abdominal pain).
C. Treatment should be continued for at least 2 week.
D. Found smelling is not a common presenting complaint.
E. Common causative organism are gran –ve organism.

All of the following statement are correct about routine urine examination,
Except:
A. Specific gravity of 1015 is normal.
B. Red blood cell cast is normal finding.
C. One red blood cell is normal finding.
D. One white blood cell is normal finding.
E. Negative sugar is normal finding.

Of the following, the most reliable for the diagnosis of urinary tract infection:
A. Fever and loin pain.
B. A numerous WBCs in the urine analysis.
C. Bacteria seen in the urine analysis.
D. The presence of a single isolated of < 105 /ml colory in urine.
E. Dysuria and frequency.

A child with chronic renal failure is expected to develop all of the following
Except:
A. Abnormal linear growth.
B. Hypophosphatemia.
C. Hypertension.
D. Hyperkalemia.
E. Rickets.
In minimal lesion nephrotic syndrome all are true Except:
A. There is proteinuria without gross hematuria.
B. Serum BUN and Creatinine are normal.
C. Ascites could be present.(pu etre)
D. Hypertension is constant finding.
E. Hepatomegally may be present.
Nephrotic syndrome is characterized by:
A. Proteinuria.
B. Hypoproteinemia.
C. Edema.
D. Hyperlipidemia.
E. All of the above.

Minimal change nephrotic syndrome is characterized by the following Except:


A. Edema.
B. Albuminurea.
C. Hypercholestrolemia.
D. Hypoproteinemia.
E. Hypertension.

Classical acute nephritis in children, characterized by all of following Except:


A. Hypertension.
B. Macroscopic hematuria.
C. Follows streptococcal infection.
D. Necessitates fluid restriction.
E. Corticosteroid are the first line of therapy.

Proteinuria is usually found in all of the following Except:


A. Nephrotic syndrome.
B. Iron deficiency anemia.
C. Acute glomonal nephritis.
D. Renal tumor.
E. Orthostatic.

The commonest cause of nephritis syndrome in child (3) years old is:
A. Idiopathic.
B. B-streptococcal group A infection.
C. System lupus erythematous.
D. Insulin dependent diabetic mellitus.
E. Urinary tract infection.

14- which are of the following statements regarding hematuria is not


true
a- If casts are present , the source of hematuria must be the kidney.
b- Bright red urine that clots usually suggests renal, or upper
urinary tract source of bleeding.
c- The addition finding of proteinuria usually suggests a renal
source.
d- The most common neoplasm associated with hematuria is
wilm's tumor.
e- It can be a cause of anemia.

Which of the following statement is true of post streptococcal


glomerulonephortic:
A. More than 10% of children develop chronic renal failure.
B. Hypertensive encephalopathy is recognized complication
C. ASO Titer is not useful marker of streptococcal infection.
D. Life long penicillin prophylactic is recommended.
E. Abnormalities of serum complement usually persist for more than 3
months.

Post streptococcal acute glamerulonephritis is associated with all of the


following Except:
A. oliguriea..
B. fall in C3 level.
C. Granular and red cell cast.
D. Smoky urine.
E. Polyurea.
Respiratory System
Infections with mycoplasma pneumonia can produce each of the following
EXCEPT:
A. Guillain – barré syndrome .
B. Lobar and bronchopneumonia.
C. Tracheobronclitis.
D. Otitis media .
E. Glomerlonephritis.

The drug of choice for treatment of mycoplasma pneumonia infection in 4 years


old child is :
A. Cefuroxime .
B. Chlorayphenicol.
C. Erythromycin .
D. Penicillin.
E. Tetracycline .

All of the following are not associated with acute Bronchiolitis EXCEPT:
A. Usually associated with High fever and rashes .
B. Usually associated with bilateral infiltrates on chest tray.
C. Commonly associated with retractions, tachypena and wheezing .
D. Characterized by the absence of cough despite respiratory distress
E. Most common between 2 and 5 years of age .

Acute epiglottises is not associated with all of the following EXCEPT:


A. Gradual onset of cough , fever and stridor over several days.
B. Infection with Para influenza virus .
C. Hemophilia influenza type B septicemia .
D. Easy in swallowing .
E. High probability of recurrence .

All of the following statements are true of acute laryngeo tracheobronclitis


(group)EXCEPT:
A. Barking cough and hoarse voice are early symptoms .
B. Boys are more likely to be admitted to hospitals than girls
C. The symptoms frequently respond to salbutamol .
D. The use of nebulized adrenaline is indicated.
E. Para influenza virus are the predominant etiological agents .
Empyema is associated with all of the following Pathogens Except:
A. Staphylococcus aureus.
G. Influenza type B.
B. Group A streptococcus.
C. Pneumococcus.
D. Mycoplasma.

Viral Bronchiolitis is characterized by all of the Following except:


A. Decreased in functional residual capacity.
B. Air trapping on the chest X- RAY.
C. Hypoxia.
D. Inflammation and partial occlusion of bronchioles.
E. Wheezing and tachypnea.

Characteristic features of pneumococcal pneumonia includes All of the following


Except:
A. Meningism.
B. Delirium.
C. Petecheal rash.
D. Hypochondral pain.
E. Fever.

Persistent stridor is caused by all of the following Except:


A. Vocal cord paralysis.
B. Laryngeomalacia.
C. Tracheal hemangioma.
D. Tracheomalacia.
E. Vitamine D deficiency.

The following are related to asthma Except:


A. IS more prevalent in boys.
B. Is more strongly associated with house dust mite than any Any other
identified allergen.
C. Nocturnal cough.
D. Excercised induced wheeze.
E. Pectus exacavatum ( funnel chest ).
The following infectious agents are generally recognized as causing pneumonia
Except:
A. Influenza viurus.
B. EcHo virus.
C. Mycoplasma pneumonia.
D. Streptococcus pneumonia.
E. Chlamydia trachomitis.

The following steps are appropriate in the management of Status asthmaticus


Except:
A. Humidified oxygen.
B. Prescription of corticosteroid therapy.
C. Prescription of inhaled beta adrenergic agonist.
D. Prescription of aminophylline intravenious.
E. Prescription of sedative to restless child.

In contrast to acute epiglottitis, acute laryngeo tracheo-Bronchitis has all of the


following features Except: .
A. A more insidious onset.
B. The etiologic agent is almost always viral.
C. A slower course.
D. The etiologic agent is almost always bacterial.
E. The patient is less likely to require intubation.

All of the following drugs could be used in controlling Acute attacks of bronchial
asthma Except:
A. Adrenaline.
B. Ketotifen (zaditin).
C. Ephidren sulfate.
D. Aminophylline.
E. Salbutamol inhaler.

The virus most commonly associated with the clinical Picture of acute
Bronchiolitis in infants and children is
A. Adeno virus.
B. Respiratory syncytial virus.
C. Rhino virus.
D. Coxsackie virus.
E. Herpes virus type 2.
16- In pneumonia consolidation all of the following is correct except:
a- there is Impaired percussion note.
b- Vocal resonanace is increased.
c- Type of breathing is harsh vesicular.
d- Mediastinum is central.
e- Chest pain may be a presenting symptom.

17- All of the following are not related to acute bronchiolitits except
a- Commonly caused by bacterial infections.
b- Associated with retractions, tachypnea and wheezing.
c- Physical examination reveals extensive signs of consolidation.
d- X-Ray reveals pneumatoceles.
e- Best treatment is broncho-dilator.

18- In staphylococcal pneumonia all of the following is correct except


a- Occurs more frequently in infant.
b- Empyema and pneumothorax are commonly seen .
c- No respond well to crystalline penicullive .
d- Therapy is usually required for one week only.
e- X-Ray reveals pneumatoceles.

19- In pleural effusion the following signs are elicited except.


a- Impaired percussion note.
b- Shifted Mediastinum .
c- Diminished breathing on the affected side.
d- Respiratory movements on the affected side are increase.
e- Respiratory movements on the affected side are decreased.

20- Physical signs of pulmonary collapse include all of the following except
a- Impaired percussion note.
b- Diminished breath sounds.
c- Shifted mediastinum.
d- Increased tactile vocal fremitus.
e- Decreased tactile vocal fremitus.

21- Stridor in the newborn may be caused by all of the following except
a- Congenital Goiter .
b- Vescular anomalies are rings.
c- Larynogo malacia .
d- Aspiration pneumonia .
e- Hypocalcemia.

22- All of the following are characteristic of Jaundice associated with breast
feeding except.
a- Significant elevations of unconjugated bilirubin .
b- A rapid fall in serum bilirubin after discontinuation of nursing.
c- Nursing can be resumed after several days without return of
hyperbilirubinemia.
d- Significant elevations of conjugated bilirubin.
e- Kernicterus has never been reported to occur as result of breast milk
jaundice.

Fever &rash
1-Compications of measles may include.
a- Encephalomyelitis.
d- Laryngitis.
c- Giant cell pneumonia.
d- Thrombocytopenic purpura.
e-All of the above.

2-All of the following are associated with meningococcal meningitis except:


a- Abrupt onset with malaise, headache and irritability.
b- Projectile vomiting and prostration.
c- Chills and convulsions.
d- Cherry-red spot of macula.
e- Petechial or purpuric skin lesion.

3-Incubation period of meningococcal meningitits is


a-7-10 days.
b-2-3 days.
c-8-15 days.
d- 3 weeks.
e-10-20 days.

4-Infection with which one of the following organisms does not require isolation
measures.
a- Diphtheria.
b-Chicken pox.
c- Pertussis.
d- Mumps.
e- Brucella.

5-Infections with or complication of streptococcal infection include all of the


following except: .
a- Scartet fever.
b- Pneumonia.
c- Endocarditits
d- Erysipelase.
e-Scalded skin syndrome.

6-All of the following diseases are water borne except:


a- Cholera.
b- Bacillary dysentery.
c- Amoebic dysentery.
d- Tetanus.
e- Hepatitis A.
7-Common manifestations of congenital rubella syndrome includes all of the
following except:
a- Intra uterine growth retardation.
b- Hydrocephalus.
c- Congenital heart lesion.
d- Cataract.
e-Mental retardation.

the rash of chicken pox is characterized by all of the following except:


Incubation period is 10-16 days.
Rapidly progresses from papules to vesicles.
Lesions appear in crops.
Greatest concentration of lesion is on extremities.
Vesicles develop on mucus membrane of mouth and genitalia.

9-In measles infection the following are characteristic except:


a- Incubation period is 10 days.
b-Rash appears on the second day.
c-The disease is infectious before the appearance of the rash.
d-Koplik spot is common.
e- The infant is immune against measles for the first 4-6 ms of life.

10-The rash in measles is .


a- Papulo vesicular.
b- Urticarial.
c- Maculo popular.
d- Appears on the first day of illness.
e-Last 24 to 48 hours.

11-which of the following is least characteristic of varicella infection.


a-an incubation period of 14-16 days.
b- A rash progressing from papular to vesicular.
c- A tendency for the rash to be heaviest on the arms and legs.
d- Temperature that is usually less than 38,9 c.
e- Itiching.

12- maternal infection and viremia with rubella virus during the early weeks of
pregnancy may result in any of the following except:
a- Congenital malformation.
b- Stillbirth.
c- Abortion.
d- Macrosomia and large for gestational age in infant.
e- Chronic infection of the fetus.

13-which of the following is not characteristic of rubella infection in young


children.
a- Mild leucopenia.
b- Post auricular and occipital lymph nodes.
c- Four to five days of fever before the rash.
d- A pink red maculopapular rash.
e- Mild thrombocytopenia.

14-clinical manifestations of scarlet fever includes all of the following except:


a- Exudative tonsillitis.
b- White strawberry tongue.
c- Hepatosplenomegally.
d- Erythemato punctiform lesions on the palate.
e- Red strawberry tongue.
15-the rash of scarlet fever is characterized by all of the following except:
a- A rash sand paper- like texture.
b-A tendency to desequmate.
c-A tendency to become generalized with in 24 hours.
d-A tendency to spare area of skin folds (axilla,groin).
e-A red,erythematous appearance.

16-the three most common bacterial causes of meningitis in childhood are.


a-Neisseria meningitides,hemophylus influenza,pneumococcus.
b-Neisseria meningitides, pneumococcus, E.coli.
c-Neisseria meningitides, group A streptococcus,staphylococcus.
d-Neisseria meningitides, hemophylus influenza, L.monocytogens.
e-Hemophylus influenza, group A streptococci, E.coli.

17- which of the following is seen most commonly as a complication of gastro


intestinal shigella infection.
a- A generalized erythematous rash.
b- Arthritis.
c- Encepallopathy.
d- endocarditis.
e- Pneumonia.

18-All of the following are common complications of measles except:


a- Otitis media.
b- Pneumonia.
c- Arthritis.
d- Laryngeotracheitis.
E- Encephalitis.

Which is most likely to occur as an isolated manifestation of acute Rheumatic


fever:
A. Arthritis.
B. Carditis
C. Chorea.
D. Erythemia Marginatum
E. Fever.

The most common roentgeno-graphic abnormality in a child with asthma is:


A. Bronchiectasis.
B. Generalized hyperinflation.
C. Lower lobe Infiltration.
D. Pneumomediastinum.
E. Right middle lobe atcleclosis.

Congenital Hypothyrodism should be included in the differential diagnosis of


newborn with:
A. Coma.
B. Prolonged Jaundice.
C. Pulmonary edema.
D. Renal failure.
E. Sever anemia.

Ordinarily, the first dose of live, attenuated measles vaccine (as MMR) should
be administered:
A. At about 3 months of age.
B. At 6 to 9 months of age.
C. At 15 months of age.
D. At about 24 months of age.
E. At the time of school entry.

A side effect of tetracycline that is limited to children is:


A. pseudotumor cerebri.
B. Hyperglycemia.
C. Tremors and convulsions.
D. Permanent discoloration of teeth. (les dents)
E. Hematuria and proteinuria.

The arthritis of acute rheumatic fever usually:


A. Is monoarticular.
B. Heals without deformity. ( ‫شفاء بدون‬
C. Appears after the fever subsides.
D. Is seen only in patients with concurrent Carditis.
E. Involves large and small joint equally.
The most common presentation of pinworm infection is:
A. Appendicitis.
B. Diarrhea.
C. Intussusceptions.
D. Perianal pruritis.
E. Vaginitis.
All of the following are features of trisomy 21 (down):expt
A. hypotonia.
B. Microcephaly.
C. Simian crease on palms.
D. Recurrent chest infection.
E. Normal development.
Causes of failure to thrive include all of the following Except:
A. Emotional deprivation.
B. Intestinal mal-absorption.
C. Renal tubular disorders.
D. Chronic heart failure.
E. Acute chest infection.

Pneumococcal infection are more frequent and more sever in all of the
following condition Except:
A. Immunodeficiency.
B. Post splenectomy.
C. sickle cell disease.
D. Congenital heart disease.
E. Formula fed infants.

All of the following are signs of lower motor neuron disease Except::
A. Fibrillation.
B. Fasciculation.
C. Spasticity.
D. Muscle wasting.
E. Hyporeflexea.

All of the following are sings of congenital hypothyrodism Except:


A. Prolonged jaundice.
B. Hypertonic muscle tone.
C. Constipation.
D. Umbilical hernia.
E. Dry skin and hoarse cry.

Convulsions in the newborn could be caused by all of the following Except:


A. Hypokalemia.
B. Hypoglycemia.
C. Meningitis.
D. intracranial hemorrhage.
E. Hypoxia.

Acute glomerulonephrities may be associated with which are of the following:


A. Glucosuria.
B. Elevated serum cholesterol.
C. Normal or elevated C3 complement.
D. Encephalopathy.
E. Hypotension.

Pallor, jaundice, hemosiderosis and extrmedallary hematopoiesis are most


chareteristic of which of the following:
A. Sickle cell
B. Thalasemia Major.
C. Iron deficiency anemia.
D. Hemoglobin- H disease.
E. Hemoglobin- C disease.

Kwashiorkor is characterized by each of the following Except:


A. A history of inadequate protein intake with relatively adequate calories.
B. A history of prolonged breast feeding.
C. Edema.
D. Hypoalbuminemia.
E. Marked vulnerability to death from inter-current enteric infection.

Manifestation of Cretinism include all of the following Except:


A. Puffiness of face.
B. Mental retardalion.
C. Diarrhea.
D. Prolonged physiological jaundice.
E. Dry skin.

Dehydration is classified into isotonic, hypotonia, hypertonic according to are of


the following:
A. Serum potassium.
B. Serum sodium level.
C. PH of Blood.
D. Blood Glucose.
E. Serum Calcium.

ll of the following are common in bronchial asthma Except:


A. Generalized wheezing.
B. Increased eosinophilis count.
C. Parotysmal nature of the attacks paroxysmal.
D. Gross hemoptysis.
E. Dyspnea.

A child which "chicken pox" is liable to all Except:


A. Lymphadenitis.
B. Encephalitis.
C. Orchitis.
D. Bronchitis.
E. Gastroenteritis.

Kwashiorkor is characterized by all of the following Except:


A. Mental apathy.
B. Hypervitaminosis A.
C. Flag sign.
D. Skin manifestation.
E. Edema.
An infant of diabetic mother is liable to all of the following Except:
A. Hyperglycemia.
B. Hypocalemia.
C. Respiratory distress.
D. Birth trauma.
E. Hyperbilirubinemia.

Arthritis in childhood occurs in all of the following Except:


A. Acute Rheumatic fever.
B. Sickle cell disease.
C. Acute leukemia.
D. Hemolytic Uremia syndrome.
E. Rheumatoid arthritis.

In Rheumatic fever all of the following is correct Except:


A. Is associated with painful arthritis.
B. Is associated with prolonged P-R interval.
C. Dose not recur.
D. Is frequently the result of streptococcal throat infection.
E. Has a peak incidence between 5-15 years.

The following statement regarding mumps are correct except:


A. Infection is spread by droplet infection.
B. Incubation period is 16-18 days.
C. The Virus can be isolated from Urine.
D. Epididymo-orthitis is commoner in young boys.
E. Parotid duct is red and pointed.

In Rickets all of the following is correct except:


A. Craniotabes is a late sign.
B. Delay closure of fontanel occurs.
C. There may be normal calcium level.
D. There is characteristic cupping of the distal ends of the radus and ulna.
E. High level of Alkaline Phosphates enzyme.
Characteristics of henoch-schonlein purpura include all of the following except:
A. A low platelet count.
B. Skin lesions which initially blanch on pressure.
C. Normal complement level.
D. Normal thrombin time and partial thromboplastin time.
E. Skin biopsy help in diagnosis.

In Patient ductus arteriosus all of the following is correct except:


A. Usually closes functionally with in 10-15hours of birth.
B. That is larg may result in Eisenmenger syndrome.
C. Closes following a prostaglandin Infusion.
D. Persists in pulmonary atresia due to arterial hypoxemia.
E. That is small produces a continuous murmur in the pulmonary area below
the left clavicle.

Bloody stools are a recognized clinical feature in children affected by all of the
following except:
A. Vero toxin producing strains of Escherichocoli.
B. Giardia lamblea.
C. Cow's milk protein intolerance.
D. Ascaris lumbriocoides.
E. Shigelliosis.

The following is live attenuated vaccine except:


A. Measles, Mumps and rubella. (MMR)
B. Poliomyelitis "Sabin".
C. Rubella.
D. B.C.G.
E. Pertussuss.

The most common ophthalmologic complication in children with juvenile


arthritis is:
A. Cataract.
B. Ptosis.
C. Glaucoma.
D. Corneal ulceration.
E. Iridocyclitis.

Concerning hypovitaminosis, all of the following statement are correct except:


A. Vitamin A deficiency lead to blindness.
B. Vitamin B "Thiamin" deficiency leads to peripheral neuropathy.
C. Vitamin C deficiency leads to bleeding.
D. Vitamin K deficiency leads to bleeding.
E. Vitamin B12 deficiency leads to Microcytic anemia.

Bone age is best evaluated by doing X-ray of the following:


A. Elbow Joint.
B. Wrist.
C. Ankle.
D. Knee Joint.
E. Hip Joint.

All of the following are advantages of breast feeding except:


A. Reduced incidence of allergy.
B. Reduced incidence of otitis media.
C. Reduced incidence of colic.
D. Increased psychologic comfort.
E. Vitamin K content.

Clinical features of Vitamin E deficiency include all of the following except:


A. Cereballer ataxia.
B. Muscle weakness.
C. Peripheral neuropathy.
D. Hemolysis.
E. Hepatosplenomegally.

Teething is associated with all of the following except:


A. drooling of saliva.
B. Gingival swelling.
C. Irritability.
D. Rash.
E. Relief with acetominophin.

Manifestation of hyperkalemia include all of the following except:


A. Paraesthesia.
B. Weakness.
C. Paralysis.
D. Wide QRS complex.
E. Tetany.

Congenital infections may be associated with all of the following Except:


A. Growth retardation in utero.
B. Polycythemia.
C. Thrombocytopenia.
D. Hepatosplenomegaly.
E. Skin Rash.

At what age meningitis cannot be excluded with confidence in children


presenting with febrile convulsion:
A. Fiver years.
B. Four years.
C. Three years.
D. Nine months.
E. all age.

Patients with sever cerebral palsy usually die because of:


A. Urinary tract infections.
B. Chest infections.
C. Meningitis.
D. Heart failure.
E. Hepatic failure.

All of the following are complication of chronic renal failure except:


A. Hyperkalemia.
B. Alkalosis.
C. Anemia.
D. Hypertension.
E. Renal Osteodystrophy.

Which of the following types of vaccines is not efficient in administered below


the age of two years:
A. Injectable polio vaccine (Salk).
B. Measles vaccine.
C. Hepatitis B vaccine.
D. Pneumo vax.
E. Hemophyllus Type B conjugated vaccine.

In general all of the following vaccine must avoided in child with


immunodeficiency except:
A. Measles vaccine.
B. Mumps vaccine.
C. Oral polio vaccine (Sabin).
D. Influenza Vaccine.
E. Pertasiss vaccine.

All of the following are true about the laboratory manifestation of kwashiorkor
except:
A. Persistent ketonuria.
B. Hypoalbuminemia.
C. Hypoglycemia.
D. Potassium deficiency.
E. Low serum amylase levels.

Which of the following infection typically has an incubation period of less than 2
weeks:
A. Mumps. 12-28
B. Varicella.
C. Rubella. 12-21
D. Measles .07-14
E. Rabies.

When the mother contact german Measles "Rubella" early in pregnancy, effects
on the fetus may include all of the following except:
A. Thrombocytosis. (Thrombocytopenia)
B. Cataract.
C. Glaucoma.
D. PDA.
E. Hepatosplenomegally.

Which of the following is contraindication to L.P. in a 9 months old infant with


suspected meningitis:
A. Uncontrolled bleeding diathesis.
B. Bulging fontanel.
C. L.P. 2 days before.
D. Marked in cooperativeness on the part of patient.
E. Significantly elevated WBCs consistent with bacteremia.

Hypokalemia is likely to occur in all of the following except:


A. Diarrhea.
B. Renal tubular acidosis.
C. Infantile Pyloric stenosis.
D. Diuretic therapy.
E. Infants of diabetic mother.

Which of the following is characteristic in Guillan-Barre Syndrome:


A. Female are affected more often than male.
B. Sensory symptoms are more striking than paralysis.
C. Muscles wasting in usual.
D. No craineat nerve involvement.
E. Sinus tachycardia is common in acute phase.
Signs of dehydration in acute gastro-enteritis include all of the following except:
A. Sunken eyes.
B. Oliguria.
C. Dry tongue and Mucus Membrane.
D. Bulging anterior fontanel.
E. Loss of skin elasticity.
1- Regarding hyaline membrane disease HMD all of the following are true
EXCEPT?
C) It may be exacerbated by hypoxia ,acidosis ,hypothermia
D) It occurs in 45%of children born before 28 weeks of completed gestation
E) Steroids given anternatally to the mother stimulate surfactant production

2- All of the following are recognized causes of recurrent cough in the pre-
school child ?
A) Parainfluenzae
B) Exercise
C) Retropharynageal abscess
D) Congenital ciliary dysmotility
E) Otitis media

3- Acute Laryngotracheobronchitis ( Croup) all of the following are true EXCEPT?


A)Usually cause by viruses ,particularly Para influenza viruses

4-Wheeze in infancy associated with


A) Is strongly associated with bronchial hyperreactivity
B) Is indicative of chronic respiratory symptoms needing inhaled corticosteroids

5- Acute bronchiolitis all of the following are true EXCEPT?


A) Is rare outside infancy
B) Is usually cause Para influenza viruses
C) May be

6- The following respiratory symptoms may be excited by GER


A) Asthma
B) Central apnea
E) All of the above

7- All of the following are complication of Foreign Body Airway Obstruction


EXCEPT?
B) Broncholectasis
C) Angioneurotic
D) Hyperinflation of the effect lung

8-Which of the following statements are true of childhood asthma EXCEPT?


A) Over 90% of Pt show exercise induced bronchoconstriction
B) Hypercapnia occurring late and is a particularly worrying sign in status
asthmaticus
C) Infants are unresponsive to bronchodilato
D)Spontaneous cure occurs before adolescence
E) Cough may be the only symptom

9- Recognized causes of acute upper airway obstruction all of the following


EXCEPT?
D) Laryngomalacia

10- All of the following are signs of severe asthma EXCEPT?


D) Decreased right sided breath sounds in 10 year old girl
E) A respiratory rate

11- All of the following symptoms are associated with respiratory tract
abnormalities EXCEPT?
A) Charge syndrome
B) Ciliary dyskinesi
C) Kartageners syndrome
D) Tay sachs disease
E) Cridu-chat syndrome

Emergycy
1. Laryngomalacia :
A. is due to abnormally complaint laryngeal cartilage.
B. is characterized by stridor which develops around 2 years of age .
C. causes lifelong symptoms in the majority of cases.
D. require endoscopic confirmation, even in mild cases.
E. requires tracheostomy in around 20% of cases.

2. Sudden infant death syndrome:


A. affects 1in 200 infants under 1 year of age.
B. may be prevented by prone lying.
C. is commoner in the winter months.
D. has double the risk of recurrence in sibling born after the index case.
E. is often caused by inhalation of vomit.

3-Complications of bacterial meningitis include all of the following except:


A-Pleural effusion.
B-Convulsions.
C-Cerebral abscess.
D-Hydrocephalus.
E-Mental retardation.

3. Recognised complications of the nephrotic syndrome include all except :


A. pyogenic infection
B. pulmonary oedema
C. urinary tract infection
D. acute tubular necrosis
E. thrombo-embolism
4-Idiopathic nephrotic syndrome in childhood characteristically true except :
A recurs after initial and apparently successful treatment
B occurs following viral infections
C responds to steroid treatment
D responds to cytotoxic therapy (e.g. cyclophosphomide,chlorambucil)
E more frequent after puberty

5. In anaphylactic shock, all of the following therapeutic approaches are


appropriate EXCEPT:
A. volume resuscitation
B. epinephrine infusion
C. isoproterenol infusion
D. diphenhydramine administration
E. oxygen administration

6. You are called to the emergency department to evaluate a child with fever,
purpura, and a heart rate of 180. Your first priority in the resuscitation is to:
A. determine blood pressure
B. establish an IV line
C. administer ceftriaxone
D. supply supplemental oxygen
E. perform endotracheal intubation

7. Acute, severe iron intoxication is best managed with:


A. oral deferoxamine
B. induced emesis
C. saline lavage
D. activated charcoal
E. none of the above

8. A 1-week-old infant has evidence of cardiogenic shock. Chest radiography


demonstrates a greatly enlarged cardiac shadow with evidence of pulmonary
edema.Of the following, the MOST appropriate initial management to reverse
shock in this patient is intravenous administration of:
A) ampicillin and gentamicin
B) calcium
C) digoxin
D) dopamine
E) epinephrine

9. A 10-year-old girl is undergoing a routine physical examination.


Of the following, the MOST appropriate blood pressure for this patient would
be:
A) 115/65 mm Hg
B) 115/80 mm Hg
C) 125/65 mm Hg
D) 125/80 mm Hg
E) 135/65 mm Hg

10. A 2-yr old boy has fever, irritability and stiff neck, he was treated with oral
Amoxicillin for otitis media, a true statement about effect of previous Antibiotic
therapy on the ability to diagnose meningitis is it:
A. Interferes with reliability of counter immunoelectrophoresis
B. Decrease likelihood of diagnosing H. influenza type B. meningitis.
C. Can delay definitive diagnosis.
D. Decrease concentration of glucose in C.S.F.

11. A 6-week-old boy has had progressively worsening projectile vomiting over
the past 2 weeks. Physical examination reveals a wasted, dehydrated infant who
is avidly sucking on a nipple.Of the following, the laboratory values (in mEq/L)
MOST consistent with this child 's Problem would be:
Sodium Potassium Chloride Bicarbonate
160 3.0 125 A) 12
140 3.8 110 B) 23
138 2.5 110 C) 23
130 2.8 80 D) 35
123 3.2 93 E) 10
12. All of the following can be used acutely to lower serum K levels during
hyperkalemia EXCEPT:
A. glucose with insulin intravenously
B. albuterol aerosol
C. sodium bicarbonate infusion
D. captopril
E. all of the above

13. The finding of marked metabolic alkalosis with acidic urine indicates:
A. marked sodium depletion
B. marked potassium depletion
C. hyperventilation
D. diabetes mellitus
E. laboratory error

14. A 10-year-old trauma victim with eye opening in response to pain,


inappropriate words in response to verbal stimuli, and extension in response to
pain has a Glasgow Coma Scale score of:
A. 5
B. 7
C. 9
D. 10
E. 11

15. A 3-month-old infant who has a heart murmur has tachypnea and diffuse
expiratory wheezing. Chest radiography reveals an enlarged heart with
increased pulmonary vascular markings.Of the following, the MOST likely cause
of this infant's wheezing is
A) aspiration of a foreign body
B)bronchiolitis
C) congestive heart failure
D)pneumonia
E)vascular ring

16. A 3-month-old child who has Haemophilus influenzae type b meningitis is


admitted to the hospital. Of the following, the MOST essential part of care prior
to discharge is to
A) obtain magnetic resonance imaging of the head
B)order brainstem auditory evoked response testing
C)perform a developmental screening test
D) refer for a physical therapy assessment
E) request a neurologic consultation

17-Management of metabolic disturbances include all EXCEPT.


a- Hydration
b- Alkalinization
c- Potassium chloride supplementation.
d- Allopurinol in oral dose 300-600mg/M²/ d in 3 divided doses.
e- IV infusion of Ca gluconate 10% solution.

18- In what setting is activated charcoal not advised EXCEPT:


a- Caustics.
b- Late presentation of pure acetaminophen ingestion.
c- Clinical ileus, hematemesis or severe vomiting.
d- Kerosene ingestion.
e- Carbamazepine and tricyclic antidepressants.

19- All these ingestions or exposures have antidote in front EXCPT:


a- Warfarin Vit K.
b- Metchlorpramide Valium.
c- Methanol Ethanol.
d- acetaminophen N- acetylcysteine .
e- Isoniazid Pyridoxine.

20- All these are features of lead toxicity EXCEPT:


a- Vague abdominal complains, such as anorexia and vomiting.
b- Vague behavioral effects such as hyperactivity.
c- Ataxia telagenectesia.
d- Basophilic stippling.
e- A history of unexplained anemia.

21. In managing an unconscious child :


A. identifying the cause of the coma takes priority over other aspects of care.
B. endotracheal intubation is contraindicated if there is no gag reflex.
C. hypoglycemia is diagnostic of insulin overdose.
D. ingestion of drugs is an important cause to consider at any age.
E. a lumbar puncture should be performed promptly .
Infection
1. A 5-year-old girl with asymptomatic HIV infection is being seen for a
preschool evaluation.Of the following, the vaccine that is CONTRAINDICATED is
A. conjugated Haemophilus influenzae type b
B. diphtheria-tetanus toxoids with pertussis
C. measles-mumps-rubella
D. oral poliovirus
E. split virus influenza

2.All are correct for measles EXCEPT :


A.The incubation period is 8-12 days.
B.Infants are protected for at least 4 months after birth.
C.Koplik spots are pathognomonic for the disease.
D.Immunoglobulins are protective if given within 3 days of exposure.
E.Measles vaccine is contraindicated in exposed children.

3. A 4-year-old boy develops fever, swelling of the parotid gland, and


headache.Of the following, which complication is MOST likely to occur in this
patient?
A. Conjunctivitis
B. Deafness
C. Meningitis
D. Myocarditis
E. Orchitis

4.The following is correct for Rubella:


A. Incubation period is 10 days.
B. Patient is infectious for 10 days after onset of rash.
C. Infants with congenital rubella secretes the virus in urine for a year.
D. Transplacental antibodies are not protective.
E. Immunoglobulins has no role in protection.

5.All of the following are correct causative organisms for the corresponding
disease EXCEPT:
A. Measles – Robeola virus.
B. German measles – RNA virus.
C. Scarlet fever – Gp B strept. Toxin.
D. Roseola infantum – herpes simplex virus.
E. Erythema infectiosum – parvovirus.

6. All of the following are correct incubation periods EXCEPT:


A. Measles 10 days.
B. Rubella 2-3 weeks.
C. Mumps 2-3 weeks.
D. Scarlet fever 6 days.
E. Chicken pox 11-20 days.
7-Vaccines can be given in all these conditions except:
A. Mild upper respiratory tract infection.
B. Two vaccines together.
C. Feeding.
D. Anaphylactic reaction to the vaccine.
E. Nasal block.

8.The proportion of patients in whom measles(rubeola)is complicated


byencephalitis is CLOSEST
A. 1 in 100
B. 1 in 1,000
C. 1 in 10,000
D. 1 in 100,000
E. 1 in 1,000,000

9. Pneumocystis carinii infection


a) occurs only in the immunosuppressed
b) responds to treatment with co-trimoxazole
c) can usually be diagnosed by examination of the sputem
d) may produce gaseous cysts in the wall of the colon
e) uncommon in cystic fibrosis

10. In measles
a) the incubation period is 1 week
b) splenomegaly is common
c) photophobia is infrequent
d) Koplik spots are common (>50%)
e) lymphadenopathy is marked

11. All of the following statements are true of pertussis (whooping cough)except
:
a) immunization is effective in preventing the disease in over 95% of immunized
people
b) for well premature infants the immunization should be carried out 2 months
after birth
c) erythromycin has been shown to inhibit the growth of the aetiological agent
in vitro
d) children under the age of 3 months are at risk from the disease
e) the incidence of permanent neurological complications from immunization is
less than 1:150,000

12.All are recognized complications for Scarlet fever EXCEPT :


A. Pancreatitis.
B. Otitis media
C. Pneumonia.
D. Meningitis.
E . Arthritis.

13. An infant is born to a mother who was seropositive for CMV infection prior
to pregnancy. The infant acquired the virus from maternal cervical secretions
during labor and delivery. Which of the following outcomes is most likely for
the infant?
A.Asymptomatic infection.
B.A prolonged mononucleosis-like illness.
C.Development in a few months of microcephaly, mental retardation, and
chorioretinitis.
D.Sensorineural hearing loss .

14. Parasitic manifestations in the third world countries is best characterized as:
A. Common cause of diarrhea.
B. Common cause of nutritional deficiencies.
C. Responsive to Vitamin A.
D. Responding to oral rehydration prevention
E. Preventable by immunizations.

15. Which one of the following statements is true about shigellosis?


A. Neurologic complications occur in about 40% of patients.
B. Animal reservoirs include poultry and shellfish.
C. Neomycin, orally, is the antibiotic of choice.
D. Bacteremia is common.
E. The disease is rare in institutions for the retarded.

16. A 4-year-old child was seen by his physician because of fever, cervical
lymphadenopathy, small ulcerative lesions on his buccal mucosa, and small
vesicles on his fingers. Which of the following examinations would be most
helpful in the diagnosis?
A. Serum IgG and IgM antirubella antibodies
B. IgM antibodies against measles virus in the serum
C. Throat and stool culture for enterovirus
D. Antistreptolysin-O titer in serum
E. Herpes simplex titers

17. Persistent jaundice with hypergammaglobulinemia, positive results of a


lupus erythematosus cell preparation, antiDNA antibodies, and an elevated
transaminase value are compatible with which disease?
A. Hepatitis A
B. Hepatitis B
C. Chronic persistent hepatitis
D. Chronic active hepatitis
E. Amebic abscess

18- L-Asparaginase side effect include:


a- drop foot.
b- cardiac toxicity
c- Hypofibrinogenaemia
d- Pulmonary fibrosis.
e- Mucositis.

19-Typhilitis is:
a- Is necrotizing colitis of the caecum
b- Is not associated with severe neutropenia.
c- Localized left lower quadrant abdominal pain.
d- Klebsiella species is the most organisms are involved.
e- CT scan may reveal thickening of rectum and sigmoid colon wall or
pneumatosis intestinalis in advanced cases.

20-Contraindication to the vaccines include:


A-Severe illness with fever.
B-Mild upper respiratory tract infection.
C-Feeding.
D-Two vaccines given together.
E-Prematurity.

21. Which of the following vaccines should NOT be given to children with severe
combined immunodeficiency syndrome:
A. DPT.
B. Measles vaccine
C. Salk polio vaccine.
D. Hep. B vaccine.
E. Pneumococcal vaccine.

22. In the Palestinian National Immunization Program, all of the following are
correct EXCEPT:
A. Salk vaccine is given at one and two months of age.
B. Hep. B vaccine is given at birth, one month and six months of age.
C. A booster dose of DPT is given at six years of age.
D. MMR vaccine is given at age of 15 months.
E. Rubella vaccine is given to girls at age of 12 years.

23. Inn BCG vaccine all are correct EXCEPT:


A. It is live attenuated vaccine.
B. It is given after birth.
C. It has nearly 80% protection against pulmonary T.B.
D. It has nearly 60% protection against systemic T.B.
E. It can be given to asymptomatic HIV infant.

24. All of the followings are correct for live attenuated vaccines EXCEPT:
A. Contraindicated in immunodeficient patients.
B. Given only after 3 months of stopping immunosuppressive drugs or large
steroid dose therapy.
C. Not given within 3 weeks before or 3 months after giving immunoglobulins.
D. Two Live attenuated vaccines are not given together.
E. Measles vaccine is not given to a patient with T.B except after

Question . 1. A 15-mo-old child is in your office with stridor at rest and cyanosis
with the presumptive diagnosis of viral croup. You should do all of the following
except:
A. Transport immediately in the parent's car to the nearest emergency
department.
B. Administer oxygen.
C. Administer dexamethasone
D. Administer racemic epinephrine
E. Transport after EMS personnel arrive 20 min later.

Question .4. A 12-yr-old presents with sneezing, clear rhinorrhea, and nasal
itching. Physical examination reveals boggy, pale nasal edema with a clear
discharge. The most likely diagnosis is:
A. Foreign body
B. Vasomotor rhinitis
C. Neutrophilic rhinitis
D. Nasal mastocytosis
E. Allergic rhinitis.

6. A 1-yr-old child presents with a history of recurrent fevers for several weeks.
On physical examination you find that the pupils fail to constrict. This finding is
associated with:
A. Toxoplasmosis
B. Ectodermal dysplasia
C. Juvenile rheumatoid arthritis
D. Hypothalamic dysfunction
E. Thyrotoxicosis

Question .7. A 9-yr-old boy presents with fever >39°C for 4 days,
myalgias, watery diarrhea, conjunctival infection, diffuse erythroderma,
strawberry tongue, blood pressure of 105/45mm Hg,
moderately elevated hepatic transaminases.
The most likely diagnosis is:
A. Staphylococcal scalded skin syndrome
B. Kawasaki disease
C. Toxic shock syndrome
D. Stevens-Johnson syndrome ( due to drugs or mycoplasma pneumonia
E. Toxic epidermal necrolysis

Question . 8. The recommended agent for treatment of pneumonia caused by


Mycoplasma pneumoniae is:
A. Amoxicillin or ampicillin
B. Ceftriaxone, cefotaxime, or cefotetan
C. Erythromycin, clarithromycin, or azithromycin
D. Gentamicin or kanamycin
E. Trimethoprim-sulfamethoxazole

Question . 10. Which of the following is associated with a poorer prognosis for
persons presenting with meningococcal disease?
A. Presence of petechiae for <12 hr (Meningococcemia )
B. Meningitis
C. Thrombocytosis
D. Leukocytosis
E. Low circulating levels of tumor necrosis factor

Question 11. Indications for cutting the oral frenulum include:


A. Short frenulum
B. Cleft palate
C. Cleft lip
D. Bifid uvula
E. None of the above

Question . 12. Immunologic diseases of the mother that may affect the fetus
include all of the following except:
A. Systemic lupus erythematosus
B. Idiopathic thrombocytopenic purpura years after splenectomy
C. Myasthenia gravis
D. Graves disease
E. Multiple sclerosis

Question . 13. Excessive intravenous fluid intake in very-low-birthweight infants


is associated with:
A. High BUN level
B. Bronchopulmonary dysplasia
C. Hyperbilirubinemia
D. Hypernatremia
E. Oliguria

Question . 14. Hypotension in a newborn suggests all of the following except:


A. Fetal-maternal hemorrhage
B. Hypoplastic left heart syndrome
C. Adrenogenital syndrome
D. Tension pneumothorax
E. Jaundice

Question . 15. Low levels of complement may be associated with all of the
following except:
A. Nephrotic syndrome
B. Vasculitis
C. Nephritis
D. Serum sickness
E. Systemic lupus erythematosus

Question . 2. A 4-wk-old female infant is presented to the emergency


department with hyponatremia and hyperkalemia. Other than the
cardiovascular abnormalities and dehydration, the physical findings are normal.
The most likely cause is:
A. New-onset diabetes mellitus type 2
B. Hypothyroidism
C. Hepatitis
D. Adrenal insufficiency
E. Celiac disease

Question . 23. All of the following statements about


hemolytic-uremic syndrome (HUS) are true except:
A. HUS is the most common cause of acute renal failure in young children
B. Verotoxin elaborated by Escherichia coli O157:H7 initiates endothelial cell
injury in HUS (shiga like toxin)
C. HUS always presents after an enteritis with diarrhea
D. The diagnosis of H US requires microangiopathic hemolytic anemia,
thrombocytopenia, and acute renal failure
E. Mortality from HUS is less than 10%.

Question . 24 The presence of renal parenchymal scarring due to


vesicoureteral reflux is best determined by:
A. DMSA scan
B. Renal ultrasonography
C. VCUG
D. CT scan
E. Intravenous pyelography
Question . 25. A 7-mo-old white male infant presents with failure to thrive and a
BUN of 75 mg/dL. He has a history of a poor urinary stream. The most likely
diagnosis is:
A. Renal artery stenosis
B. Renal hypoplasia
C. Urogenic bladder
D. Posterior urethral valves
E. Nephrolithiasis

Question . 26. To be effective for preventing myelomeningocele, administration


of folic acid needs to begin:
A. By 3 months of gestation
B. At the first missed period
C. Before conception
D. By 30 days of gestation
E. At 3 months prior to delivery

Question . 27. A 2-yr-old boy with the spastic diplegia form of cerebral palsy is
being evaluated. MRI of his brain is most likely to show:
A. Multicystic encephalomalacia
B. Periventricular leukomalacia
C. Normal anatomy
D. Basal ganglia abnormalities
E. Agenesis of the corpus callosum

Question . 28. - Acute bronchiolitis


A is caused by a single strain of respiratory synctial virus
B is readily transmitted by nursing staff
C is an indication for short-term prednisolone
D is a cause of hepatomegaly
E can be prevented by active immunization

Question . 29. - Coarctation of the aorta


A is usually proximal to the left subclavian artery (near to ductus
arteriosus)
B results in an pan systolic murmur audible in the interscapular area (if
significant collaterals are developed)
C is seldom symptomatic in school age children
D is an isolated disorder in approximately 90% of cases (10%)
E results in biventricular hypertrophy

Question .30. - Atopic eczema


A usually arises within the first month of life
B resolves by 10 years of age .
C is an indication for a cow's milk free diet
D contraindicates triple immunization
E contraindicates BCG vaccination

Question . 2. Autosomal dominance inheritance is characterized by all of the


following except:
A. It affects individuals in every generation
B. It has a high spontaneous mutation rate
C. It affects males more commonly than females
D. Phenotypically normal parents do not transmit the disease to the child
E. There is a 50% chance of transmission .

Question . 3. A baby is born with classic features of Down syndrome. Indications


for chromosome studies include:
A. Maternal age older than 35 yr
B. Family history of Down syndrome
C. Prospective adoption of the child
D. Maternal age younger than 35 yr
E. All of the above.

5. Pyelonephritis is a frequent cause of fever in young infants. All of the


following patient groups have an increased risk of urinary tract infections
except:
A. Infant girls
B. Uncircumcised infant boys
C. Infants with urinary tract anomalies
D. Infants with Mediterranean ancestors
E. Infants with vesicoureteral reflux.

Question . 9. A 7-mo-old girl presents with a temperature of 103.8°F, blood


pressure of 70/30 mm Hg, diffuse petechiae first noted 4 hr before
presentation, platelet count of 88,000/mm3, and white blood cell count of
4300/mm3, with 23% neutrophils and 42% bands. The infant has received all
recommended vaccinations. Which of the following is the most likely bacterial
etiology of this presentation?
A. Staphylococcus aureus
B. Streptococcus pneumoniae
C. Neisseria meningitidis
D. Haemophilus influenzae type b
E. Escherichia coli O157:H7

Question . 16. A 4-yr-old white girl has had joint swelling in multiple joints for
over 6 mo. She is slow to move in the morning and moves as if stiff for the first
hours of the day. Thereafter, she is a very active child. She has no rash and very
little limitation of range of motion. Her erythrocyte sedimentation rate is 4. The
most likely diagnosis is:
A. Hypermobility syndrome
B. Dermatomyositis
C. SLE
D. JRA
E. Henoch-Sch nlein purpura

Question . 17. All of the following statements regarding Kawasaki disease are
true except:
A. It occurs in outbreaks
B. Exposure to an affected patient greatly increases the
C. risk of the disease
D. Asian children have the highest risk
E. 80% of patients are younger than 5 yr of age
F. It is the leading cause of acquired heart disease in children in the United
States

Question . 23. An 11-yr-old boy complains of increasing headaches and recent


visual changes affecting his lateral fields of vision. MRI of the brain reveals a
midline mass measuring 2 cm in diameter in the region of the anterior pituitary
gland. He undergoes transsphenoidal resection of the mass and is transferred to
the pediatric intensive care unit for postoperative care. Six hours later he is
noted to have a brisk urine output of 4-6 mL/kg/hr. Vital signs are stable: heart
rate 120/min, blood pressure 120/75 mm Hg. He appears well hydrated. His
serum
osmolality is 310 mOsm/kg and urine osmolality is 100 mOsm/kg. Which of the
following is the most likely diagnosis?
A. SIADH
B. Cerebral salt wasting
C. Normal postoperative diuresis
D. Central diabetes insipidus
E. Nephrogenic diabetes insipidus

Question . 20. A 3-yr-old boy presents to your office with sudden onset of
colacolored urine, progressive facial swelling over the past 3 days, and
decreased urine volume over the past day. His examination is notable for blood
pressure 130/80 mm Hg, periorbital edema, bibasilar rales, and ankle swelling.
His urinalysis is remarkable for 3+ hematuria, 1+ proteinuria, 100 red blood cells
per high-power field, and red blood cell casts. His serum electrolytes are normal
and the serum albumin is 3.2 g/liter. This clinical presentation is most consistent
with:
A. Acute renal failure
B. Acute pyelonephritis
C. Nephrotic syndrome
D. Acute glomerulonephritis (edema – hematuria – proteinuria – high BP )
E. Chronic renal failure

Question . 21. A 15-yr-old boy with a 12-yr history of microscopic hematuria is


noted to have bilateral high-frequency sensorineural hearing loss, blood
pressure of 140/90 mm Hg, serum creatinine of 1.5 mg/dL, and urinary protein
of 2,000 mg/24 hr. This patient's mother also has microscopic hematuria. The
most likely mode of inheritance for this child's glomerular disease is:
A. Autosomal dominant with incomplete penetrance
B. X-linked dominant
C. Autosomal recessive
D. X-linked recessive
E. Autosomal dominant (Alport syndrome )

Question . 22. A 3-yr-old boy presents to an urgent care clinic with a 3-day
history of abdominal pain and difficulty walking. Abnormal findings include
blood pressure of 120/80 mm Hg, diffuse abdominal tenderness, purpuric rash
of the hands and ankles, and diffuse periarticular tenderness and swelling of the
ankles. The most likely diagnosis is:
A. Systemic lupus erythematosus
B. Kawasaki's disease
C. Juvenile rheumatoid arthritis
D. Henoch-Sch nlein purpura
E. Stevens-Johnson syndrome

Você também pode gostar